X



トップページ数学
1002コメント374KB
大学学部レベル質問スレ 20単位目
レス数が1000を超えています。これ以上書き込みはできません。
0001132人目の素数さん垢版2022/11/22(火) 12:15:20.37ID:aDS36Zer
大学で習う数学に関する質問を扱うスレ

・質問する前に教科書や参考書を読むなりググるなりして
・ただの計算は
http://wolframalpha.com
・数式の表記法は
http://mathmathmath.dote ra.net
・質問のマルチポストは非推奨
・煽り、荒らしはスルー

※前スレ
大学学部レベル質問スレ 19単位目
https://rio2016.5ch.net/test/read.cgi/math/1659623368/
0002132人目の素数さん垢版2022/11/23(水) 09:53:53.63ID:re4Vphli
>990 名前:132人目の素数さん[sage] 投稿日:2022/11/22(火) 23:53:41.80 ID:qlFg3LTl
>どうゆうこっちゃ?
>つまりΣ| axᵢ - yᵢ +b |²/(a²+b²)が最小になるa,b?
>
>993 名前:132人目の素数さん[] 投稿日:2022/11/23(水) 00:58:07.58 ID:62ydA4JG [2/2]
>>>990
>距離^2なら分母は1+a^2では?

993はΣ| axᵢ - yᵢ +b |²/(a²+1)が正しいのではって言ってるんだろ。
0003132人目の素数さん垢版2022/11/23(水) 13:39:55.67ID:LaN1UZyW
写像がwell-defined じゃなくてもいいじゃないか。俺は天才か?
0004132人目の素数さん垢版2022/11/23(水) 19:23:09.67ID:vW8BLQ30
デミング回帰だと決定係数は定義されてないらしい
相関係数の二乗とイコールにならないのかな?
0005132人目の素数さん垢版2022/11/26(土) 09:41:17.40ID:dhPfjN8j
Tom M. Apostol著『Mathematical Analysis 2nd Edition』

非常にクリアな説明でいい本だと思いますが、特に日本では、あまり話題になりませんね。
0006132人目の素数さん垢版2022/11/26(土) 11:44:52.13ID:pJdY2w+z
E.M.Steinの本ほどにはね
0007132人目の素数さん垢版2022/11/26(土) 13:19:00.53ID:dhPfjN8j
Tom M. Apostol著『Mathematical Analysis 2nd Edition』

S を R の開部分集合とする。(有限 or 無限)開区間 I は以下の条件を満たすとき、
S の component interval であるという:

(1) I ⊂ S
(2) I ⊂ J ⊂ S となるような I とは異なる開区間 J は存在しない。

定理3.11
R の空でない任意の開集合 S は S の可算無限個の、互いに共通部分のない、 component interval たちの
和集合として一意的に表される。

この定理がほとんどの微分積分の本や集合と位相の本に書かれていないのはなぜでしょうか?
0008132人目の素数さん垢版2022/11/26(土) 13:21:50.58ID:dhPfjN8j
この定理の系として、 R の開区間は連結であることがわかります。
0009132人目の素数さん垢版2022/11/29(火) 07:36:11.28ID:yRyy2icV
「Lindelöf covering theorem」は書いていない本が多いですが、役に立ちますか?
0010132人目の素数さん垢版2022/11/29(火) 17:52:48.02ID:Fea49eUi
リンデレフ空間(リンデレフくうかん、英: Lindelöf space; リンデレーフ空間)は、
任意の開被覆が可算部分被覆を持つような位相空間である。
リンデレフ性は、有限部分被覆の存在を要求するコンパクト性の概念を弱めたものである。
強リンデレフ空間 (strongly Lindelöf) あるいは遺伝的リンデレフ空間
(hereditarily Lindelöf) は任意の開集合がリンデレフ、
すなわち任意の部分空間にリンデレフ性が遺伝するような位相空間である。
0011132人目の素数さん垢版2022/11/30(水) 19:47:47.92ID:99OkqfWe
球面 S^2 上で三角不等式が成り立つことはどうやって証明するのでしょうか?
0012132人目の素数さん垢版2022/11/30(水) 19:50:21.71ID:99OkqfWe
距離空間 S^2 について詳しく書いてあるのはどんな本ですか?
0013132人目の素数さん垢版2022/11/30(水) 20:07:04.71ID:8sxpn/Sm
>>11
リーマン幾何の一般論による
0014132人目の素数さん垢版2022/11/30(水) 20:33:24.33ID:99OkqfWe
>>13
ありがとうございました。
0015132人目の素数さん垢版2022/12/01(木) 01:34:25.87ID:ntqUKS0A
数字で遊ぼ。って漫画はここの人達から見てどんな感想?
0016132人目の素数さん垢版2022/12/01(木) 14:22:05.36ID:cHtwEXAb
S^4→HP^5→OP^2というファイバー束が存在しない事を示せという問題を考えているのですが
ヒントにはp=3でのスチーンロッドのベキ作用素Pを使って示せるとあります。
球面束なのでGysin完全列を使うと射影p:HP^5→OP^2はコホモロジーでは
環H^*(OP^2)の生成元aをH^*(HP^5)の生成元bの2乗にうつすことはわかりましたが
この先で詰まっています。
S^2→CP^5→HP^2というファイバー束は存在するのでこの時は成り立つ議論が
上の一般化した束がもし存在するとすると成り立たないようになっているはずなのですが
分かる方いたら教えて下さい。
(出典はHatcherのAlgebraic Topologyのp.517の問題3です)
http://pi.math.cornell.edu/~hatcher/AT/AT+.pdf
0017132人目の素数さん垢版2022/12/01(木) 18:47:16.24ID:H52/tenW
>>16
セールスペクトラルシーケンスへのスチーンロッド作用の一般論使うんじゃ無いの?
0018132人目の素数さん垢版2022/12/01(木) 21:22:44.15ID:cHtwEXAb
>>17
スペクトル系列を解説する前の問題なので使わずに解けるんだろうとは思いますが
スペクトル系列使って分かるならそちらも知りたいです。
一般論というのは転入とスチーンロッド作用素が可換であるという事でしょうか。

まずそもそもH^*(OP^2)=Z_3[a]/(a^3)(|a|=8)やH^*(HP^5)=Z_3[b]/(b^6)(|b|=4)で
P(a)やP(b)がどうなるかというところからわかっておらず
Z_3係数では基本性質からP^4(a)=a^4=0やP^2(b)=b^3,P^3(b)=P^4(b)=0はわかりますが
それ以外のP^2(a)やP^1(b)がどうなるかがわからず困っています。
0019132人目の素数さん垢版2022/12/02(金) 15:14:35.81ID:o+gZK0RY
>>18
次数を見よ
0020132人目の素数さん垢版2022/12/02(金) 20:35:07.58ID:VR5SunJr
>>19
係数はZ_3なのでP^iが次元を2i(p-1)=4iだけ上げていて
そのため例えばP^2(a)は16次元でa^2と同じ次元となり
次数だけからはどうなるかが分からないように思えます。
他に次数の情報を使う方法があるのでしょうか
0021132人目の素数さん垢版2022/12/04(日) 18:53:14.04ID:or2l1ANr
C[n,k]^2=c_kとおいてn次の整式 f(x)=Σ(c_k)*x^k (k=0からnの和)とします。
0<r<1として、f(r)/f(1)→0 (n→∞) はいえますか。

C[n,k]^2じゃなくC[n,k]なら明らかですが
0023132人目の素数さん垢版2022/12/04(日) 20:50:47.56ID:9pTn/zu3
大田春外著『はじめよう位相空間』

有限個の点からなる位相空間 X = {x_1, …, x_n} がコンパクトであることを証明しています。
その証明は以下のような感じです:

X の任意の開被覆 U をとる。
各 x_i ∈ X に対して、 U の要素 V_i で、 x_i ∈ V_i となるようなものが存在する。
V_1, …, V_n は X の有限開被覆である。
X はコンパクトである。

でも X は有限集合なので、 X の開集合の個数も有限個しかないはずです。
ですので、 X がコンパクトなのは自明で、証明するまでもないことです。
0024132人目の素数さん垢版2022/12/04(日) 23:13:01.53ID:2HXiSFuF
>>23
具体的に開被服作って見せたってことでしょうね
0025132人目の素数さん垢版2022/12/04(日) 23:15:46.59ID:2HXiSFuF
開集合が有限個でも開被覆構成する開集合は無限個のこともあり得るからということでしょうね
有限個だから証明不要とは自分もそう思いますが
0026132人目の素数さん垢版2022/12/05(月) 09:26:37.96ID:n6CZDJQU
>>22
A、Bってどうゆうことですか
0027132人目の素数さん垢版2022/12/05(月) 10:22:26.56ID:36HivrxM
>>26
22はあまり厳密な議論ではない。
0028132人目の素数さん垢版2022/12/05(月) 10:33:36.54ID:Bbn8ovl9
雑なだけならまだマシだけど
0029132人目の素数さん垢版2022/12/05(月) 12:24:00.95ID:1Z1B77zR
Tom M. Apostol著『Mathematical Analysis Second Edition』

基本的にいい本だとは思うのですが、ちょっと雑なところを見つけてしまいました。
0030132人目の素数さん垢版2022/12/05(月) 12:53:27.52ID:1Z1B77zR
線積分について詳しく書いてある本を教えてください。
0031132人目の素数さん垢版2022/12/05(月) 13:02:16.74ID:36HivrxM
グリーンの定理の証明が書いてある本なら
みんな詳しく書いてある。
0032132人目の素数さん垢版2022/12/05(月) 13:05:07.74ID:1Z1B77zR
>>31
ありがとうございました。
具体的に書名を挙げることはできますか?

岩堀さんのベクトル解析の本はどうですか?
0033132人目の素数さん垢版2022/12/05(月) 13:59:41.31ID:36HivrxM
>>32
それを見てわかりにくいところがあれば
別の本を推薦したいと思います。
0034132人目の素数さん垢版2022/12/06(火) 11:53:56.74ID:y5tkMCpm
X, Y を有限な位相空間とする。

X から Y への連続な全単射が存在すれば、 X と Y は位相同型であるか?
位相同型であるならば、それを証明せよ。
位相同型でないならば、 X から Y への連続な全単射が存在するが、 X と Y は位相同型でないような
X, Y の例を挙げよ。
0035132人目の素数さん垢版2022/12/06(火) 12:07:42.40ID:y5tkMCpm
X = {a, b, c}
T_X = {∅, X, {a}, {b, c}}

Y = {1, 2, 3}
T_Y = {∅, Y, {1}}

f(a) = 1
f(b) = 2
f(c) = 3

f は X から Y への全単射で連続であるが、 f^{-1} は連続ではない。
0036132人目の素数さん垢版2022/12/06(火) 12:20:41.62ID:FKh1L/i8
同じ空間で粗い位相と細かい位相を入れると、集合としての恒等写像は片方は連続に、もう片方は不連続になる。
上の書き込みには頭の悪さしか感じない。
0037132人目の素数さん垢版2022/12/06(火) 12:37:24.54ID:y5tkMCpm
>>33

岩堀さんの本は非常に古いせいか分かりにくかったです。
一様連続性が重要だということは分かりました。
0038132人目の素数さん垢版2022/12/06(火) 12:55:41.10ID:y5tkMCpm
一様連続性が重要だというのは、リーマン和による線積分の定義においてです。
0039132人目の素数さん垢版2022/12/06(火) 12:56:01.20ID:iE3s/xAS
>>37
ストークスの定理の応用として示してありますね。
分かりにくいとしたらそのせいではないかと思います。
グリーンの定理以外のところも読みたいと思ったら
古さが気になるかもしれませんが。
溝畑先生の本は古いのから新しいのまでありますが
「微分積分学」(学術図書)というのがお勧めです。
岡村先生の講義をもとにしていますから岩堀先生の本よりずっと古いですが、こっちの方が多分わかりやすいでしょう。
0040132人目の素数さん垢版2022/12/06(火) 12:57:35.96ID:y5tkMCpm
>>39

ありがとうございました。
溝畑さんのその本を見てみます。
0041132人目の素数さん垢版2022/12/06(火) 17:53:18.33ID:zKho/eSn
>>34,36
離散位相と密着位相ね
0042132人目の素数さん垢版2022/12/06(火) 19:10:32.55ID:sPCYEpjA
実数の連続性の同値条件に関する質問です。
有名な同値条件は、下記の4つでしょう。
1)デデキントの切断
2)上に有界な集合の上限の存在
3)上に有界な単調増加数列の収束
4)区間縮小法の原理+アルキメデスの公理
なぜ、4)だけアルキメデスの公理が
必要なのでしょうか。
実数に順序構造と四則演算を認めれば、
アルキメデスの公理が成り立つのは、
当たり前に思えます。
区間縮小法の場合だけ、
なぜアルキメデス性にこだわるのでしょうか。
0043132人目の素数さん垢版2022/12/06(火) 20:51:52.96ID:zilW6Si8
>>実数に順序構造と四則演算を認めれば、
>>アルキメデスの公理が成り立つのは、
>>当たり前に思えます。

では証明をどうぞ
0044132人目の素数さん垢版2022/12/07(水) 11:36:10.97ID:lUIrRHgJ
Springerのセールのときに注文した、以下の本が届きました。

Ebbinghausらの本を読んだことがある人はいますか?

Mathematical Logic (Graduate Texts in Mathematics, 291) 3rd ed. 2021 Edition
by Heinz-Dieter Ebbinghaus (Author), Jörg Flum (Author), Wolfgang Thomas (Author)

Real Mathematical Analysis (Undergraduate Texts in Mathematics) 2nd ed. 2015, Corr. 2nd printing 2017 Edition
by Charles Chapman Pugh (Author)

Number Fields (Universitext) 2nd ed. 2018 Edition
by Daniel A. Marcus (Author)
0045132人目の素数さん垢版2022/12/07(水) 11:39:15.45ID:lUIrRHgJ
Amazonで買うよりもセールのときにSpringerで直接買うほうがいいです。

今回注文した本は、オンデマンド印刷ですが、発送元が埼玉県の大日本印刷だったので、
おそらく印刷は大日本印刷でされたものだと思います。

コンディションはパーフェクトでした。
0046132人目の素数さん垢版2022/12/07(水) 20:44:24.51ID:cKt4NobS
>>21 をもういちど教えてください。
>>22 だと馬鹿には理解できないです。
0047132人目の素数さん垢版2022/12/07(水) 22:20:05.28ID:eWHkXKCj
>>46
22がバカ
0048132人目の素数さん垢版2022/12/08(木) 06:54:24.88ID:mckES4hm
「r^Nが小さくなるように」ってのがすごく謎だけど、例えば 0<a<1/2, N=an でぶった切って
・前半 A/(A+B)はStirlingの評価式を使って->0
・後半 r^kはそのまんま->0
みたいなことかもしれない予感な気がしないわけでもない
0049132人目の素数さん垢版2022/12/08(木) 07:33:53.81ID:BybxetFD
急激に増えていくんだから大雑把にAの最後の項とBの最初の項を使うだけでいい
0050132人目の素数さん垢版2022/12/08(木) 08:42:36.49ID:xpFZils6
今はそういう議論が一般的なわけ?
0051132人目の素数さん垢版2022/12/08(木) 08:56:48.90ID:BALjvuN9
A<=(Aの最後の項)(項数)も(Bの最初の項)<=Bも厳密で昔から使われている方法だが
0052132人目の素数さん垢版2022/12/08(木) 10:59:31.60ID:pbvBLD/z
だからAとBって何なの
0053132人目の素数さん垢版2022/12/08(木) 23:50:25.66ID:4IEw09R5
>>39
ISBNを明示してもらいたい。

>>岡村先生の講義をもとにしていますから
岩波全書の「ルベーグ積分」と混同していないか?
0055132人目の素数さん垢版2022/12/09(金) 08:36:42.00ID:lK+WckRr
>>53
ISBN978-4-87361-139-6
定理7.4の証明は
「ルベーグ積分」の定理4.11を
一年生向けにやさしく述べている。
0056132人目の素数さん垢版2022/12/09(金) 12:21:48.94ID:P2rGQfxL
>>48
なにをぶった切るんですか。
aとかNは何ですか?
0057132人目の素数さん垢版2022/12/09(金) 12:31:43.99ID:XK8g+YUj
小出しで尋ねたりしないで、完璧な解答をのみを要求したらどうでしょうか
0058132人目の素数さん垢版2022/12/09(金) 13:01:15.90ID:P2rGQfxL
>>21 の解答をお願いします。

いままでいくつか解説?されてますが
新しい文字を使うときはその定義をしてもらわないと読めませんので
宜しくお願いします。
0059132人目の素数さん垢版2022/12/09(金) 16:37:29.36ID:TdZ8olG0
>>21
f_n(x)=Σ[k=0〜n] C[n,k]^2 x^k と置く。
0<r<1とする。lim[n→∞] f_n(r)/f_n(1) = 0 を示したい。

M≧1を任意に取る。n>Mのとき

f_n(r) = Σ[k=0〜M]C[n,k]^2 r^k+Σ[k=M+1〜n]C[n,k]^2 r^k

≦ Σ[k=0〜M]C[n,k]^2 + Σ[k=M+1〜n]C[n,k]^2 r^{M+1}

≦ Σ[k=0〜M]C[n,k]^2 + Σ[k=0〜n]C[n,k]^2 r^{M+1}

であるから、

f_n(r) / Σ[k=0〜n]C[n,k]^2

≦ ( (Σ[k=0〜M]C[n,k]^2) / (Σ[k=0〜n]C[n,k]^2) ) + r^{M+1}

≦ ( (Σ[k=0〜M]C[n,k]^2) / C[n,M+1]^2 ) + r^{M+1}

= Σ[k=0〜M] (C[n,k] / C[n,M+1])^2 + r^{M+1}

となる。
0060132人目の素数さん垢版2022/12/09(金) 16:39:34.91ID:TdZ8olG0
すなわち、

f_n(r)/f_n(1) ≦ Σ[k=0〜M] (C[n,k] / C[n,M+1])^2 + r^{M+1} … (1)

となる。各 k∈{0,1,…,M} に対して lim[n→∞] C[n,k] / C[n,M+1] = 0 であるから、
(1)でlimsup[n→∞]を取って、

limsup[n→∞] f_n(r)/f_n(1) ≦ r^{M+1} … (2)

となる。M≧1 は任意だったから、(2)でlim[M→∞]を取って

limsup[n→∞] f_n(r)/f_n(1) ≦ 0

となる。f_n(r)/f_n(1) は非負だから、以上より、lim[n→∞] f_n(r)/f_n(1) = 0 である。
0061132人目の素数さん垢版2022/12/09(金) 16:50:00.63ID:ajaSMxdU
問題(もちろん超簡単)と解答を事前に発表して、激アマ採点をするくらいでいいんじゃないかな
こんなに厳しかったら鬼扱いされるだろうけど
0062132人目の素数さん垢版2022/12/09(金) 17:03:44.27ID:pZ+cIqKL
おみごと
0063132人目の素数さん垢版2022/12/09(金) 19:09:54.61ID:P2rGQfxL
おおおありがとうござします
0064132人目の素数さん垢版2022/12/09(金) 19:39:57.28ID:UtJOaLdJ
>>59
前提を間違えてる
0065132人目の素数さん垢版2022/12/09(金) 19:43:23.63ID:lK+WckRr
>>64
問題そのものを間違えているという意味?
それとも解答がでたらめで話にならない?
0066132人目の素数さん垢版2022/12/09(金) 22:12:25.74ID:P2rGQfxL
係数の条件として
>>60 の3行目の、k<M+1ならC[n,k]/C[n,M+1]→0 (n→∞) という部分が要
ということでしょうか。

つまり 一般にg_n(x)=Σ[k=0〜n]c_k*x^k において、係数c_kが
 c_k>0かつ  k<mなら (c_k)/(c_m)→0(n to infty)
を満たしていれば、g_n(x)についても同様のことが示せるということでしょうか。
0067132人目の素数さん垢版2022/12/09(金) 22:49:33.66ID:lK+WckRr
>>66
全くその通り。
結局、そこに気づくかどうかだったわけですね。
0068132人目の素数さん垢版2022/12/10(土) 18:43:33.89ID:MGy2MlKW
S を位相空間とする。
M を S の部分集合とする。
M の閉包の内部が M の内部に含まれないような例ってありますか?
0069132人目の素数さん垢版2022/12/10(土) 18:52:33.47ID:vYdhbWp4
>>68

S = R, M = R - {0}
0070132人目の素数さん垢版2022/12/10(土) 18:55:10.92ID:MGy2MlKW
>>69

ありがとうございました。
0072132人目の素数さん垢版2022/12/10(土) 21:54:43.32ID:DV2XUKqW
ぜひ原文の3まで翻訳を進めてください
0073132人目の素数さん垢版2022/12/10(土) 22:09:33.14ID:zGfmyAMq
数学的には1の翻訳で完結してると思いますが
0074132人目の素数さん垢版2022/12/10(土) 22:13:44.91ID:zGfmyAMq
質問はつまり、数学ネイティブの人にとってもこれは面白そうかどうかです
0075132人目の素数さん垢版2022/12/10(土) 22:15:44.99ID:DV2XUKqW
計算、論理、そして数学というまとまりが大切です。
1だけでは4値論理とは何であるかというだけで
論理的には完結しているのかもしれませんが
その数学的な実体というものは
これによってどんな問題が解決されたかについての
記述がないと不完全であると言わざるを得ません。
ぜひ原文の完全な翻訳を期待します。
0076132人目の素数さん垢版2022/12/10(土) 22:31:15.41ID:cCnzIf6U
最初の方のベルナップのセリフの訳がめちゃくちゃなとこまで読んでやめた
0077132人目の素数さん垢版2022/12/10(土) 22:47:19.53ID:zGfmyAMq
>>75
いや、一般受けは既に良いのです
矛盾許容論理や多値論理等が既出だった世界だとどうなのかな、という話です
0078132人目の素数さん垢版2022/12/10(土) 22:58:59.29ID:zGfmyAMq
矛盾許容論理、多値論理、
「ああ、その一つが登場したのね」ってとこでしょう
どの程度の「その一つ」だと思うか、です
0079132人目の素数さん垢版2022/12/10(土) 23:08:07.81ID:zGfmyAMq
こっちがおかしいのかな
数学的営為ってもっと自己目的化してるイメージありますけど
0080132人目の素数さん垢版2022/12/11(日) 06:16:22.07ID:SdZ8e8cF
あの、じゃあ質問変えます
「面白そう」と、一般受けはなかなかいいのですが
数学的にはどういった位置づけになるでしょうか

https://en.wikipedia.org/wiki/Four-valued_logic
0082132人目の素数さん垢版2022/12/11(日) 07:01:11.56ID:SdZ8e8cF
>>72,75,76
結局、何も答えてもらってない
0083132人目の素数さん垢版2022/12/11(日) 07:27:31.42ID:lxcHhNkX
多値論理が出てきたときは
現れるべきものがようやく出て来たかという印象を持ちましたが
一般にはファジーという言葉を流行らせただけのように思います。
4値論理によってはじめて多値論理にふれた人の「受けが良い」のは
当然かと思いますが、私などは4値論理のどこが特別の興味に
値するのかということが気になります。
最初のポイントに戻ります。ハーン・バナッハの拡張定理というのが
函数解析の初歩の部分にありますが、ウィキペディアでは
「関数解析学の分野における中心的な道具で」と書き始められています。
どんな役に立つのかという話を落とすのは良くありません。
0084132人目の素数さん垢版2022/12/11(日) 08:37:58.28ID:SdZ8e8cF
意味ありげなのはこの部分です。他はまあ普通の論理学の話だと
思います。哲学的な意味でもあるかもしれないしないかもしれない

Belnap has this interpretation: "The worst thing is to be told
something is false simpliciter. You are better off (it is one of
your hopes) in either being told nothing about it, or being told
both that it is true and also that it is false; while of course
best of all is to be told that it is true."
ベルナップは次のように解釈する。「最悪なのは何かがFの単純化
であるとみなされることです。あなたはそれについては何も言わず
去った方がいいでしょう(それはあなたの希望の一つです)。あるいは、
BothとはすなわちTでもFでもあることだ、と言うこともできます。
しかしそれは単にTであるとする方がもちろん何よりも良いことです」。
0085132人目の素数さん垢版2022/12/11(日) 08:42:38.59ID:lxcHhNkX
>>84
意味ありげではありますが
役に立つかどうか以前に
訳になっていませんね。
0086132人目の素数さん垢版2022/12/11(日) 09:05:49.77ID:SdZ8e8cF
>>85
真理値表を見ればいいじゃないですか
0087132人目の素数さん垢版2022/12/11(日) 10:10:32.41ID:OAqohdIz
3値論理もいろいろあるがハイチング代数であるものは1つ
それは直観主義者論理に深く関わるという意味で重要
何をどう定義しても自由だけど
(数学的に)意味が自然だとか(数学的に)応用ができるとか
何かないと無視されるだけよな
0088132人目の素数さん垢版2022/12/11(日) 10:56:04.24ID:G4G3fajU
なんか或る種の矛盾は
ロジックを一周してくると真偽値が反転する発振回路に思える。

まるでメビウスの輪を一周してきてもう一周しないと元に戻らんようなスピノールに値を持つ。
0089132人目の素数さん垢版2022/12/11(日) 11:40:39.57ID:OAqohdIz
>>88
>ロジックを一周してくると真偽値が反転する発振回路に思える。
真でも偽でもある
真でも偽でもない
も認める立場なんだろな
0090132人目の素数さん垢版2022/12/11(日) 11:42:53.96ID:OAqohdIz
>>88
>まるでメビウスの輪を一周してきてもう一周しないと元に戻らんようなスピノールに値を持つ。
ああそうか
ダブルカバーで底空間を拡張すると考えると
命題というものの存在空間を拡張すれば良いのかな
0091132人目の素数さん垢版2022/12/11(日) 14:14:45.62ID:AOl08Lj9
松坂和夫著『集合・位相入門』pp.167-168

S を空でない1つの集合とする。
M を 2^S の任意の部分集合とする。

M に属する有限個の集合の共通部分

∩_{i∈I} A_i (A_i ∈ M, I は有限集合)

として表される S の部分集合の全体を考え、それを M_0 とする。

---------------------------------------------------------------------
M は 2^S の任意の部分集合と書いてあるので、 M が空集合の場合も
考えなければなりません。

I も有限集合なので、それが空集合である場合も考えなければなりません。

(1) M が空集合で、 I が空集合である場合。
(2) M が非空集合で、 I が空集合である場合。
(3) M が空集合で、 I が非空集合である場合。
(4) M が非空集合で、 I が非空集合である場合。

(4)は問題ありません。
(2)の場合には、 ∩_{i∈I} A_i = S でしょう。
(3), (4)の場合には、どう考えるのでしょうか?
0092132人目の素数さん垢版2022/12/11(日) 14:16:40.90ID:AOl08Lj9
松坂和夫著『集合・位相入門』pp.167-168

S を空でない1つの集合とする。
M を 2^S の任意の部分集合とする。

M に属する有限個の集合の共通部分

∩_{i∈I} A_i (A_i ∈ M, I は有限集合)

として表される S の部分集合の全体を考え、それを M_0 とする。

---------------------------------------------------------------------
M は 2^S の任意の部分集合と書いてあるので、 M が空集合の場合も
考えなければなりません。
訂正します:

I も有限集合なので、それが空集合である場合も考えなければなりません。

(1) M が空集合で、 I が空集合である場合。
(2) M が非空集合で、 I が空集合である場合。
(3) M が空集合で、 I が非空集合である場合。
(4) M が非空集合で、 I が非空集合である場合。

(4)は問題ありません。
(2)の場合には、 ∩_{i∈I} A_i = S でしょう。
(1), (4)の場合には、どう考えるのでしょうか?
0093132人目の素数さん垢版2022/12/11(日) 14:17:26.97ID:AOl08Lj9
>>91-92

訂正します:

松坂和夫著『集合・位相入門』pp.167-168

S を空でない1つの集合とする。
M を 2^S の任意の部分集合とする。

M に属する有限個の集合の共通部分

∩_{i∈I} A_i (A_i ∈ M, I は有限集合)

として表される S の部分集合の全体を考え、それを M_0 とする。

---------------------------------------------------------------------
M は 2^S の任意の部分集合と書いてあるので、 M が空集合の場合も
考えなければなりません。

I も有限集合なので、それが空集合である場合も考えなければなりません。

(1) M が空集合で、 I が空集合である場合。
(2) M が非空集合で、 I が空集合である場合。
(3) M が空集合で、 I が非空集合である場合。
(4) M が非空集合で、 I が非空集合である場合。

(4)は問題ありません。
(2)の場合には、 ∩_{i∈I} A_i = S でしょう。
(1), (3)の場合には、どう考えるのでしょうか?
0094132人目の素数さん垢版2022/12/11(日) 14:21:39.28ID:SdZ8e8cF
>>90
当てずっぽうですが命題というものの存在空間の拡張と言うと、
例えばある命題が「宇宙の外において真」だったりするのですか?
0095132人目の素数さん垢版2022/12/11(日) 14:26:06.43ID:AOl08Lj9
松坂和夫著『集合・位相入門』pp.298-299

「次に、 A または B が空集合である場合、 A から B への(一意的に存在する)対応 Γ
が写像であるかどうかを考えてみよう。 A = ∅ である場合には、p.27に挙げた写像の条件(*)
はtrivialに満足されるから、 Γ は写像であると考えられる。しかし A ≠ ∅, B = ∅ の場合には、
上記の条件(*)はもちろん満足されないから、 Γ は写像ではない。したがって結局、次のように
述べることができる:”A = ∅ ならば、(B が何であっても)、 A から B への写像はただ1つだけ
存在する。また A ≠ ∅, B = ∅ ならば、 A から B への写像は存在しない。”
0096132人目の素数さん垢版2022/12/11(日) 14:27:10.39ID:SdZ8e8cF
翻訳記事には登場しませんが、哲学板で相談したら
「真、偽、両、無」という訳語が妥当っぽいです
0097132人目の素数さん垢版2022/12/11(日) 14:27:29.30ID:AOl08Lj9
(3)の場合が問題になりそうです。
0098132人目の素数さん垢版2022/12/11(日) 14:46:41.49ID:OAqohdIz
>>91
M_0={∩N|N⊂M,#N<∞}
∩N={x∈S|∀A∈N(x∈A)}={x∈S|∀A(A∈N→x∈A)}
∩φ=S
∩{φ}=φ
>(1) M が空集合で、 I が空集合である場合。
{S}
>(2) M が非空集合で、 I が空集合である場合。
{S}
>(3) M が空集合で、 I が非空集合である場合。
{S}
(5)N={φ}
{φ}
0099132人目の素数さん垢版2022/12/11(日) 14:47:22.46ID:OAqohdIz
>>94
さあ
0100132人目の素数さん垢版2022/12/11(日) 15:00:37.05ID:SdZ8e8cF
>>99
意地悪しないで教えてください
「宇宙の外において真」という真理値があるならそれは「真、偽、両、無」のどれだろう、
というレベルの事しか考えてませんので
0101132人目の素数さん垢版2022/12/11(日) 16:05:52.13ID:AOl08Lj9
>>98

ありがとうございました。

松坂和夫著『集合・位相入門』では、写像の前に、「対応」というのを定義しています。
対応の特別なものが写像という流れです。
写像を対応の特別なものと考えるやり方は標準的ですか?

例えば、斎藤毅著『集合と位相』では「対応」は登場しません。
0102132人目の素数さん垢版2022/12/11(日) 17:40:21.71ID:AOl08Lj9
>>98

p.168に、M_0 に属する任意個数の集合の和集合

∪_{λ ∈ Λ} B_λ (B_λ ∈ M_0, Λ の濃度は任意)

としてあらわされる S の部分集合の全体が S における位相になると書いてあります。

この集合が(Oiii)を満たすことは定義から明らかだとしていますが、どう証明するのでしょうか?
0103132人目の素数さん垢版2022/12/11(日) 17:43:45.44ID:AOl08Lj9
>>98

M に属する有限個の集合の共通部分

∩_{i∈I} A_i (A_i ∈ M, I は有限集合)

として表される S の部分集合の全体を考え、それを M_0 とする。

I が非空集合で、 M が空集合の場合には、 A_i という写像は存在しないため、
∩_{i∈I} A_i (A_i ∈ M, I は有限集合)自体がナンセンスではないでしょうか?
0104132人目の素数さん垢版2022/12/11(日) 20:14:53.15ID:AOl08Lj9
>>103

M が空集合の場合は、個別に考えて、 O(M) は密着位相になると書くべきですね。
0105132人目の素数さん垢版2022/12/11(日) 22:50:50.25ID:OAqohdIz
>>100
イジワルじゃなくて
さあ
0106132人目の素数さん垢版2022/12/11(日) 22:52:33.68ID:OAqohdIz
>>101
写像は関係の特別なものと考えるのが普通
関係は直積の部分集合とするのが普通
対応というのはおそらく関係のことでは?
0107132人目の素数さん垢版2022/12/11(日) 22:52:55.92ID:OAqohdIz
>>102
>(Oiii)
とは?
0108132人目の素数さん垢版2022/12/11(日) 22:58:29.86ID:OAqohdIz
>>103
Aiが存在しないので∩φ=S
0109132人目の素数さん垢版2022/12/11(日) 23:04:52.23ID:OAqohdIz
>>103
>I が非空集合で、 M が空集合の場合には、 A_i という写像は存在しないため、
{Ai|i∈I}, #I<∞よりN⊂M, #N<∞の方が考えやすいと思う
前者ならi≠jでAi=Ajの場合も入れることができるというだけ
∩{Ai|i∈I}考えるのに重複は1つに考えて良いから後者の方がMが空の場合に誤解もしにくい
0110132人目の素数さん垢版2022/12/11(日) 23:31:35.90ID:AOl08Lj9
>>106,108-109

ありがとうございました。

>>107
開集合の和集合は開集合になるという条件です。
0111132人目の素数さん垢版2022/12/11(日) 23:34:17.49ID:AOl08Lj9
∪_{λ∈Λ} (∪_{μ∈M_λ} B_μ) が開集合になることはどう示すかという話です。

2重の「∪」を1重の「∪」にしないとだめですよね?
0112132人目の素数さん垢版2022/12/11(日) 23:36:16.38ID:AOl08Lj9
訂正します:

∪_{λ∈Λ} (∪_{μ∈M_λ} B_μ) が開集合の条件(Oiii)を満たすことをどう示すかという話です。

2重の「∪」を1重の「∪」にしないとだめですよね?
0113132人目の素数さん垢版2022/12/11(日) 23:47:28.11ID:OAqohdIz
>>110
>開集合の和集合は開集合になるという条件です。
ならばホボ自明
ΛとΛ'による合併はΛ+Λ'(直和)をインデックスにするだけでしょ
0114132人目の素数さん垢版2022/12/11(日) 23:49:22.87ID:OAqohdIz
無限個でも直和で
0115132人目の素数さん垢版2022/12/11(日) 23:50:44.34ID:OAqohdIz
こういう場合はN⊂MじゃなくてAi,i∈Iの方がいいね
0116132人目の素数さん垢版2022/12/12(月) 10:34:28.37ID:LwnfEzBw
>>113-115

ありがとうございました。

>>113
∪_{λ∈Λ} (∪_{μ∈M_λ} B_μ)

M_λ が λ に依存しているので、1重の「∪」にできないのですが、どうすればいいのでしょうか?
0117132人目の素数さん垢版2022/12/12(月) 13:04:31.06ID:3B4BE7/S
>>105
てゆうか、想像つきますよ
メビウスの輪の、ある場所に「真」があって、
180度回ると「真でも偽でもある」がある
さらに180度回ると「偽」があって
さらに180度回ると「真でも偽でもない」がある
そしてさらに180度回ると「真」に戻ってくる
0118132人目の素数さん垢版2022/12/12(月) 13:48:13.50ID:3B4BE7/S
この真理値表は、しかし
¬Both = Both および ¬None = None
としたから意味が生まれたんじゃないでしょうか
0119132人目の素数さん垢版2022/12/12(月) 14:33:56.09ID:3B4BE7/S
使っていいのかどうか知りませんが、ド・モルガンの定理もクリアしてそうに見えます

B&B=(~B)&(~B)=~(B|B)=~B=B

B&F=(~B)&F=~(B|~F)=~(B|T)=F

B&T=(~B)&T=~(B|~T)=~(B|F)=B

B&N=(~B)&N=~(B|~N)=~(B|N)=F
0120132人目の素数さん垢版2022/12/12(月) 14:35:43.60ID:4G2c6HXU
>>116
M1={a,b,c}
M2={p,q,r,s}
M3={x,y}

M1+M2+M3={a,b,c,p,q,r,s,x,y}
にしますよ
0121132人目の素数さん垢版2022/12/13(火) 06:40:08.33ID:FLA8AhkN
>>120

ありがとうございました。

∪_{λ∈Λ} (∪_{μ∈M_λ} B_μ)
=
∪_{μ ∈ ∪_{λ∈Λ} M_λ} B_μ

ということですね。
0122132人目の素数さん垢版2022/12/13(火) 08:55:07.03ID:FLA8AhkN
松坂和夫著『集合・位相入門』

p.170 定理15
T を S における1つの位相とするとき、その部分集合 B が T の基底となるためには、任意の
O ∈ T および任意の x ∈ O に対して、

x ∈ W, W ⊂ O

となるような W ∈ B が存在することが必要十分である。
0123132人目の素数さん垢版2022/12/13(火) 09:24:50.30ID:FLA8AhkN
B が T の基底であれば、任意の O ∈ T に対して、

O = ∪_{λ∈Λ} W_λ (W_λ ∈ B)と書ける。

x ∈ O に対して、

∃λ ∈ Λ such that x ∈ W_λ (⊂ O)

逆に、

任意の O ∈ T および任意の x ∈ O に対して、

x ∈ W, W ⊂ O

となるような W ∈ B が存在すると仮定する。

O ∈ T とする。
x ∈ O とする。
仮定により、 {W ∈ B | x ∈ W, W ⊂ O} は空集合ではない。
選出公理により、以下の写像 a が存在する。

a : O ∋ x → a(x) ∈ A_x := {W ∈ B | x ∈ W, W ⊂ O}

明らかに、 ∪_{x∈O} a(x) = O である。

よって、 B は T の基底である。
0124132人目の素数さん垢版2022/12/13(火) 09:26:09.77ID:FLA8AhkN
選出公理は使う必要がありますか?
0125132人目の素数さん垢版2022/12/13(火) 09:35:07.70ID:bQIwX/Ff
使うんじゃない?
0127132人目の素数さん垢版2022/12/13(火) 21:34:07.01ID:FLA8AhkN
松坂和夫著『集合・位相入門』

(1)と(2)ではどちらが良い書き方でしょうか?
(1)は分かりにくくないですか?分かりにくい理由は、T = T(M) ならば必然的に M は T の部分集合になるにもかかわらず、
T は M の部分集合であると仮定しているからです。

(1) S における1つの位相 T が与えられたとき、 T のある部分集合 M に対して
T = T(M) が成り立つならば、 M は位相 T の準基底であるという。

(2) S における1つの位相 T が与えられたとき、 2^S の部分集合 M に対して
T = T(M) が成り立つならば、 M は位相 T の準基底であるという。
0128132人目の素数さん垢版2022/12/13(火) 21:41:07.88ID:FLA8AhkN
(1)と(2)はどちらのほうが良い書き方ですか?
↑と同様の理由で、(1)は分かりにくいと思います。

(1) B が位相 T の部分集合で、 T の任意の元 O は、 O に含まれる B の元の和集合として

O = ∪_{λ∈Λ} W_λ, W_λ ∈ B

と表されるとき、 B は T の基底であるという。

(2) B が位相 T の部分集合で、 T の任意の元 O は、 B の元の和集合として

O = ∪_{λ∈Λ} W_λ, W_λ ∈ B

と表されるとき、 B は T の基底であるという。
0129132人目の素数さん垢版2022/12/13(火) 22:04:59.19ID:FLA8AhkN
一番いいのは以下の(3)だと思います。

(3) B ∈ 2^{2^S} とする。 位相 T が

T = {W ∈ 2^S | W = ∪_{λ∈Λ} W_λ, W_λ ∈ B}

と表されるとき、 B は T の基底であるという。
0130132人目の素数さん垢版2022/12/13(火) 22:10:28.84ID:FLA8AhkN
あと、気になったこととして、 B ∈ 2^{2^S} が基底となるような S の位相は一意的に存在するという
ことも自明なことですが、書いたほうが良くないですか?
0132132人目の素数さん垢版2022/12/13(火) 22:35:29.91ID:FLA8AhkN
>>131

そうですね。
0133132人目の素数さん垢版2022/12/13(火) 22:45:52.04ID:FLA8AhkN
p.174 問題3.

S を空でない集合とするとき、 2^S の部分集合 B が O(B) の基底となるためには、
B が次の性質(O*i)および(O*ii)をもつことが必要十分であることを証明せよ。

これも

S を空でない集合とするとき、 2^S の部分集合 B が S のある位相 O の基底となるためには、
B が次の性質(O*i)および(O*ii)をもつことが必要十分であることを証明せよ。

のほうが良いと思います。
0134132人目の素数さん垢版2022/12/14(水) 07:59:05.67ID:HhU2DiVl
どうでもよい感じ>>128.133
0135132人目の素数さん垢版2022/12/14(水) 23:47:38.53ID:HhU2DiVl
どうでもよい感じ>>128,133
0136132人目の素数さん垢版2022/12/14(水) 23:48:48.75ID:HhU2DiVl
.は分かりにくいので,にしてみたらずっと良くなりました
0137132人目の素数さん垢版2022/12/16(金) 19:32:42.10ID:RIF1nIOn
松坂和夫著『集合・位相入門』pp.184-185

同相写像についてですが、

f を同相写像とすると、


f によって (S, O) の各開集合は (S', O') の開集合にうつされ、逆に f^{-1} によって
(S', O') の各開集合は (S, O) の開集合にうつされる。


と書いてあります。そのほか、2つの位相空間の閉集合系、近傍系、開核作用子、閉包作用子
なども同相写像によって互に移し変えられると書いてあります。

ところが、

O_1, O_2 を S の開集合とすると、

f(O_1 ∩ O_2) = f(O_1) ∩ f(O_2)

が成り立つことや、

O_λ を S の開集合とすると、

f(∪ O_λ) = ∪ f(O_λ)

が成り立つことや、

f(S) = S'
f(∅) = ∅

が成り立つことについては書いてありません。

なぜでしょうか?
0138132人目の素数さん垢版2022/12/16(金) 19:40:25.06ID:RIF1nIOn
これらは非常に重要な事実だと思います。
0139132人目の素数さん垢版2022/12/16(金) 22:15:25.74ID:RttI7qBw
同相でなくとも全単射であれば成り立つような性質、改めて書く必要もなかろう
0140132人目の素数さん垢版2022/12/17(土) 14:38:02.44ID:yPjIRhLU
松坂和夫著『集合・位相入門』pp.184-185

同相な位相空間は位相空間として同じ構造をしているというのは自明なことなんですか?

松坂さんは、

「一般に、開集合系にもとづいて定義される諸概念については、 (S, O), (S', O') の一方の
上で成り立つことは、他方の上でもそのまま成り立つのである。」

と書いています。

なぜ、こんな一般的なことを証明なしに言い切ることができるのでしょうか?

群の場合は、同型な群が全く同じ構造をもつから一方の上で成り立つことは他方の上でも成り立つ
というのは確かにそうだろうと思いました。ですが、実際には群の同型の場合にも一方の上で成り
立つことは他方の上でも成り立つというのは証明すべきことですよね。ですが、あまりにも
主張が一般的にすぎると思います。どう証明するんですか?
0141132人目の素数さん垢版2022/12/17(土) 14:41:09.68ID:yPjIRhLU
>>139
確かにそうですが、重要な事実だと思いました。
0142132人目の素数さん垢版2022/12/17(土) 14:44:21.93ID:yPjIRhLU
同型な群の場合には、確かに自明感が非常に強くてひっかかりませんでしたが、
同相な位相空間の場合は、群の場合のように自明とまでは言えないように思います。
0143132人目の素数さん垢版2022/12/17(土) 15:56:25.25ID:EZ3lMfvr
点も開集合もなんもかも全単射よ
感覚的なことをつかみたいなら
自明としか言い様がない
0144132人目の素数さん垢版2022/12/17(土) 16:04:38.50ID:EZ3lMfvr
>>140
>実際には群の同型の場合にも一方の上で成り
>立つことは他方の上でも成り立つというのは証明すべきことですよね
どうかなあ
厳密なこと求めたい人は証明したらいいだけじゃない?
あと初学者
0145132人目の素数さん垢版2022/12/17(土) 16:29:41.42ID:IFMi52dQ
厳密を求める数学の、初学者向けの入門の本だから、やっぱり書くべきってことか
0146132人目の素数さん垢版2022/12/17(土) 16:33:58.41ID:2At8pOU5
集合として必要なこととその上の位相構造で必要なことの区別がついてない
松坂くんのレス読んだことなかったけどこんな状態なんだな
0147132人目の素数さん垢版2022/12/17(土) 16:43:05.26ID:EZ3lMfvr
>>145
いんや
初学者が自分でやれってこと
0148132人目の素数さん垢版2022/12/18(日) 14:54:33.53ID:WSjazB5j
位相空間 (S, O) から (S', O') への全単射 f が同相写像であるためには、
S の任意の部分集合 M に対して、 f(closure(M)) = closure(f(M)) が成り立つことが
必要十分であることを示せ。
0149132人目の素数さん垢版2022/12/18(日) 17:15:46.96ID:WSjazB5j
全単射 f が同相写像であるとする。

A は閉集合 ⇔ f(A) は閉集合

が成り立つのは自明。

M ⊂ closure(M) であるから、 f(M) ⊂ f(closure(M)) が成り立つ。
f(closure(M)) は f(M) を含む閉集合であるから、

closure(f(M)) ⊂ f(closure(M))

が成り立つ。

y ∈ f(closure(M)) とする。
y = f(x), x ∈ closure(M)
A' を f(M) を含む任意の閉集合とする。
M ⊂ f^{-1}(A') かつ f^{-1}(A') は閉集合である。
∴ closure(M) ⊂ f^{-1}(A')
∴ y = f(x) ∈ A'
∴ y ∈ closure(f(M))

∴ closure(f(M)) = f(closure(M))
0150132人目の素数さん垢版2022/12/18(日) 17:32:02.92ID:WSjazB5j
逆に、 S の任意の部分集合 M に対して、 f(closure(M)) = closure(f(M)) が成り立つとする。


A を S の任意の閉集合とする。

f(A) = f(closure(A)) = closure(f(A)) が成り立つから、

f(A) は S' の閉集合である。


f(A) を S' の閉集合とする。

f(closure(A)) = closure(f(A)) が成り立つ。

closure(A) = f^{-1}(f(closure(A))) = f^{-1}(closure(f(A))) = f^{-1}(f(A)) = A であるから、

A は S の閉集合である。
0151132人目の素数さん垢版2022/12/19(月) 17:17:17.40ID:AeCqYypj
位相空間の間の写像が開写像であるという性質は、その定義域を縮小した場合には必ずしも
保存されないことを証明せよ。
0152132人目の素数さん垢版2022/12/19(月) 17:25:05.77ID:AeCqYypj
簡単ですね。

f : R ∋ x → x ∈ R

は開写像であるが、

g = f の相対位相空間 {0} への制限とすると、

g : {0} → R

は開写像ではない。
0153132人目の素数さん垢版2022/12/19(月) 17:27:06.51ID:AeCqYypj
簡単ですね。

f : R ∋ x → x ∈ R

は閉写像であるが、

g = f の相対位相空間 R - {0} への制限とすると、

g : R - {0} → R

は閉写像ではない。
0154132人目の素数さん垢版2022/12/19(月) 17:29:02.08ID:AeCqYypj
松坂和夫著『集合・位相入門』p.190

注意 位相空間の間の写像が開写像あるいは閉写像であるという性質は、その定義域を
縮小した場合には必ずしも保存されない。(反例を挙げることは練習問題とする。)

簡単すぎます。
0155132人目の素数さん垢版2022/12/19(月) 17:30:36.87ID:AeCqYypj
松坂和夫著『集合・位相入門』ですが、演習問題が非常につまらないものばかりです。
多少難しい、興味を引き起こすような問題を入れるべきですよね。
0156132人目の素数さん垢版2022/12/19(月) 17:53:06.25ID:AeCqYypj
なお、終集合 A を特に指定しないで、単に’Λ によって添数づけられた族 (a_λ)λ∈Λ’というときは、
Λ を定義域とし、その各元 λ において値 a_λ をとる(終集合を重視しない)写像のことを意味する。

写像は写像です。

「終集合を重視しない写像」の定義が書いてありません。

これは本当に数学の本なのでしょうか?
0157132人目の素数さん垢版2022/12/19(月) 18:38:30.57ID:AeCqYypj
集合族 (A_λ)λ∈Λ については終集合は重視されないなどと松坂さんは書いています。

単に逃げているだけではないでしょうか?
0158132人目の素数さん垢版2022/12/19(月) 20:59:36.75ID:AeCqYypj
直積 Π A_λ は厳密にはどうやって定義されるんですか?

直積について、松坂さんの本はいい加減すぎます。
0160132人目の素数さん垢版2022/12/19(月) 23:55:52.85ID:XYL5mQAX
自己解決しました
気持ち悪い一行空けだけきれいに消せました
正規表現ありがとうございました
0161132人目の素数さん垢版2022/12/20(火) 10:02:41.85ID:/Opgtnd5
松坂和夫著『集合・位相入門』

A := Π A_λ とする。
pr_λ : A → A_λ を射影とする。
B ⊂ A_λ とする。

pr_{λ_i}^{-1}(B) = Π_{λ∈Λ-{λ_i}} A_λ ✕ B

というような記述がありますが、これは駄目ですよね。
終集合を重視しないなどと逃げていますが、終集合を重視しないとは一体どういうことなのか
説明しなければなりませんよね。
0162132人目の素数さん垢版2022/12/20(火) 10:07:43.67ID:/Opgtnd5
A := Π A_λ の各 A_λ をある一つの集合 U の部分集合とすれば解決できるのではないでしょうか?

そうすれば、 Λ から U への写像で、 各 λ∈Λ に対して a(λ) ∈ A_λ であるようなものの
集合を A と定義すればいいですよね。
0163132人目の素数さん垢版2022/12/20(火) 10:11:48.18ID:/Opgtnd5
A, B を任意の集合とするとき、 A ∪ B を考えてもいいんですか?

A, B がある一つの集合の部分集合のときだけに制限しなくてもいいんですか?
0164132人目の素数さん垢版2022/12/20(火) 10:13:41.73ID:/Opgtnd5
もし、任意の集合たちの和集合を考えることができるのならば、

Λ から ∪ A_λ への写像で、 各 λ∈Λ に対して a(λ) ∈ A_λ であるようなものの
集合を A と定義すればいいですよね。

松坂さんは「終集合を重視しない」などと書いてごまかしていますよね。
0166132人目の素数さん垢版2022/12/20(火) 13:38:12.38ID:BYGqj2gX
>>163
集合論の基礎で当たり前ですから
別にどうでも良いのです
0167132人目の素数さん垢版2022/12/20(火) 14:15:29.18ID:uNkrytOl
スペクトル系列を勉強していて係数が体などの条件下で
H^p(B,H^q(F))=H^p(B)⊗H^q(F)
が普遍係数定理より成り立つという主張を見ました。(Hはすべて局所係数でない通常のコホモロジーです)
これは加群としてはたしかにコホモロジーをコホモロジーで表す形の普遍係数定理より同型ですが,
環としての同型つまり積が一致している事はどのようにしてわかるのでしょうか。
左辺の積はカップ積であり,右辺では次数付き代数としてのテンソル積での積(つまり成分ごとにカップ積でかけて符号を適当につけたもの)が入っています。
0168132人目の素数さん垢版2022/12/20(火) 15:28:37.04ID:6N95v7vI
>>167
何で見たの?
0170132人目の素数さん垢版2022/12/20(火) 17:14:57.31ID:iDV5H5la
(1/s)F(s)+sF(s)=4
についてインパルス関数の扱いがよくわかっておらず、
また、4sint 、4cost どちらでも解になるのではないかと困っております。
よろしくお願いします。
0171132人目の素数さん垢版2022/12/20(火) 19:57:23.16ID:p4bgKPK4
>>167
チェインの段階で関を定義するよ
0172132人目の素数さん垢版2022/12/21(水) 03:19:35.86ID:d2Z4gYmn
>>170
F=?
0173132人目の素数さん垢版2022/12/21(水) 11:42:48.74ID:JCsjOROs
https://imgur.com/a/qEVM9bE
これの解き方がわからないので教えてください🙏
0174132人目の素数さん垢版2022/12/21(水) 11:47:07.31ID:xQ0Waria
>>172
逆ラプラス変換でf(t)を求めるという問題です。
説明不足ですみません。
0176132人目の素数さん垢版2022/12/22(木) 21:24:04.91ID:AWr57wCa
代数的サイクルとエタールコホモロジーを読みたいと思っています。
レベルとしてはハーツホーンの次に読むくらいなのでしょうか?あるいはハーツホーンの次にもう一冊読んだりしたほうが良いのでしょうか?
0179132人目の素数さん垢版2022/12/23(金) 12:22:55.26ID:zMkw+dgl
>>178
全部解いた。全部解ける。完璧に内容を理解し使えるようになった。ってかこれは普通じゃね?
このスレってレベル低いのか?
0180132人目の素数さん垢版2022/12/23(金) 13:56:17.94ID:HOHbxetQ
線形代数におけるフレームとはベクトルの順番付けという認識で良いですか?
0181132人目の素数さん垢版2022/12/23(金) 13:57:41.83ID:HOHbxetQ
>>173
U_1,U_2をそれぞれ基底の線形結合で表してから点(1,2,3)^Tとの距離の最小値を求めれば良いのでは?
0182132人目の素数さん垢版2022/12/23(金) 20:52:02.16ID:mb8I8J6T
>>180
枠だから基底じゃないかな
0184132人目の素数さん垢版2022/12/25(日) 01:44:21.76ID:PUsn34if
順番付けて言うか基底は順番も含めた概念でしょ
a,b,cとc,b,aは別の基底

いずれにせよ基底の(順序付きの)組だからGL(V)でしょ
0186132人目の素数さん垢版2022/12/25(日) 07:37:56.33ID:PUsn34if
順序がないと座標が決まらないじゃん
v=xa+yb+zcのときこの基底に関する座標は(x,y,z)
0187132人目の素数さん垢版2022/12/25(日) 08:47:29.30ID:F9p1cVv0
「座標と集合の違い」を繰り返してる馬鹿が「常駐」してるな
隙を見せるとこの話
よっぽど感動したんだろうな
0188132人目の素数さん垢版2022/12/25(日) 12:01:19.56ID:PUsn34if
>>187
それ俺のこと?
そんな話記憶無いけど
座標を決定できるために
基底は順序を含んだ概念だというのは常識
0189132人目の素数さん垢版2022/12/25(日) 12:01:45.79ID:BY5vNfZH
以下の定理があります。

f : (a, b) → R を単射な連続関数とする。
f の逆関数も連続関数である。

この定理より、

(a, b) と f((a, b)) は同相であることが分かります。

一般の位相空間では、 f が全単射かつ連続であっても同相写像であるとは限りません。

位相空間がどんな条件を満たせば、実関数のように、全単射かつ連続であれば同相写像となるのでしょうか?
0190132人目の素数さん垢版2022/12/25(日) 13:51:20.18ID:PUsn34if
>>189
>f の逆関数も連続関数
0191132人目の素数さん垢版2022/12/25(日) 14:57:33.68ID:Gv6nxX5y
位相空間XとYの間の連続な全単射fが
同相写像になるための必要十分条件としては
自明なものとしてはfが開写像であるというものがあります。
位相空間の条件としては

コンパクトな空間からハウスドルフ空間への連続な全単射は
同相写像であるというものがあります。
0192132人目の素数さん垢版2022/12/25(日) 15:10:43.09ID:FyIxLN+8
>>185
嘘ですから信じちゃダメですよ
座標云々はまさにフレームの考え方です
0193132人目の素数さん垢版2022/12/25(日) 15:25:37.50ID:O2DsbGsM
誰も真に受けちゃいないさ
0194132人目の素数さん垢版2022/12/25(日) 15:35:30.25ID:YQKvC9IW
そもそもただの線形空間にフレームって言葉あんまり使わないでしょ
ベクトル束の用語。

基底に順序が付いてるかどうかは考えたことなかったけど、文脈によるんじゃない?
「任意のベクトル空間に基底が存在する」とかだと順序は考えていないことが多いけど、「線型写像fを基底〜について行列表示したものを…」だと順序付きを考えてる
0195132人目の素数さん垢版2022/12/25(日) 18:31:55.23ID:PUsn34if
>>192
基底が何のために導入されたか考えたら自明だけどね
0196132人目の素数さん垢版2022/12/25(日) 18:57:01.63ID:PUsn34if

実際は順序(基数)でなくても
適当なインデックス集合Λでいいけどね
b:Λ→Vがベクトルの「組」
c:Λ→kで有限個以外は0な係数の「組」による
bc=Σb(λ)c(λ)が「線形結合」
bc=0→c=0が「線形独立」
V={bc|c:Λ→kで有限個以外は0}が「生成系」
線形独立かつ生成系が基底
v=bcとなるcが「座標」
0197132人目の素数さん垢版2022/12/26(月) 11:56:45.78ID:icEJQw3y
>>177
そこも質問したいです。
代数的サイクルとエタールコホモロジーを読むのに、ハーツホーンを最後の章の演習問題まで全て理解するレベルが必要なのか、そうではないのか、分かる人いますか?
0198132人目の素数さん垢版2022/12/26(月) 14:58:02.98ID:WC0SGAMo
ハーツホーンの前の段階で「参考書ルートを夢想してるだけ」の馬鹿か…

ヤフー知恵袋とか(この低レベル掲示板も同じか)にいっぱいいるタイプの質問するだけで終わる馬鹿。
能力の低い人間は数学の独学は諦めなさい。
0199132人目の素数さん垢版2022/12/26(月) 15:06:32.91ID:6aszGTVw
黄チャートを何周したら青チャートに進めますか?
と同種の質問
0200132人目の素数さん垢版2022/12/26(月) 15:40:54.82ID:ES3aOlxy
>>199
詳しく教えてください。
ハーツホーンが黄チャートだとすると、代数的サイクルと~は青チャートくらいのレベル感ということですか?
0201132人目の素数さん垢版2022/12/26(月) 15:44:45.82ID:ES3aOlxy
もしそうなら、ハーツホーンを完璧には理解できていなくても、ある程度理解していたら読めるレベル感ということですね。
ありがとうございます!
0202132人目の素数さん垢版2022/12/26(月) 15:45:09.09ID:pBtAJJTQ
やっぱりただの参考書キチガイか
0203132人目の素数さん垢版2022/12/26(月) 17:31:16.68ID:H2GyuS7K
ただのプロブレムソルバーかも
0204132人目の素数さん垢版2022/12/27(火) 16:12:09.55ID:YfaDKW1g
ウリゾーンの補題はユークリッド空間上の場合だとより簡単な証明はありますか?
0205132人目の素数さん垢版2022/12/27(火) 18:38:03.26ID:LoNJljOz
自明としか言いようがない
0207132人目の素数さん垢版2022/12/27(火) 19:31:23.10ID:VRfHkim5
しかしこれを自明と思わずにきれいだと思う向きも多いようだ
0208132人目の素数さん垢版2022/12/27(火) 19:38:52.20ID:+2oTtsq/
>>204
作り方見たら
まあこうする他無いなって証明もまんま
0209132人目の素数さん垢版2022/12/27(火) 19:40:40.23ID:+2oTtsq/
>>207
何も知らずに作れと言われて
これを思いつくのはセンス抜群
自分がだったら喜んで小躍りしちゃうね
だから自分は自明っては言い難いかな
0210132人目の素数さん垢版2022/12/27(火) 19:52:53.78ID:LoNJljOz
>>207
>>209
距離空間とわかっている位相空間でウリゾーンの補題のような連続関数が存在のは自明。
この補題の重要性はそこではない。
0211132人目の素数さん垢版2022/12/27(火) 19:55:11.82ID:LoNJljOz
存在のは→存在するのは
0214132人目の素数さん垢版2022/12/27(火) 22:26:38.44ID:+2oTtsq/
>>210
>距離空間とわかっている位相空間で
そうかそれに限定しての話か
そんな限定する必要あるの?
0215132人目の素数さん垢版2022/12/27(火) 22:43:51.27ID:spSCBpDW
次の極限が存在するならその値とそうなる理由を、存在しないならその理由を答えよ。
https://i.imgur.com/JOXdv0A.jpg

これもお願いします。
https://i.imgur.com/gvopWqT.jpg


間違えて高校数学のとこに貼ってしまったのですが、解いて頂いて答え合わせ願えると幸いです。
0217132人目の素数さん垢版2022/12/27(火) 23:13:42.04ID:+2oTtsq/
>>216
d(p,A)/(d(p,A)+d(p,B))でいいのでは?
0220132人目の素数さん垢版2022/12/28(水) 01:12:54.58ID:TqIRTjQW
>>170
F(s)=4s/(1+s^2)から一発
なんで4sintと思うのか意味不明
デルタ関数(インパルス)関係なし
ていうか何でそれ考えるのか意味不明
0221132人目の素数さん垢版2022/12/28(水) 01:19:27.58ID:TqIRTjQW
>>218
てゆか思いつけよ
0223132人目の素数さん垢版2022/12/28(水) 06:40:19.06ID:I3rgIA/8
ねっ、自明でしょ?
0224132人目の素数さん垢版2022/12/28(水) 07:01:49.18ID:TqIRTjQW
それでもやっぱり自明とは言わないな俺なら
0225132人目の素数さん垢版2022/12/28(水) 08:31:42.70ID:ZjPViOV7
関数d(x,A)の連続性、関数f(x)=d(x,A)/{d(x,A)+d(x,B)}が求める性質を満たすことを示せw
0226132人目の素数さん垢版2022/12/28(水) 09:47:50.94ID:I3rgIA/8
>>225
d(x,A)のリプシッツ連続性も自明、fが求める性質を満たすのも自明。
0228132人目の素数さん垢版2022/12/28(水) 10:42:10.67ID:I3rgIA/8
>>227
わからなくて質問した側が教員気取りか
0230132人目の素数さん垢版2022/12/28(水) 11:32:09.84ID:TqIRTjQW
>>225
d(x,A)=inf{d(x,a)|a∈A}
d(y,x)+d(x,A)=min{d(y,x)+d(x,a)|a∈A}≧min{d(y,a)|a∈A}=d(y,A)
d(y,A)-d(x,A)≦d(y,x)
y→x⇔d(y,x)→0⇒d(y,A)→d(x,A)

x∈A
d(x,A)=0
f(x)=0
x∈B
d(x,B)=0
d(x,A)>0(∵d(x,A)=0⇒x∈A:closed⇒x∈A∩B≠φ⇒人)
f(x)=1
0232132人目の素数さん垢版2022/12/28(水) 12:22:05.59ID:I3rgIA/8
君に正しいと認定されなくて良かった
0233132人目の素数さん垢版2022/12/28(水) 13:10:00.04ID:I3rgIA/8
もう一回言っておくがウリゾーンの補題は実数と関係がなさそうな位相空間上で実数への連続関数が構成できることであり、距離空間のような実数と関係がある位相空間への適用はあまり意味がない。
0234132人目の素数さん垢版2022/12/28(水) 13:29:29.30ID:ZjPViOV7
存在が分かっているだけよりも具体的に書ければ得することがあるかもしれない
0235132人目の素数さん垢版2022/12/28(水) 14:09:49.80ID:j/v0ecos
「てへへ」と書いていたときの謙虚さはどこへやら
0236132人目の素数さん垢版2022/12/28(水) 14:29:14.51ID:ZjPViOV7
>>219で分かったと言ってる。
>>223が自明だと言い張るので突っ込んでみただけだよ。
大元の質問の背景の1つが>>234。もう一つは略。
以上。
0238132人目の素数さん垢版2022/12/28(水) 16:11:35.00ID:eplsP5DQ
今年はもうダメだから必死にならなくてもいいって
0239132人目の素数さん垢版2022/12/28(水) 16:17:58.61ID:sFhs0LXw
>>237
つまんないから
もっと面白いやつで
0241132人目の素数さん垢版2022/12/28(水) 17:38:19.73ID:gmoQ1tYe
(x,y)(t)=(tcosθ,tsinθ)

(5x^2y^2+2x^2+4y^2)/(x^2+2y^2)
=(5t^4cos^2θsin^2θ+2t^2(1+sin^2θ))/t^2(1+sin^2θ)
=(5t^2cos^2θsin^2θ+2(1+sin^2θ))/(1+sin^2θ)
t→0
=2
0242132人目の素数さん垢版2022/12/28(水) 17:48:06.00ID:cWaYk46x
学部レベルかわからんけど
平均値の定理f(x)=f(a)+f'(c)(x-a)のcは定数らしいけど
このcが定数なのってxを動かさない場合だけで
もし微分とかでxを動かす場合になったらcはxの関数として扱うことになるよね?
としたらf'(x)=f'(c)+{df'(c)/dc}{dc/dx}(x-a)
とすることになるのかな
0243132人目の素数さん垢版2022/12/28(水) 18:14:06.24ID:ZtOgHRbC
n=1,2,3,4,…として、
例えば 2^n をmod5でみると 2,4,3,1と 0以外のすべてが現れるますが
2^nをmod7だと2,4,1の3つしか現れず、
また例えば 7^nをmod12でみると 7,1の2つしか現れないです。

一般に、a^n をmodNでみたときにどのような剰余が現れるか現れないかを
求めるにはどういう方法によれば分かりますか?
0244132人目の素数さん垢版2022/12/28(水) 18:33:57.09ID:VFGfWLjJ
(2)
(x,y)(t)=(tcosθ,tsinθ)

x^2y/(x^2+y^2)^2
=t^3cos^2θsinθ/(t^4)
=cos^2θsinθ/t

sign(lim[t→+0]cos^2θsinθ/t)sign(lim[t→-0]cos^2θsinθ/t)=-1

(5),(7)計算する

(6)
z=x*∂f/∂x(1/2,-1/2)+y*∂f/∂y(1/2,-1/2)
0246132人目の素数さん垢版2022/12/28(水) 19:26:42.33ID:TqIRTjQW
>>242
>f(x)=f(a)+f'(c)(x-a)
cは関数とは限らないそれはxに対して「1つ」決まると限らないから
関数になる場合でも微分可能とは限らない
しかしfが2階微分可能でcが微分可能となる場合は
>f'(x)=f'(c)+{df'(c)/dc}{dc/dx}(x-a)
すなわち
f'(x)=f''(c)c'(x-a)+f'(c)
となる
0247132人目の素数さん垢版2022/12/28(水) 19:28:45.96ID:TqIRTjQW
>>243
素数の場合考えてフェルマーの小定理
それから中国剰余定理かね
0248132人目の素数さん垢版2022/12/28(水) 20:20:02.43ID:I3rgIA/8
>>236
自明のことをわざわざ考察しても益がないという例だな。
0249132人目の素数さん垢版2022/12/28(水) 21:33:40.13ID:cWaYk46x
>>246
そっかあ三次とかだとc2つあったりするもんね
ありがとう答えてくれて
0250132人目の素数さん垢版2022/12/29(木) 10:25:54.70ID:z3lxyOnC
>>247
たとえば 5^n mod169 が 2 になるような n はありますか。
0251132人目の素数さん垢版2022/12/29(木) 11:06:42.22ID:6jydkCdO
>>250
5の累乗を13で割った余りは1,5,12,8に限られるので5の累乗で169で割った余りが2になることはない。

数値実験もせずに質問するのは感心しないな。
0252132人目の素数さん垢版2022/12/29(木) 11:32:16.12ID:Di8S/3Xg
わざわざ数値実験なんかしたくないから聞いてるんじゃないですか
0253132人目の素数さん垢版2022/12/29(木) 11:54:32.43ID:6jydkCdO
その態度は感心しないな
0255132人目の素数さん垢版2022/12/29(木) 15:29:51.93ID:99GHPvoL
いや、質問に答えたいではなくマウントが取りたい、俺に足る質問をもってこい、って人が割合増えたんだと思う
0256132人目の素数さん垢版2022/12/29(木) 15:31:09.14ID:99GHPvoL
要するに、衰退気味の5ちゃんねるに残ってる時点で「回答者」がヤバイやつしかいない
0257132人目の素数さん垢版2022/12/29(木) 15:38:00.83ID:6jydkCdO
>>255
>>256
何を馬鹿なことを言っている
0258132人目の素数さん垢版2022/12/29(木) 15:46:35.18ID:99GHPvoL
他の質問サイトとかでも手を動かさない質問というのは普通によくある
それに文句をつける回答者もどこにでもいる
ただ、レベルの高い質問サイトは、そうでない真摯な回答者もいっぱいいる
0259132人目の素数さん垢版2022/12/29(木) 15:51:14.21ID:6jydkCdO
>>258
5ちゃんねるは質問サイトではない
0260132人目の素数さん垢版2022/12/29(木) 15:59:00.39ID:99GHPvoL
>>259
5ちゃんねるは質問サイトじゃないから、まともな回答者は残ってないってこと?
つまり結論は同じと
0261132人目の素数さん垢版2022/12/29(木) 16:02:37.73ID:99GHPvoL
まあ何にせよ、たまたまここに流れ着いてしまう質問者もいるみたいだから書いておきたい
ここの回答者はレベル高くない
0266132人目の素数さん垢版2022/12/30(金) 13:35:00.74ID:gsC8gvdQ
松坂和夫著『集合・位相入門』

p.194 問13

位相空間族 (S_n)_{n ∈ N} において、どの S_n も第2可算公理を満足するならば、
直積空間 S = Π_{n ∈ N} S_n も第2可算公理を満足することを示せ。


その松坂さんによる解答ですが、以下のような感じです:

(1)

(Π_{n ∈ N - {i_1, …, i_m}} S_n) ✕ O_{i_1} ✕ … ✕ O_{i_m}

と表されるもの全体の集合は、 S の基底になる。

ただし、{i_1, …, i_m} は N の有限部分集合、 O_{i_1}, …, O_{i_m} はそれぞれ
S_{i_1}, …, S_{i_m} の基底の要素である。

(2)

可算集合の有限部分集合全体の集合は可算集合である。
0267132人目の素数さん垢版2022/12/30(金) 13:39:00.37ID:gsC8gvdQ
各 S_i の基底 B_i から一つの要素 O_i をとる。

(Π_{n ∈ N - {i_1, …, i_m}} S_n) ✕ O_{i_1} ✕ … ✕ O_{i_m}

と表されるもの全体の集合は可算集合になる

というのは分かるのですが、 O_{i_j} が S_{i_j} の基底 B_{i_j} の任意の要素として、
(Π_{n ∈ N - {i_1, …, i_m}} S_n) ✕ O_{i_1} ✕ … ✕ O_{i_m}

と表されるもの全体の集合は可算集合になるのはなぜですか?
0268132人目の素数さん垢版2022/12/30(金) 14:01:46.43ID:gsC8gvdQ
可算個の可算集合の和集合は可算だからですか?
0269132人目の素数さん垢版2022/12/30(金) 14:18:56.52ID:aeuVh7Tn
>>267
加算(Πのインデックス)×加算(Oのバリエーション)=加算
0270132人目の素数さん垢版2022/12/30(金) 14:42:48.18ID:gsC8gvdQ
N の有限部分集合全体の集合がインデックスになるんですよね?

インデックスを固定したときに、可算集合の有限個の直積は可算集合である。
可算個(インデックス)の可算集合(可算集合の有限個の直積)の和集合はまた可算であるから、
「位相空間族 (S_n)_{n ∈ N} において、どの S_n も第2可算公理を満足するならば、
直積空間 S = Π_{n ∈ N} S_n も第2可算公理を満足する」わけですよね?
0271132人目の素数さん垢版2022/12/30(金) 19:43:09.61ID:gsC8gvdQ
圏論で対象というのがあります。

「対象の集まり」は集合とは限らないそうです。
では「対象の集まり」とは何なんですか?
0272132人目の素数さん垢版2022/12/30(金) 19:51:30.40ID:gsC8gvdQ
圏論を勉強する人はこのあたりはどうやって乗り越えているのでしょうか?

数学のより基礎的な本を読むのでしょうか?
0273132人目の素数さん垢版2022/12/30(金) 20:13:00.32ID:aeuVh7Tn
>>271
クラスね
クラスの元が集合
0274132人目の素数さん垢版2022/12/31(土) 13:10:06.77ID:RkBXavrD
公理論的集合論でググる
wikiでわかればそれでよし
わからなくてももう解説文なんかゴロゴロ転がってるよ
0275132人目の素数さん垢版2022/12/31(土) 15:36:26.06ID:cop8ToDC
>273-274

ありがとうございました。

数学ではよくあることですが、基礎へ遡らないと厳密には理解できないんですね。

というか、普段、集合とは何かとか全く気にもしていませんが、圏論に限らず、厳密に
理解しようというのなら、公理的集合論とか勉強しないといけないですよね。

集合には似ているが集合ではないものが登場すると公理的集合論とかを勉強しなければという
気持ちがより強くなりますね。

圏論を勉強する人はまず公理的集合論を勉強してから勉強をスタートするのでしょうか?
0276132人目の素数さん垢版2022/12/31(土) 15:46:04.20ID:kAzwkGjo
>>275
>圏論を勉強する人はまず公理的集合論を勉強してから
イランと思うよ
0277132人目の素数さん垢版2022/12/31(土) 16:19:54.41ID:9MFFgARv
>>275
小さな圏っていう基本的なワードが理解できなかったりするように、集合とクラスの違いくらいは基礎知識ではある
ただ、例えば分野によっては線形代数の基礎知識を理解できなくてもいいように、結局どこまで理解するかは人それぞれ
0278132人目の素数さん垢版2022/12/31(土) 19:13:59.22ID:RkBXavrD
公理論的集合論勉強してからって言ったってホントに勉強して一区切り着くまでなんて言ってたらキリがないからとりあえずクラスと集合の区別ついたらそれでいい
0279132人目の素数さん垢版2023/01/02(月) 11:50:51.52ID:X+NmUptl
>>271
圏論ではどうでもいい事
集合論との対応で持ち出しただけ
圏論で重要なのは射
射で見れば「対象の集まり」は恒等射の事
0280132人目の素数さん垢版2023/01/02(月) 15:24:52.70ID:FoFhDZ+Q
tanhのいい感じの近似式ないかと探してるんですが https://i.imgur.com/pn2PYK9.png
2点パデ近似というやつでx=0とx=+∞に目を配ったものはできるようなのですが(上図黄色)
では (-∞, -1), (0, 0), (+∞, 1) の3点を考慮して、絶対値や場合分けを用いない近似有理関数って作れますか?
内容は高校レベルなのかもしれんが高校数学の質問ではないのでここで
0281132人目の素数さん垢版2023/01/02(月) 15:52:53.87ID:TFIhRBBE
有理関数ならx=-∞とx=+∞は同じになるので無理
0282132人目の素数さん垢版2023/01/02(月) 16:34:09.86ID:Q4ALVMLQ
圏論なら準同型射を理解してから齧りにきてほしい。

クラスとか集合とかよりも型論理のタイプから入ってくる人たちは準同型定理と商代数をやって。
0283132人目の素数さん垢版2023/01/02(月) 16:47:28.35ID:VTCDMslz
>>279
scholzeとclausenのcondensed mathematicsのアイデアって実は、
BarwickとHaineっていう数学者達も思い付いてたんだけど、
どこで差がついたかと言うとcondensed mathematicsはZFCの中で(グロタンディーク宇宙の選択に依存せず)展開できることを示したという点

こういうリスクを受け入れてでも勉強しないか、それとも勉強しておくかは個人の選択だけどね
0284132人目の素数さん垢版2023/01/02(月) 18:40:39.56ID:Osy1kgqQ
圏論とか具体性無いの勉強したけりゃしたが良いが
勉強だけして具体的な結果出せないだけになりがち
0285132人目の素数さん垢版2023/01/02(月) 20:06:24.74ID:Zu26poyh
>>283
BarwickとHaineが圏論の論理的基礎について不勉強だったという滅茶苦茶なことを言ってる
0286132人目の素数さん垢版2023/01/02(月) 20:59:43.11ID:Osy1kgqQ
>>281
なば
tanh^2xを近似するのどうかしら
正負で√の別の分枝を取るみたく
0287132人目の素数さん垢版2023/01/03(火) 05:46:39.69ID:aK+JUGNI
f を R の部分位相空間 [0, 1] から位相空間 S への連続写像とする。
f_1 : [0, 1/2] → S such that f_1(x) := f(2*x) とする。
f_1 は R の部分位相空間 [0, 1/2] から位相空間 S への連続写像であることを示せ。
0288132人目の素数さん垢版2023/01/03(火) 05:52:38.47ID:aK+JUGNI
g : [0, 1/2] → [0, 1] such that g(x) = 2*x とする。
g が R の部分位相空間 [0, 1/2] から R の部分位相空間 [0, 1] への連続写像であることを示せればOKです。
直感的には明らかに連続写像ですが、どうやって示しますか?

R の部分位相空間 [0, 1] の開集合は、 O を R の開集合として、 [0, 1] ∩ O と書ける。
g^{-1}([0, 1] ∩ O) が R の部分位相空間 [0, 1/2] の開集合であることを示す必要がある。
すなわち、 O' を R の部分位相空間 [0, 1/2] の開集合として、 g^{-1}([0, 1] ∩ O) = [0, 1/2] ∩ O'
と書けることを示す必要がある。
0289132人目の素数さん垢版2023/01/03(火) 06:02:14.81ID:aK+JUGNI
g_1 : R → R such that g_1(x) = 2*x とする。
g_1 は連続写像である。
よって、 g_1^{-1}(O) は R の開集合である。
g^{-1}([0, 1] ∩ O) = [0, 1/2] ∩ g_1^{-1}(O) である。
よって、 g^{-1}([0, 1] ∩ O) は R の部分位相空間 [0, 1/2] の開集合である。
0290132人目の素数さん垢版2023/01/03(火) 07:49:02.49ID:/2OXHbUw
>>287-289
ここは質問スレだよ
0291132人目の素数さん垢版2023/01/03(火) 09:44:34.02ID:2jtVfc7P
>>288
すぐでしょ?
0292132人目の素数さん垢版2023/01/03(火) 12:02:30.24ID:ldk54mVj
そもそも大学の解析の教科書なら
2^x = exp( x log2 )
なので
・f(x) = x log2は連続
・g(x) = exp( x )は連続
・連続関数の合成関数は連続
で終わり
0293132人目の素数さん垢版2023/01/03(火) 12:04:48.55ID:kM4MrsXY
大学の解析の教科書ってすごいね!
0294132人目の素数さん垢版2023/01/03(火) 12:30:13.69ID:LWfTnsQs
>>292
>連続関数の合成関数は連続
ホントにそうなのか、よくわかりません。証明してください
0295132人目の素数さん垢版2023/01/03(火) 12:49:02.76ID:2jtVfc7P
>>294
シオモナ
0296132人目の素数さん垢版2023/01/03(火) 12:49:45.30ID:aK+JUGNI
みなさん、ありがとうございました。

f : R → R が連続であるとき、

f の終域を f(R) を含む R の部分集合に変えても、連続である。
f の定義域を R の部分集合に制限しても、連続である。

という定理がありましたね。
0297132人目の素数さん垢版2023/01/03(火) 12:51:45.54ID:aK+JUGNI
a, b を R^n の異なる2元とする。
f : [0, 1] → R^n を f(t) = (1 - t) * a + t * b で定義する。

f は同相写像であることを示せ。

f の逆写像が連続であることはどうやって示しますか?
0298280垢版2023/01/03(火) 12:53:24.52ID:nu46fVka
>>281
当方が行間読めてないのかもしれんがこれは嘘じゃないか? y=(x^2+1)/x とか
0299132人目の素数さん垢版2023/01/03(火) 12:56:22.60ID:aK+JUGNI
訂正します:

a, b を R^n の異なる2元とする。
f : [0, 1] → R^n を f(t) = (1 - t) * a + t * b で定義する。

g : [0, 1] → f(R) such that g(x) = f(x) for all x in [0, 1] は同相写像であることを示せ。

g の逆写像が連続であることはどうやって示しますか?
0300132人目の素数さん垢版2023/01/03(火) 12:58:40.47ID:/RUEyiE5
改訂三版まだあ
0301132人目の素数さん垢版2023/01/03(火) 13:05:04.99ID:aK+JUGNI
h を1次関数とすると、

h : [a, b] → [c, d]

は連続関数である。

全単射 f_i : [0, 1] → f_i([0, 1]) は、1次関数だから連続。
この逆関数も1次関数だから連続。

よって、 f_i は開写像。
よって、 f は開写像。
よって、 f^{-1} は連続。

これでOKですか?
0305132人目の素数さん垢版2023/01/03(火) 19:48:51.24ID:aK+JUGNI
原島鮮著『力学I』

1つの量が直交座標系 S について (Ax, Ay, Az) の3個の実数で与えられ、他の直交座標系 S'
(S と原点を共通に持つとする)について (Ax', Ay', Az') の3個の実数で与えられるとし、
これらの成分間の変換が、点の位置を表わす座標の変換と同様に行なわれるとき、 (Ax, Ay, Az)
または (Ax', Ay', Az') は1つのベクトル A であるとよぶ。

このベクトルの定義が全く分からないのですが、解説をお願いします。
0306132人目の素数さん垢版2023/01/03(火) 21:00:53.54ID:1A5bcamd
>>305
数学的な概念としての定義ではないので
物理板できけば?
0308132人目の素数さん垢版2023/01/04(水) 05:28:42.86ID:/CSklpWk
>>307
擬ベクトル(反変ベクトル)と区別するためよ
0310132人目の素数さん垢版2023/01/04(水) 08:41:29.28ID:uVcKfXVJ
多様体の接ベクトルの局所表示のつもりでは?
0311132人目の素数さん垢版2023/01/04(水) 10:43:00.20ID:Vwme59dQ
物理だとベクトルもテンソルも全て成分の間の関係式で定めるよ
成分って言葉を付ければちゃんと数学の言葉になるしこれが物理で聞くものだと思えるのは何も理解してない証拠
0312132人目の素数さん垢版2023/01/04(水) 12:54:55.56ID:NeOtiWXB
数学の言葉になるかどうかをちゃんと判定できるのは
物理屋ではないか?
0314132人目の素数さん垢版2023/01/04(水) 14:17:33.19ID:NeOtiWXB
>>313
物理を知らない数学バカにはわからない
0315132人目の素数さん垢版2023/01/04(水) 22:18:14.23ID:VGgCQMYJ
次元解析ぐらいしか純然たる物理学的直感って感じの手法は思いつかないな。
0316132人目の素数さん垢版2023/01/04(水) 22:56:18.67ID:/CSklpWk
>>315
単位ってのを文字(変数名)に解釈する
Mathematcaの扱いが正しいと思った
0317132人目の素数さん垢版2023/01/05(木) 16:30:18.72ID:LQnw801X
球の最密充填やってるんだけど
容器の形を変更した場合の計算って衝突シミュして観察するしかないかな?
容器の変更は壁の角度60,90,120度なら綺麗に収まるけど、他は配置が歪むよねって話でその変化を計算したいです
物理板でする質問かもしれなくてすまんです
これ読めみたいな本があれば教えて下さい
0318132人目の素数さん垢版2023/01/05(木) 18:45:32.57ID:D2871bN6
ワールド座標系での回転値がrx1,ry1,rz1、位置がpx1,py1,pz1のオブジェクトがあります。

同様に、ワールド座標系での回転値がrx2,ry2,rz2、位置がpx2,py2,pz2のカメラがあります。

rx1,ry1,rz1,px1,py1,pz1のオブジェクト座標を、カメラ座標系に変換するには、どのような計算式を使用して求めることが出来ますか?
0319132人目の素数さん垢版2023/01/06(金) 19:35:28.28ID:leDZeL9Q
nが奇数のとき
2^m≡5^n mod 17^2 をみたす自然数mが存在しないことはすぐ示せますか。
136通り総調べしなくてもできますか?
0320132人目の素数さん垢版2023/01/06(金) 20:35:31.02ID:N5LCtAnP
>>319
nが奇数のとき
2^m≡5^n mod 17 をみたす自然数mが存在しないことを示せばよい

2と5の平方剰余を調べると
(2/17)=1
(5/17)=(17/5)=(2/5)=-1
任意の自然数mにおいて(2^m/17)=1
nが奇数のとき(5^n/17)=(-1)^n=-1
よってnが奇数のとき
2^m≡5^n mod 17 をみたす自然数mは存在しない
0321132人目の素数さん垢版2023/01/07(土) 06:47:07.01ID:O2zekELH
>>318
ワールド座標系での回転値と位置およびカメラ座標系の定義を
0322132人目の素数さん垢版2023/01/07(土) 06:50:24.36ID:O2zekELH
>>317
ホボホボ数学でなさげ
あるいは数学でもニッチで無意味な分野だから
ホボホボ理論などなさげ
0324132人目の素数さん垢版2023/01/07(土) 14:54:02.83ID:bSNYCGms
S ⊃ S_1 ⊃ S_2 とする。
(S, O) を位相空間とする。
(S_1, O_1) を S の部分空間とする。
(S_2, O_2) を S_1 の部分空間とする。
(S_2, O_3) を S の部分空間とする。

O_2 = O_3 である。

これって重要な事実ではないでしょうか?
それにもかかわらず、松坂和夫著『集合・位相入門』には書いてありません。
0325132人目の素数さん垢版2023/01/07(土) 15:07:19.67ID:Nv3ylZsj
>>324
当たり前だしね
0326132人目の素数さん垢版2023/01/07(土) 15:22:28.19ID:bSNYCGms
事実として重要であることと、証明するのが簡単であることは無関係ではないでしょうか?

以下の事実も非常に簡単な事実ですが、定理としてこの本には書いてあります。

p.189 定理24
M が位相空間 S の開集合ならば、 M の任意の開集合は S においても開集合である。
また、 M が S の閉集合ならば、 M の任意の閉集合は S においても閉集合である。
0327132人目の素数さん垢版2023/01/07(土) 15:38:54.00ID:bNP+KRLl
へー
ーーーーーーー
 若い頃憧れた広中平祐氏。氏のフィールズ賞受賞業績である「特異点解消定理」※1。
世紀の難問をいわば力業でねじ伏せた証明は218ページとか。当時の数学史上最長論文で「広中の電話帳」として有名なくらいです。
理解できる人は世界に10人と、よく言われる表現で語られました。

 その後、証明は洗練されて、今では「大学院初年級の学生でも“ごまかしなし”に全体が読める証明がある程に理解が進んでいる.」
(松木謙二, 数学69巻1号2017年1月)といいます。さらに、智慧の結晶である数学理論が、知識化のおかげで、
今では機械学習モデルを支える定理として、実世界の課題に対して具体的な解決をもたらしているそうです。
0330132人目の素数さん垢版2023/01/07(土) 16:27:47.15ID:bSNYCGms
p.189 定理24
M が位相空間 S の開集合ならば、 M の任意の開集合は S においても開集合である。
また、 M が S の閉集合ならば、 M の任意の閉集合は S においても閉集合である。

これを定理として書いて、

>>324

については何も書かない。

フェアじゃないですよね。

どちらも簡単ですが、重要な事実だと思います。
0332132人目の素数さん垢版2023/01/07(土) 18:51:01.09ID:bSNYCGms
斎藤毅さんが、「現代の数学は、集合と位相の抽象的なことばで書かれ、線形代数と微積分の2本の柱で支えられている。」
と書いています。

線形代数が重要な分野って何ですか?
Jordanの標準形とかあまり使わないですよね。
0333132人目の素数さん垢版2023/01/07(土) 22:15:56.28ID:tyaEY32W
自分はトポロジーの人間だが、>>324 よりもJordan標準形の方が研究で使うという意味では重要だな
>>324 は当たり前過ぎて意識してないだけかもしれないけど。
0334132人目の素数さん垢版2023/01/08(日) 00:11:57.21ID:JhPhX9nv
>>332
Lie群Lie環
0336132人目の素数さん垢版2023/01/08(日) 18:24:46.48ID:BT5jXl5F
集合と位相(内田伏一 裳華房)p.136にこう書かれています。
『例25.4 通常の位相をもったR^2の部分空間
X={(x, sin(1/x)) | x>0}∪{(0,y) | -1≦y≦1}
について、Xは連結であるが、局所連結ではなく、弧状連結でもないことを示そう。
Xの部分集合
A={(x, sin'(1/x) | x>0}
は区間(0, ∞)と同相であるから連結集合である。(以下略)』
ここでAが区間(0, ∞)と同相であることはどのように示せるのでしょうか?教えて下さい。
0337132人目の素数さん垢版2023/01/08(日) 18:45:28.98ID:4Ngj40gO
>>336
写像 (0,∞)--->Aを
xに(x,sin(1/x))を対応させることにより定義すれば
連続な逆写像を持つ連続写像になるから
0338132人目の素数さん垢版2023/01/08(日) 19:32:11.44ID:KHAADs5L
M が R^n の凸集合ならば、その閉包 M^a も凸集合であることを示せ。
0339132人目の素数さん垢版2023/01/08(日) 19:37:41.51ID:GdZoBVKa
ここは出題スレじゃないよ
0340132人目の素数さん垢版2023/01/08(日) 20:32:16.83ID:4Ngj40gO
>>338
M^aの任意の2点はMの2点の極限であり、Mの2点を結ぶ線分はMの凸性よりMに含まれるので、その極限はM^aに含まれるからである。
0341132人目の素数さん垢版2023/01/08(日) 21:54:33.59ID:JhPhX9nv
>>340
>その極限はM^aに含まれる
ちょっとギャップ無い?
0342132人目の素数さん垢版2023/01/08(日) 21:59:01.68ID:kd1KJcTO
整数x,y
198x-yが272の倍数になることと 2x-11yが272の倍数になることは
同値らしいのですがどう示せますか
0343132人目の素数さん垢版2023/01/08(日) 21:59:28.02ID:BT5jXl5F
>>337
逆写像が連続写像になることはどのようにして示せますか?
0344132人目の素数さん垢版2023/01/08(日) 22:06:37.31ID:JhPhX9nv
>>343
曲線上の一点取ってその極狭い近傍で考えたら?
0345132人目の素数さん垢版2023/01/08(日) 22:08:42.70ID:JhPhX9nv
>>342
学部レベルかしら
0346132人目の素数さん垢版2023/01/08(日) 22:41:42.00ID:4Ngj40gO
>>343
(x,y)平面からx軸への射影の制限だから連続
0347132人目の素数さん垢版2023/01/08(日) 23:02:32.73ID:4Ngj40gO
>>341
Mの点の極限からなるのがM^a
0348132人目の素数さん垢版2023/01/08(日) 23:10:54.65ID:JhPhX9nv
>>347
M^aの2点を結ぶ線分上の点がMの点の極限ということを言うべきでは?と思うのだが
まあ細かすぎるかもね
0349132人目の素数さん垢版2023/01/09(月) 06:44:02.39ID:ipY7Dqt+
>>348
そういう生活もあろう
0351132人目の素数さん垢版2023/01/09(月) 12:02:49.93ID:pWn/OWnV
clを活字体のaと見違ったとか?
0352132人目の素数さん垢版2023/01/09(月) 12:20:35.23ID:xGoFPm4X
accumulationのaじゃないの
complemntで^cを使っちゃったのでは
0353132人目の素数さん垢版2023/01/09(月) 15:43:01.25ID:TdbGR1yR
そういえば昔はよく^aも使っていたなと確かめたら
数学辞典第4版でも色々な中の一つとしてあげてある
adherentのaだそうだ
0355132人目の素数さん垢版2023/01/09(月) 20:16:55.69ID:XPYlyFXZ
集積点って閉包の元(触点)のうち孤立点でないものじゃなかったっけ
触点そのものをaccumulated pointと呼ぶことある?
0356132人目の素数さん垢版2023/01/09(月) 21:24:17.33ID:l/SgpgpA
>>355
すまん、解析系の英語よく分からん
とりあえず集積値、英語でググったら出てきただけで
0357132人目の素数さん垢版2023/01/10(火) 00:21:32.64ID:lER0ff0g
>>355
accumulateしてないから呼ばない
0358132人目の素数さん垢版2023/01/10(火) 08:20:41.69ID:S6YN2KAV
M が R^n の凸集合ならば、その閉包 M^a も凸集合であることを示せ。

a, b ∈ M^a とする。

ε を任意の正の実数とする。

||a' - a|| < ε
||b' - b|| < ε

をみたす M の点 a', b' が存在する。

0 ≦ t ≦ 1 とする。

c := (1 - t) * a + t * b
c' := (1 - t) * a' + t * b'

とおく。

M は凸集合であるから、 c' ∈ M である。

||c' - c|| = ||(1 - t) * (a' - a) + t * (b' - b)||

(1 - t) * ||a' - a|| + t * ||b' - b||
<
(1 - t) * ε + t * ε
=
ε

よって、 c ∈ M^a である。

ゆえに、 M^a は凸集合である。
0359132人目の素数さん垢版2023/01/10(火) 08:59:19.15ID:S6YN2KAV
数学的帰納法ではない超限帰納法って使う機会がありますか?
0360132人目の素数さん垢版2023/01/10(火) 11:03:43.16ID:lER0ff0g
>>359
扱う集合に制限が無い場合とかでは有るんじゃナイ?
0361132人目の素数さん垢版2023/01/10(火) 22:08:12.40ID:xhk9SK5Z
>>345
大学院あるいは研究者レベルなのでしょうか。
0362132人目の素数さん垢版2023/01/10(火) 22:25:42.56ID:mKyw77m0
198x-yが272の倍数
⇔2178x-11yが272の倍数
⇔2x-11yが272の倍数
0363132人目の素数さん垢版2023/01/11(水) 10:44:05.36ID:AaBuTt5e
小学校レベルでしたすまないです
0364132人目の素数さん垢版2023/01/11(水) 11:10:03.99ID:K/hifS/8
約数倍数とか素因数分解関係するから
中学生レベルかな
0365132人目の素数さん垢版2023/01/11(水) 11:47:14.22ID:z8/JADrJ
>>360
松坂和夫著『集合・位相入門』

Zornの補題の証明で超限帰納法を使っていました。

もっと具体的な定理の証明で使われることはなさそうですね。
0366132人目の素数さん垢版2023/01/11(水) 11:50:03.78ID:z8/JADrJ
松坂和夫著『集合・位相入門』

でのZornの補題の証明ですが、分かりにくいですね。

同じ著者の解析入門シリーズでは、Zornの補題の証明(Bourbakiによると書いてあった)はもっと分かりやすかったと思います。
0367132人目の素数さん垢版2023/01/11(水) 13:40:25.41ID:z8/JADrJ
松坂和夫著『集合・位相入門』

のZornの補題の証明を読んでいて思ったのですが、
普段、選出公理を「選出公理により」などとは書かずに
勝手に好きに使っているにもかかわらず、Zornの補題の
ところになると必ずその証明に選出公理が使われていると
強調するのはなぜでしょうか?
0368132人目の素数さん垢版2023/01/11(水) 13:57:10.28ID:z8/JADrJ
松坂和夫著『集合・位相入門』

p.110
補題3

A を極大元をもたない順序集合とすれば、 A から A への写像 φ で、 A のすべての元 x に対して
φ(x) > x となるものが存在する。

これなんて、選出公理というのがあるということを知らなければ、「明らか」の一言で済ましてしまう
命題ですよね。
0369132人目の素数さん垢版2023/01/11(水) 15:42:40.89ID:AaBuTt5e
>>362
ありがとうございました。
ちなみに、前もって同値といわれてなくても
「198x-yが272の倍数」「2x-11yが272の倍数」をみればすぐに「同値だろ」って
数学科ならわかるもんなんですか?
0371132人目の素数さん垢版2023/01/11(水) 16:44:42.75ID:rg6smOBw
確率変数Xがf(x)に従い、確率変数Yがg(x)に従うとする。
このとき確率変数Z=aX+bYはどんな密度関数に従うのでしょうか?

P(Z<=c)=∫∫_[D]af(x)+bg(y)dxdy
D={(x,y):ax+by<=c}

と思ったのですが、これではR^2上で積分しても確率が1になりません。

同様に確率変数W=X*Yはどうなるのでしょうか?
0372132人目の素数さん垢版2023/01/11(水) 18:52:50.69ID:3pGlE+2C
convolutionで調べると有益な情報が得られると思います
0373132人目の素数さん垢版2023/01/11(水) 19:16:03.85ID:2ceBPpht
>>371
>確率変数Xがf(x)に従い、確率変数Yがg(x)に従うとする。
独立なの?
0374132人目の素数さん垢版2023/01/11(水) 19:32:22.38ID:3pGlE+2C
以下の問題を考えて下さい

連続的確率変数Xに対して新しい確率変数YをY=aX+b a>0と定める。X、Yの確率密度関数をそれぞれf(x)、g(y)とするときg(y)=(1/a)f((y-b)/a)を証明せよ
0375371垢版2023/01/11(水) 19:51:52.03ID:rg6smOBw
みなさんコメントありがとうございます。

>>372
和についてはなんとなくわかりました。
もう少し計算してみます。

>>373
申し訳ないです。
X,Yは独立です。

>>374
感覚的にはなりそうな気がします。
証明考えてみます。
0376132人目の素数さん垢版2023/01/11(水) 21:35:47.35ID:K/hifS/8
>>375
>X,Yは独立です。
独立なら
P((X,Y)∈D)=∬_D f(x)g(y)dxdy
ですよ
0377132人目の素数さん垢版2023/01/12(木) 10:48:40.66ID:+Y9GHwvf
>>368,370
寧ろ選択公理を前提に「明らか」じゃなくて?
0378132人目の素数さん垢版2023/01/12(木) 13:12:00.59ID:+bxs2kcf
松坂くんは掘り返そうとするのはいいけど結局何も掘り起こすことがなく直観が育たず重箱の隅をつつくばかりで何も進歩が無くこれからも質問し続ける

多変数の微積分 多様体 微分形式や位相の初等的教科書の存在理由を考えてアスペ丸出しの質問はやめてほしいと思う
0379132人目の素数さん垢版2023/01/12(木) 13:15:27.18ID:aDi05gu9
(行列を扱っていた時代の)高校数学から、一歩進むために編入試験数学している。
多変数微積線形代数はもちろん(工学向けの厳密証明無)の複素積分計算とか。
編入物理は微積使った古典物理とかか。(編入物理は実質数学の解析力学無)
0380132人目の素数さん垢版2023/01/12(木) 15:15:37.97ID:MHb5H1Rg
松坂和夫著『集合・位相入門』

p.212 定理13 (Tychonoff)

の証明を読み終わりました。

こういう抽象的な証明は論理を負うだけで十分ですよね。
0381132人目の素数さん垢版2023/01/12(木) 15:19:26.74ID:MHb5H1Rg
こういう定理は、結果だけ覚えれば十分ですよね。
0382132人目の素数さん垢版2023/01/12(木) 15:22:49.06ID:MHb5H1Rg
松坂和夫著『集合・位相入門』ですが、やっと最後まで読み終われそうです。

長いロジックゲームみたいなもんですね。
0383132人目の素数さん垢版2023/01/12(木) 15:25:19.50ID:MHb5H1Rg
松坂和夫著『集合・位相入門』の距離空間の章は簡単ですよね?

おそらく、この本の最大の難所はTychonoffの定理ですよね?

そこを乗り越えたので、最後まで読めると思います。
0384132人目の素数さん垢版2023/01/12(木) 16:00:02.61ID:aDi05gu9
定理・公式は証明できる必要がある、という意見も聞く。
高校数学レベルまでは公式等は範囲は決まってるから証明できて、なおかつ空で暗記している必要あるけど。(公式導出していてはテストでタイムロス)
0385132人目の素数さん垢版2023/01/12(木) 16:24:42.12ID:wB+npVq3
選択公理を自明だと思ってはダメという話の後で「この証明のこの部分で使ってます」と注意書きするのは当たり前
話が進んだ後では「どこに使われているか」を一々強調する必要がないことも当たり前
0386132人目の素数さん垢版2023/01/12(木) 23:06:52.93ID:hxlQYgUb
選択小売りって当たり前すぎませんか?
認められないひとは何が不満なんですか
0387132人目の素数さん垢版2023/01/13(金) 00:26:50.05ID:bBNosLwt
球が2倍になるとか
0390132人目の素数さん垢版2023/01/13(金) 12:44:42.54ID:gdldVpFp
C_λ (λ ∈ Λ)を位相空間 S のコンパクト閉集合とする。
A_λ (λ ∈ Λ)を位相空間 S の閉集合とする。

このとき、 (∩ C_λ) ∩ (∩ A_λ) は S のコンパクト閉集合である。

これは、 C を S のコンパクト集合、 A を S の閉集合とすると、 C ∩ A は C のコンパクト閉集合になるからですか?
0391132人目の素数さん垢版2023/01/13(金) 13:48:46.53ID:0dt8e/mh
>>390
コンパクトって閉集合じゃ無かったっけ?
0392132人目の素数さん垢版2023/01/13(金) 14:04:44.19ID:MTgkXF+h
わからないんですね
0394132人目の素数さん垢版2023/01/13(金) 16:45:42.34ID:gdldVpFp
松坂和夫著『集合・位相入門』

1点コンパクト化について書いてありますが、その応用が書いてありません。

こういうのはどうなんですか?
0395132人目の素数さん垢版2023/01/13(金) 16:55:10.73ID:0dt8e/mh
>>394
必要になれば思い出せばいいんじゃないの?
0396132人目の素数さん垢版2023/01/13(金) 17:40:47.80ID:3oZtV3BI
>>391
有限集合が必ず閉集合だと思ってそう
0397132人目の素数さん垢版2023/01/13(金) 18:35:16.89ID:YH1Vd150
>>391
ザリスキ位相を学んだとき、そこで混乱した。
0398132人目の素数さん垢版2023/01/13(金) 19:12:11.43ID:gdldVpFp
位相はなぜ重要なんですか?
0399132人目の素数さん垢版2023/01/13(金) 19:17:54.67ID:bBNosLwt
そうか
ハウスドルフ性が要るのか
0400132人目の素数さん垢版2023/01/13(金) 19:25:29.99ID:MTgkXF+h
>>398
連続性を定義できるからです
0401132人目の素数さん垢版2023/01/13(金) 19:26:44.20ID:gdldVpFp
>>400

連続性はなぜ重要なんですか?
0402132人目の素数さん垢版2023/01/13(金) 19:36:29.76ID:0iZLoNYs
>>401
こういうことが分からずに数学(位相)を勉強しても何もならない。
独学の恐ろしい負の側面を見た。
なるほどこいつの勉強がいつも中途半端に終わる訳が分かった。
0403132人目の素数さん垢版2023/01/13(金) 19:51:48.02ID:gdldVpFp
微分積分において、連続性が重要なのは分かります。

一般の位相空間で連続性はなぜ重要なんですか?
0404132人目の素数さん垢版2023/01/13(金) 20:11:27.17ID:0iZLoNYs
>>403
この馬鹿は連続性の重要性が分からない。分かろうともしないで重箱の隅をつついて残り少ない人生を終える。
0405132人目の素数さん垢版2023/01/13(金) 20:19:36.93ID:gdldVpFp
やっと、1点コンパクト化のところを読み終わりました。

あとは、「分離公理」のセクションを読めば第5章を読み終わることになります。

そうなれば、あとは距離空間の章だけとなります。

距離空間は位相空間の一般論と比べれば具体的なので簡単に読めるだろうと推測します。
0406132人目の素数さん垢版2023/01/13(金) 20:19:48.57ID:iioOhFF1
独学が危険なんじゃなくて、重箱の隅をつつき続ける性格だから独学にならざるを得なかっただけだと思うけど
0407132人目の素数さん垢版2023/01/13(金) 21:08:01.01ID:bBNosLwt
>>403
位置(topos)の様相だから位相(topology)
それを保存する写像だからだよ
0408132人目の素数さん垢版2023/01/13(金) 22:24:02.41ID:xVntyZvm
重要だとも面白いとも思わない本を読み続けてる時点で自分には理解し難い類の人間なんだろうな
0410132人目の素数さん垢版2023/01/14(土) 13:14:32.12ID:rs7nZxqF
S および ∅ および S の有限部分集合全体からなる S の部分集合系を A とする。
その閉集合系が A となる位相を O とする。

(S, O) はコンパクトであることを示せ。
0411132人目の素数さん垢版2023/01/14(土) 13:22:18.43ID:rs7nZxqF
S = ∪_{λ∈Λ} O_λ とする。
λ_1 ∈ Λ such that O_{λ_1} ≠ ∅ とする。
そうでなければ、 S - O_{λ_1} は有限集合である。
S - O_{λ_1} = {a_2, …, a_n} とする。
S - O_{λ_1} ⊂ ∪_{λ∈Λ-{λ_0}} O_λ である。
a_2 ∈ O_{λ_2} となるような λ_2 が存在する。

a_n ∈ O_{λ_n} となるような λ_n が存在する。
S - O_{λ_1} ⊂ O_{λ_2} ∪ … ∪ O_{λ_n} が成り立つ。
S ⊂ O_{λ_1} ∪ … ∪ O_{λ_n} が成り立つ。
S = O_{λ_1} ∪ … ∪ O_{λ_n} が成り立つ。
∴ S はコンパクトである。
0412132人目の素数さん垢版2023/01/14(土) 13:23:55.54ID:rs7nZxqF
訂正します:

S = ∪_{λ∈Λ} O_λ とする。
λ_1 ∈ Λ such that O_{λ_1} ≠ ∅ とする。
S - O_{λ_1} は有限集合である。
S - O_{λ_1} = {a_2, …, a_n} とする。
S - O_{λ_1} ⊂ ∪_{λ∈Λ-{λ_0}} O_λ である。
a_2 ∈ O_{λ_2} となるような λ_2 が存在する。

a_n ∈ O_{λ_n} となるような λ_n が存在する。
S - O_{λ_1} ⊂ O_{λ_2} ∪ … ∪ O_{λ_n} が成り立つ。
S ⊂ O_{λ_1} ∪ … ∪ O_{λ_n} が成り立つ。
S = O_{λ_1} ∪ … ∪ O_{λ_n} が成り立つ。
∴ S はコンパクトである。
0413132人目の素数さん垢版2023/01/14(土) 18:41:26.90ID:YDar5/al
>>410
有限交差性を持つ閉集合族 B (⊂ A)
つまり [ ∀n ∀ x₁, x₂, ... , xₙ ∈ B ] ∩[k=1..n] xₖ ≠ ∅.
これが常に無限交差性 ∩[x∈B] x ≠ ∅ を持つことを示せばよい.

∩[x∈B] x = ∅ を仮定すると,
特定の要素 x₀={a₁,a₂,...,aₕ} (∈B) に対しては, a₁を含まない x₁, ..., aₕを含まない xₕ が存在するはずである ( x₀, x₁, ..., xₕ ∈ B ).
このとき ∩[k=1..h] xₖ = ∅ つまり有限交差性に反する.
よって ∩[x∈B] x ≠ ∅.

対偶をとって開集合族C (⊂O) で書き換えると
∪[u∈C] u = S ⇒ [∃n ∃u₁, u₂, ... , uₙ ∈ C] ∪[k=1..n] uₖ = S.
0414132人目の素数さん垢版2023/01/14(土) 20:31:51.93ID:rs7nZxqF
S_1, S_2 を位相空間とする。

(x, y) ∈ S_1 ✕ S_2 とする。

(x, y) の近傍系はどうあらわされますか?
0415132人目の素数さん垢版2023/01/14(土) 20:34:45.59ID:rs7nZxqF
V_{S_1}(x) ✕ V_{S_2}(y) ⊂ V_{S_1 ✕ S_2}((x, y))

は成り立ちますよね?
0416132人目の素数さん垢版2023/01/14(土) 21:41:12.77ID:rs7nZxqF
S_1, S_2 を位相空間とする。

U ✕ V ⊂ S_1 ✕ S_2 とする。

U ✕ V の開核は U の開核 ✕ V の開核と等しいか?
0417132人目の素数さん垢版2023/01/14(土) 21:57:25.94ID:7VHB9XzW
>>416
直積位相でしょ?開基から作り直し
0418132人目の素数さん垢版2023/01/14(土) 22:15:40.79ID:rs7nZxqF
松坂和夫著『集合・位相入門』

S_1, S_2 を位相空間とする。
A を S_1 ✕ S_2 の閉集合とする。
(x, y) ∉ A ならば、 (U ✕ V) ∩ A = ∅ となるような x の近傍 U および y の近傍 V が存在する。
U は S_1 の開集合と仮定してよい。

というような流れがあります。


「U は S_1 の開集合と仮定してよい。」という箇所が不自然ではないですか?

以下のように、自然な議論をしていけば、 U は x の開近傍に自然になるからです。

(x, y) ∈ S_1 ✕ S_2 - A で、 S_1 ✕ S_2 - A は開集合だから、
(x, y) ∈ U ✕ V ⊂ S_1 ✕ S_2 - A となるような S_1 の開集合 U、 S_2 の開集合 V が存在する。
もちろん、 U は x の近傍であり、 V は y の近傍である。
0419132人目の素数さん垢版2023/01/14(土) 22:18:10.54ID:7VHB9XzW
>>418
>自然な議論をしていけば
その結果だからでしょ?
0420132人目の素数さん垢版2023/01/14(土) 22:18:55.53ID:rs7nZxqF
松坂さんの議論では、 U と V が近傍であるということは重要ではなく、
U と V が開集合であるということが本質的です。
0421132人目の素数さん垢版2023/01/14(土) 22:22:01.73ID:rs7nZxqF
>>418

の流れがあるのは、

S_1 をコンパクトな位相空間、 S_2 を位相空間とすると、
S_1 ✕ S_2 ∋ (x, y) → y ∈ S_2 は閉写像である。

という問題の解答の中です。
0422132人目の素数さん垢版2023/01/14(土) 22:32:05.91ID:7VHB9XzW
>>420
でそれは「自然な議論」で自明としているのでしょうね
0423132人目の素数さん垢版2023/01/14(土) 22:56:45.32ID:HLLh5ayS
WMA は数学やってたら最もよく使う中心的な考え方のひとつ
それに初めて出あったときどんな反応なのかでその人の才能が見てとれる
0424132人目の素数さん垢版2023/01/14(土) 23:58:57.12ID:wLVCboGc
The Wellington Mathematics Association
0425132人目の素数さん垢版2023/01/15(日) 00:38:56.79ID:UWdCVrxn
WMA Neko is Kichigai.
0426132人目の素数さん垢版2023/01/15(日) 13:40:04.64ID:yoyvzBBE
位相空間の分離公理ですが、人工的に設定した公理ですよね。

位相の応用を勉強するときに、ふさわしい分離公理について勉強するというのがいいのではないでしょうか?

一般位相の本に書いてある分離公理は飛ばしてもOKなのではないでしょうか?
0427132人目の素数さん垢版2023/01/15(日) 13:43:35.84ID:yoyvzBBE
多様体について、

Tuの有名な本とJohm M. Leeの『Introduction to Smooth Manifolds 2nd Edition』のどちらがおすすめですか?

Leeの本は辞書みたいに分厚いですが。
0428132人目の素数さん垢版2023/01/15(日) 13:44:07.87ID:yoyvzBBE
訂正します:

多様体について、

Tuの有名な本とJohn M. Leeの『Introduction to Smooth Manifolds 2nd Edition』のどちらがおすすめですか?

Leeの本は辞書みたいに分厚いですが。
0429132人目の素数さん垢版2023/01/15(日) 13:47:09.85ID:JsmYsaf5
正直位相がっつりやるのでもなければ、ハウスドルフ空間さえわかれば分離公理周りはいらない気がしますよね
0430132人目の素数さん垢版2023/01/15(日) 14:19:30.24ID:Kr5k4CZx
>>429
代数幾何ではザリスキ位相が一般的だから
定義だけは認識しておくべきかと
歴史的にはコンパクトは
ハウスドルフを前提に導入されたが
代数多様体はハウスドルフではないので
>>391のような誤認?ブレ?を招く
0431132人目の素数さん垢版2023/01/15(日) 16:20:05.05ID:Dv0qPTtZ
T1は一点が閉集合となる空間のことでごくありふれた空間
T2(ハウスドルフ)は言わずもがな
T3はあまり要らんかも
T4(+T1=正規空間)はその抽象度合いにも関わらずウリゾーンやティーツェのようにRという具体的なものに関係してくるという事実くらいは知っていたほうがよいだろう
0432132人目の素数さん垢版2023/01/15(日) 21:18:23.61ID:yoyvzBBE
コンパクトなHausdorff空間 S は正規である。

証明が結構面白いですね。
0433132人目の素数さん垢版2023/01/15(日) 22:10:39.11ID:yoyvzBBE
S が正則空間で第2可算公理を満足するならば、 S は正規空間である。

この定理の証明はどうやって思いついたんですかね?

この証明も面白いですね。
0434132人目の素数さん垢版2023/01/15(日) 22:23:33.80ID:yoyvzBBE
Urysohnの補題って、いかにも成り立ちそうな補題ですが、どんな場面で応用されるんですか?
0435132人目の素数さん垢版2023/01/16(月) 04:49:53.43ID:MojHV22l
初歩的な質問で申し訳ないんだけど、今環論の勉強を始めたばかりで、「単項イデアル」の概念を学んだんだけど単項イデアルでないイデアルってどういう例があるの?
0436132人目の素数さん垢版2023/01/16(月) 04:53:20.30ID:bPykN0ED
>>435
R[x,y]⊃I=(x,y)={P:定数項のない多項式}
0437132人目の素数さん垢版2023/01/16(月) 05:21:36.66ID:jmQQeZ02
>>436
確かにn変数多項式を想像するとわかりやすいな
ありがとう
0438132人目の素数さん垢版2023/01/16(月) 12:07:43.90ID:PnUys56B
Urysohnの補題の証明を読み終わりました。

この証明も面白いですが、どうやって思いついたんですか?
0439132人目の素数さん垢版2023/01/16(月) 17:32:19.91ID:rKP3wIRa
>>426
グロタンディークがEGA執筆中に達した考えにあなたも達したわけだ、お見事。


Fourier Analysis in Several Complex Variables:Leon Ehrenpreis
を読んでみてください
数セミの小松彦三郎さん追悼文に、この本は各ページに間違いがあると書いてありました。
0441132人目の素数さん垢版2023/01/16(月) 18:07:27.26ID:WUc6XIWM
>>440
どのレスとどのレスの書き手が同じか
ちょっと教えてもらえますか?
0442132人目の素数さん垢版2023/01/16(月) 22:06:40.05ID:06j3NB39
カントールの対角線論法について素朴な疑問なんだけど、
被らない数列をどう作り出そうが無限の中に元々存在する数列になるんではないのかって思うんだが
俺は間違ってるんだろうか
0443132人目の素数さん垢版2023/01/16(月) 22:32:13.95ID:PnUys56B
松坂和夫著『集合・位相入門』

p.233 問題5ですが、「完全正則空間は正則空間であることを示せ」という問題の解答
が不必要に難しいです。

f^{-1}((-1/2, 1/2)) ∋ x_0
f^{-1}((1/2, 3/2)) ⊃ A

であり、

f^{-1}((-1/2, 1/2))
f^{-1}((1/2, 3/2))
がdisjointな2つの開集合であることから、完全正則空間は正則空間であることが容易に分かります。
0444132人目の素数さん垢版2023/01/17(火) 00:07:03.30ID:MQW/SESK
>>442
有限で
1111
1010
0110
0001
から対角線論法で作られるのは
0100
ですが元の4つにない理由が分かればそんなこと思いも寄らないはずですが
0446132人目の素数さん垢版2023/01/17(火) 01:20:48.07ID:Cv0G9A7d
>>444
いや無限の中にその数列元々あるだろって話
有限の話ではない
0447132人目の素数さん垢版2023/01/17(火) 01:24:48.02ID:DMZOyFDW
だから、無限にも色々あって、ある場合とない場合がありますよねってのがカントールの対角線論法ですよね
0448132人目の素数さん垢版2023/01/17(火) 01:34:15.64ID:VMB7SaIO
>>445
教科書によく出てくる例↓
{ ( 0, t ) ; -1≦ t ≦ +1 } ∪ { ( t, sin(1/t) ) ; 0< t ≦1 }
これが「連結」であることを示せって練習問題にありがちー
0450132人目の素数さん垢版2023/01/17(火) 01:55:14.78ID:1KmICKnf
>>448
それが弧状連結でないことを示せって問題出すとまともな答案ほとんどないとかもありがちー
0451132人目の素数さん垢版2023/01/17(火) 07:29:57.97ID:MQW/SESK
>>446

有限の場合に必ず違うモノになるわけは分かった?
無限の場合にも理由は同じ
0452132人目の素数さん垢版2023/01/17(火) 21:38:26.70ID:qb3Ps0Tm
数学科の先輩なのですが
お付き合いはしていない、というかできないのですが、両想いはほぼ確定で、この間、10回目くらいとなる自宅にお邪魔して、一緒に映画を観ました。
その映画が過激な内容を多く含む映画だったので、彼が変な気持ちになったのかもしれませんが、
ふくらはぎや太ももを触られました。
これは前回もあって、もともといやらしくないスキンシップの多いかたなので、好意を示してくれているんだな、と私は恥ずかしがったりビクビクしながらも、嫌悪感は抱きませんでした。好きな相手だからこそなのですが。
最初のうちは、「うわ、すべすべだねー」なんて軽口を言いながらじゃれていたのですが、急に無言になり、ゆっくりと下着がぎりぎり見えないぐらいまでスカートをたくし上げられ、脚の付け根を撫でられたときには、男性経験のない私も「あれ?」と思いました。
太ももを撫でられているあたりから、私は声が出そうになってしまって、口元を軽く抑えながら我慢していました。それに夢中になっていると、今まで決して触れることのなかった股間に、下着の上から少しだけ触られました。
といっても、膣を触られたわけではなく、その上あたりを軽く、とんとん、と指で叩かれるように触られました。
私はびっくりしたやら、感じてしまったやら、恥ずかしいやらで、声を抑える吐息混じりに泣いてしまいました。でも、不思議とそこに嫌悪感は無く、嬉しかったです。
彼はとっさに触るのをやめ、「ほんとにごめん、理性がとんだ」「 びっくりしたよね、もうしない」と何度も謝ってくれ、頭を撫でたり、ハグしてくれました。
彼は私に経験がないことを知っているので、怖がらせたと、本当に罪悪感を感じたらしく、「もう連絡とるのやめようか?」と提案までしたのですが、それは絶対に嫌だったので、泣きながら首を横に振りました。
私が怒っていない旨を伝えても、まだ申し訳なさそうな様子でした。
0453132人目の素数さん垢版2023/01/17(火) 21:40:42.02ID:qb3Ps0Tm
彼は明朗快活かつ頭が切れ、会話の引き出しが多くコミュニケーション能力も高いです。そのおかげで気まずい雰囲気を引きずらず、今では前みたいに楽しく会話しているのですが、前のような軽めのスキンシップさえ無くて、寂しいです。
怖がらせないようにという彼の配慮なのはわかります。
でも、はしたない女だと思われるかもしれませんが、本当に嫌じゃなかったし、もう一度してほしいとさえ思います。
態度には出さずとも、恐らく未だ罪悪感を感じているであろう彼に、
大丈夫だから、もう一度してほしいな、という旨を自然に伝えたいです。
よろしくお願いします。
0454132人目の素数さん垢版2023/01/17(火) 22:32:26.31ID:pQX0zaUG
わかいってすばらしい
0455132人目の素数さん垢版2023/01/18(水) 08:53:17.73ID:WxHQyTAk
そんな助駒氏野郎は数学はできないに違いないので
別れた方がよろしい
0456132人目の素数さん垢版2023/01/18(水) 09:30:31.36ID:aoNtqhKs
映画館であろうが山小屋であろうが
個室喫茶であろうが
することは変わらないのだから
場所を変えて再度チャレンジさせてあげたらよかろう
0457132人目の素数さん垢版2023/01/18(水) 16:43:28.20ID:uMDEH4Ba
>>444
4つにはないが、16個にはある。対角線論法は5番目〜16番目を無いことにして議論しているから問題。
0458132人目の素数さん垢版2023/01/18(水) 16:58:00.73ID:/m5/pdrS
>>457
は?
個数では無く
「そこにあるものの中に無い」
という理由が分かってるのかな?
可算無限でもそこに全部あるんですがw
0459132人目の素数さん垢版2023/01/18(水) 17:01:50.61ID:/m5/pdrS
>>457
君は集合Aから2^Aへの全射があると言いたいようだねw
0460132人目の素数さん垢版2023/01/18(水) 19:39:20.38ID:9iaz983i
位相空間族 (S_λ) の直積位相空間が完全正則空間であるための必要十分条件は、
各 S_λ が完全正則空間であることであることを証明せよ。
0461132人目の素数さん垢版2023/01/18(水) 20:39:29.93ID:69kDKgv6
>>460
その問題だと、各 S_λ は空でないと仮定していますよね?
0462132人目の素数さん垢版2023/01/18(水) 20:58:01.61ID:9iaz983i
>>460
の問題は、松坂和夫著『集合・位相入門』の問題です。

この本では、位相空間は空集合ではありません。
0463132人目の素数さん垢版2023/01/18(水) 21:04:20.81ID:69kDKgv6
位相空間 X が完全正則空間なるための必要かつ十分なる条件は,
X にその位相と両立する一様構造がつくことだから,
結論は明らかじゃないの?
0464132人目の素数さん垢版2023/01/18(水) 21:36:25.32ID:p1S5t3WY
>>451
無限の中にあるアナログになぜ有限の定理が当て嵌められる?
無限の中に有限を見出だしてるだけじゃねえか
0465132人目の素数さん垢版2023/01/18(水) 21:43:50.96ID:BkT9AYaf
リストにあるどの列とも小数点以下n位の数が異なるからリストに入らないという話ってそんなにわかりづらいのか……?
0466132人目の素数さん垢版2023/01/18(水) 22:06:34.24ID:IhkaGRPE
まず次の命題が正しいのはわかる?
「Xを集合、a∈X、Y⊆Xとする。全てのb∈Yに対してa≠bならばaはYの元ではない」
この命題ではYが有限か無限かは一切関係ない
0467132人目の素数さん垢版2023/01/18(水) 23:19:14.92ID:k8Sew8ME
>>464
まず有限の場合で対角線論法で得られるものが「その中にない」と認識出来れば
可算無限の場合でも(もっと多くてもだけど)「その中にない」のは同じ理由なんですが?
0468132人目の素数さん垢版2023/01/19(木) 03:22:07.66ID:/2ZTFZ+n
>>457
対角線論法が全数列をリストできるという理不尽を否定する論法ってのはわかってるのか?
その例だと12個も漏れちゃうくらいに自然数という入れ物が小さすぎるんよ

>>465
もっと単純なヒルベルトの無限ホテルですら分からないって人がいるからなぁ
どの部屋にも客がいるのに満室じゃないとか言いだす
0470132人目の素数さん垢版2023/01/19(木) 07:02:22.09ID:Ww3J2oi+
>>468
>対角線論法が全数列をリストできるという理不尽を否定する論法ってのはわかってるのか?
ああそうか
彼は「対角線論法」が「背理法」だと認識してないんだな
0472132人目の素数さん垢版2023/01/19(木) 09:06:05.83ID:Jj+BcxCf
>>460

の解答ですが、

math-prof-test.mods.jp/kaneko/pdf-TS/matsuzaka.5-3.pdf

の7(δ)に書いてあります。

この解答中の f ですが、間違っていますよね?

f = 1 - (1- f_1) * (1 - f_2) * … * (1 - f_m) が正しいですよね?
0473132人目の素数さん垢版2023/01/19(木) 13:19:40.05ID:WOYU/1C+
>>468
対角線論法の問題点の詳細
https://tnomura9.exblog.jp/24436044/

また、山本泉二の「数学の無限 ~カントールの無限を超えて」にも同様の指摘がある。
0474132人目の素数さん垢版2023/01/19(木) 13:45:07.87ID:Ww3J2oi+
>>473
問題点は無いということが分からないんですね
0475132人目の素数さん垢版2023/01/19(木) 14:01:43.92ID:Ww3J2oi+
無限公理が嫌いな人も居ると思うので
無限公理無しの数学
有限数学?を1から作り上げてみてはどうかな
0476132人目の素数さん垢版2023/01/19(木) 16:16:19.09ID:Jj+BcxCf
[0, 1] はコンパクト集合である。

証明:

R の部分空間 {0, 1} は有限集合だから、コンパクトである。
チコノフの定理により、 {0, 1} ✕ {0, 1} ✕ … ✕ {0, 1} ✕ … はコンパクト集合である。
{0, 1} ✕ {0, 1} ✕ … ✕ {0, 1} ✕ … から R への写像を
(a_1, a_2, …, a_n, …) → a_1/2 + a_2/4 + … + a_n/2^n + … で定義する。
この写像は連続だから、その像である [0, 1] もコンパクトである。
0477132人目の素数さん垢版2023/01/19(木) 16:16:52.36ID:Jj+BcxCf
>>476
の写像はなぜ連続なんですか?

証明をお願いします。
0478132人目の素数さん垢版2023/01/19(木) 16:54:32.61ID:Jj+BcxCf
小松彦三郎さんが、

「世界の指導的な数学者の講演は、さすがと思わせる説得力があるが、彼らの多くは、ちょっとした講義の
ためにもテキストを作り、それを憶えて、あたかもアド・リブのような印象を与えながら、すじ書き通り
首尾一貫した話をしていることを注意しておきたい。」

と書いています。

数学者って見えっ張りなんですね。
0480132人目の素数さん垢版2023/01/19(木) 18:19:01.23ID:0UeVB6I4
>>476
端点が2進有理数であるような開区間の逆像が開集合になること
[0, 1]の開集合は端点が2進有理数であるような開区間のいくつかの合併で表されること
この2つが成り立つかどうか考えてみて下さい
0481132人目の素数さん垢版2023/01/20(金) 02:38:17.98ID:GocIhPXv
任意の写像f:A→Bに対して適切にベクトル空間V,V'、単射な写像g,hをg:A→V、h:V'→B、線形写像mをm:V→V'として置くとf=h ∘ m ∘ gとなり得ますか
0482132人目の素数さん垢版2023/01/20(金) 04:45:05.09ID:GocIhPXv
集合S,T,Uについて、S=T/Uとなるとき、T=U×VとなるようにVを取ったとき右辺でTからUを削ってS=Vとなるのですか?
0483132人目の素数さん垢版2023/01/20(金) 07:31:19.11ID:EgCmbKYq
>>481
A,B=φなら無理
実数体上のベクトル空間ならBが有限集合では単射なhはV'が1点しかあり得ないから一般に無理
0484132人目の素数さん垢版2023/01/20(金) 07:32:29.54ID:EgCmbKYq
>>482
>集合S,T,Uについて、S=T/U
Sの定義は?
0485132人目の素数さん垢版2023/01/20(金) 07:43:39.16ID:yMHTp3H6
>>481
何を考えた上でその問題に行き着いたかを教えてほしい
多分設定を間違えている
0487132人目の素数さん垢版2023/01/20(金) 10:27:50.63ID:bwtFlHJ0
できるけど本当にそんなのが必要なのか
0488132人目の素数さん垢版2023/01/20(金) 10:35:49.00ID:bwtFlHJ0
h単射な写像か
0489132人目の素数さん垢版2023/01/20(金) 10:40:58.31ID:GocIhPXv
>>485
解説書でとある写像について前触れ無く写像の階数という概念が出てきたように思えたので、写像には一般的に階数があるのかと思いました
再度よく確認してみたところ、実際には解説書の中では写像は写像でも線形写像であることに言及があったので単なる私の勘違いでした
お騒がせしました
0492132人目の素数さん垢版2023/01/20(金) 13:10:15.92ID:LoAWX/vX
そういうことじゃなくて、バカは死ねって言いたいだけじゃないの
0494132人目の素数さん垢版2023/01/20(金) 14:36:58.81ID:EIsIuNo7
連結だが弧状連結でない空間の例として

t∈[0,1]として
X=[0,1]^2-∪_{n=1,∞}{(1/(2n),t(1-1/(2n)))}∪{(1/(2n+1),1-t(1-1/(2n+1)))}

というのを聞いた記憶があります。これって正しい主張ですか?
感覚的にはつながっているし、{(0,0)}と{(1,1)}という点は
連続写像c:[0,1]→Xで結べなさそうです。
0496132人目の素数さん垢版2023/01/20(金) 15:01:01.89ID:/UZ8j3YI
>>494
正方形から可算個の点を抜いただけじゃ?
0497132人目の素数さん垢版2023/01/20(金) 15:12:59.58ID:EIsIuNo7
すみません。

X=[0,1]^2-∪_{t∈[0,1]}∪_{n=1,∞}{(1/(2n),t(1-1/(2n)))}∪{(1/(2n+1),1-t(1-1/(2n+1)))}

です。
0498132人目の素数さん垢版2023/01/20(金) 15:14:10.09ID:JsIi6Ak0
連結かつ弧状連結で間違いないかと。
ついでにy方向に取り除く長さは一定(2/3ずつとか)でも大丈夫
0500132人目の素数さん垢版2023/01/20(金) 18:08:21.60ID:ABxSQxoh
ベーシック圏論の問題で
モノイドGを単対象圏とみなしたものをもGと書くし
関手G→Setを考えた時これは一般に忠実もしくは充満か?
という問題があり,その答えが忠実ではあるが充満ではないとなっています
しかしこの忠実であるというのは間違いではないでしょうか?

この関手は集合へのG作用を与えると見る事が出来ますが
これが忠実であるという事は群作用の意味での忠実性と一致するように思えます
よって忠実でない作用があるように上の関手も一般には忠実ではないのではと思ったのですが
自信がないので詳しい人いたら教えて欲しいです。
0502132人目の素数さん垢版2023/01/20(金) 19:25:53.90ID:ABxSQxoh
>>501
ありがとうございます
他の例を調べる問題でも解答がよくわからない点があったので
これと見比べながら読んでみようと思います。
英語だと解答がネットにあがってるというのは盲点でした大変ありがたいです。
0504132人目の素数さん垢版2023/01/20(金) 19:40:15.77ID:ABxSQxoh
>>503
丁寧にありがとうございます
こっちはチェックしていたのですがこの間違いはまだ正誤表には載ってないものでした
他にも気付く点があるかもしれないので読み終わったら出版社にでも送っておこうかと思います
0505132人目の素数さん垢版2023/01/20(金) 20:26:58.05ID:EgCmbKYq
>>500
>よって忠実でない作用があるように上の関手も一般には忠実ではないのではと思ったのですが
忠実ってある意味単射ってことでしょ
>関手G→Setを考えた時
どんな関手?
モノイドを圏と考えたとき
その単一のオブジェクトを集合としてのモノイドGに対応させ
圏として考えたときのモノイドの射すなわち集合としてのモノイドの元aをGからGへの作用a(x)=axとかxaとかに対応させる?
0506132人目の素数さん垢版2023/01/20(金) 20:35:12.14ID:EgCmbKYq
任意の関手では問題にならないでしょ
圏としてのモノイドGの単一のオブジェクトを空集合φに
射を全部φの恒等写像に対応させる関手では
忠実もへったくれもない
無意味に充満ではあるけどさ
0507132人目の素数さん垢版2023/01/20(金) 20:53:28.61ID:ABxSQxoh
>>505
関手F:G→SetでGのただ一つの対象*の行き先をXと書くと
Gの射(集合としてのモノイドでみた時のモノイドの元)はSetの射つまりmapを定めます
このmapに対して関手の基本性質を確認すると
それがまさに群作用の定義の結合法則や単位法則となっているため
任意の関手FがG作用の入った空間Xと同一視できます。

この読み替えの元で逆に忠実でない群作用のあるG空間Xを考えると
上の逆の読み替えにより単対象をXにうつしていると見れて
Gの異なる射で同じ作用を与えるものg,hを取ると
これはSetでのXからXへのmapとして等しい
つまりF(g)=F(h)となるので忠実ではないと思います。
506の例だと極端な場合なのでさらに分かりやすいですね。
空集合φに自明なG作用を入れればそれが忠実でない事と対応してるわけですよね。
参考になります。
0508132人目の素数さん垢版2023/01/20(金) 20:59:25.77ID:yMHTp3H6
>>506
非本質的な例を持ち出すのはあまり良くないよ
0509132人目の素数さん垢版2023/01/20(金) 21:16:23.92ID:ABxSQxoh
>>508
言われてみればそれもそうですね
特殊な例を除いてもごく普通にあり得る事が重要ですね
0510132人目の素数さん垢版2023/01/20(金) 21:47:40.84ID:EgCmbKYq
>>508
はぁ
その問題が任意の関手を考えているわけではないだろうという本質的な疑問(>>500の疑問へのね)なんですが?
0511132人目の素数さん垢版2023/01/20(金) 21:49:11.91ID:EgCmbKYq
>>508
君はその問題を読んだようだから
正確にここで問題文呈示して
0512132人目の素数さん垢版2023/01/20(金) 21:51:57.12ID:EgCmbKYq
>>503
その正誤表にその問題のその解答への言及がないようだが?
0513132人目の素数さん垢版2023/01/20(金) 21:55:09.22ID:EgCmbKYq
>>507
>関手F:G→SetでGのただ一つの対象*の行き先をXと書くと
想像だけどX=Gでa(x)=axとかxaとかが想定されてない?
君でも良いよ
問題文正確に呈示して
0514132人目の素数さん垢版2023/01/21(土) 13:55:47.30ID:dI7S+mP8
>>513
横からだけど気になったので。

英語版が無料で読めるから見てみたけど、問題1.2.28っぽい。
その章に出てきた関手が忠実か充満かそれぞれ答えよ、って問題で、 >>500 の例もその章にある。
特に関手に条件は付いてないから、単に訳者が解答付ける時にミスったか、「関手として忠実であることは作用が忠実であることと同値」ってことを言いたかったか、そんなところだろう。
0515132人目の素数さん垢版2023/01/21(土) 14:49:50.35ID:ZOMJ4Xnj
初歩的な質問なのですが、複素解析において
1/{1-(z/3)}=Σ[0,∞] (z/3)^n
と表せるのは何故なのでしょうか……?
整級数の分野で何か定理とかあったでしょうか……
0516132人目の素数さん垢版2023/01/21(土) 14:53:27.16ID:TKWH4rnP
高校生でもわかることだと思いますけど
0517132人目の素数さん垢版2023/01/21(土) 15:04:41.89ID:ZOMJ4Xnj
>>516
等比数列の無限和のやつを忘れてました……
勉強し直します
0518132人目の素数さん垢版2023/01/21(土) 15:30:06.96ID:M4j7+gKQ
>>514
>問題1.2.28
これか
>1.2.28 (a) Of the examples of functors appearing in this section, which are
>faithful and which are full?
>(b) Write down one example of a functor that is both full and faithful, one
>that is full but not faithful, one that is faithful but not full, and one that is
>neither
これのfaithfulの例として日本語訳の方では
>Example 1.2.8 Let G be a monoid, regarded as a one-object category G . A
>functor F : G → Set consists of a set S (the value of F at the unique object
>of G ) together with, for each g ∈ G, a function F(g) : S → S , satisfying
>the functoriality axioms. Writing (F(g))(s) = g · s, we see that the functor F
>amounts to a set S together with a function
>G × S → S
>(g, s) 7 → g · s
>satisfying (g′g) · s = g′ · (g · s) and 1 · s = s for all g, g′ ∈ G and s ∈ S . In
>other words, a functor G → Set is a set equipped with a left action by G: a left
>G-set, for shor
を挙げているということ?ホントに?日本語訳の方を読みたいね
あと
>As for the exercises, I join every other textbook author in exhorting you to do
>them; but there is a further important point. In subjects such as number theory
>and combinatorics, some questions are simple to state but extremely hard to
>answer. Basic category theory is not like that. To understand the question is
>very nearly to know the answer. In most of the exercises, there is only one
>possible way to proceed. So, if you are stuck on an exercise, a likely remedy is
>to go back through each term in the question and make sure that you understand
>it in full. Take your time. Understanding, rather than problem solving, is the
>main challenge of learning category theory
てことだから
日本語訳の方には解答が付いているとしたら
現著者の趣旨にはあって無いとも言えまいかね
0519132人目の素数さん垢版2023/01/21(土) 15:41:18.97ID:ojQRv+n7
>>500の者ですが挙げた問題はすでに出ている通りで1.2.28の(a)です
その中で本文中の例それぞれが忠実か充満かで○×の表がついており
その中の1.2.8のG作用の例に対する答えが忠実の方が○で充満の方が×に日本語訳の解答だとなっています
上で挙げてもらった>>501の個人が作ってる解答では正しくモノイドの作用が忠実でない時は関手も忠実でないと書いてありましたし
単純に訳者の勘違いかと思います
0521132人目の素数さん垢版2023/01/21(土) 16:11:56.72ID:M4j7+gKQ
>>520
持ってるから何よ
0522132人目の素数さん垢版2023/01/21(土) 16:24:19.99ID:gz6yy8lp
>>521
Gをモノイド、Cを
obj(C) = { G }
C(G,G) = G
と定義してunderlying func F : C → Setsは
C(G) = G
C(f) : g → fg (モノイドの積)
が定義ならFaithfulやん?
Gが単元持ってるんだから
0523132人目の素数さん垢版2023/01/21(土) 16:35:55.36ID:M4j7+gKQ
>>522
俺も最初そう思ってたが
君上の書き込み読んでないか
Fの定義はそれじゃ無く
単にモノイドの作用なんだと
トリビアルな作用でもいいんだと
0524132人目の素数さん垢版2023/01/21(土) 16:39:03.62ID:oUYC8qlh
>>523
“モノイドの作用”ってのが“モノイドの積による作用”じゃないの?
モノイドなんか積しか持ってないのに
>>522以外にどうやってCに圏の構造入れんの?
てか
>>522が最も簡単な圏の構成法でよく出てくる奴やん
どう考えてもこれの意味やろる
0525132人目の素数さん垢版2023/01/21(土) 16:44:22.37ID:Vnop0AY8
てか教科書の読み方おかしいよ
数学の用語なんて一応数学辞典的なものはあってもその場その場で著者は好きな書き方するのが当たり前
その時点で“そこではどういう意味で使ってるのか推察しながら読む”が基本、もちろん一冊の教科書内部では統一すべきだけどな
この問題は答えが“FaithfulだがFullではない”が答えなんだからそれが答えになるような意味で問題は作られてるんだよ、そこから問題の意味が推察できればそれでいい、むしろある程度以上の教科書はそういうふうに使うもん
0526132人目の素数さん垢版2023/01/21(土) 16:45:39.98ID:M4j7+gKQ
>>524
だから上の英文嫁って
0527132人目の素数さん垢版2023/01/21(土) 16:48:25.62ID:M4j7+gKQ
>>524
>Cに圏の構造入れんの?
圏としてのGは君の書いたCじゃない
単に射の全体がGであるてだけ
0528132人目の素数さん垢版2023/01/21(土) 16:51:16.54ID:M4j7+gKQ
Cじゃないは言い過ぎか
Cでもいいが
FでSetsに持って行くとき
GのGへの作用G×G→Gを
(g,h)→hにしてもいい
0529132人目の素数さん垢版2023/01/21(土) 16:53:42.52ID:M4j7+gKQ
>>525
>この問題は答えが“FaithfulだがFullではない”が答え
現著者の趣旨ではなさげ
訳者が追加したとすると
ちょっと勇み足と思うね
0530132人目の素数さん垢版2023/01/21(土) 16:55:22.15ID:Vnop0AY8
>>528
たからこの例は圏論の教科書でよく最初に書いてある例なんだよ
元々associative、monoidalな積をもつ集合Mがあってmor(C) = Mとなり、合成はMのそれをそのまま引き継ぐ例、同じ事を単元を持つ非可換環でやればadditive categoryが作れる
この程度で詰まってるといつまでも圏論なんかマスターできんよ
0531132人目の素数さん垢版2023/01/21(土) 17:00:52.27ID:M4j7+gKQ
>>530
うーん分かってないのかな
それがよくある例だってのは当然
俺も最初そう思ってたから>>505と書いたが
この本の趣旨はそれじゃ無かったって訳
0532132人目の素数さん垢版2023/01/21(土) 17:02:51.95ID:M4j7+gKQ
まあいいや
日本語訳のその部分どう書いてるか読んでみたいが
読んでみたところで重箱の隅つつくようなことだし
0534132人目の素数さん垢版2023/01/21(土) 19:10:51.38ID:uEwNVw2T
S を集合とする。
T を S の部分集合の有限集合とする。
∅ ∈ T とする。
S ∈ T とする。
T の任意の2元 U, V に対し、 U ∩ V ∈ T とする。
T の任意の2元 U, V に対し、 U ∪ V ∈ T とする。

T は位相空間であることを証明せよ。
0535132人目の素数さん垢版2023/01/21(土) 19:42:22.29ID:uEwNVw2T
以下であっていますか?

T は有限集合だから、 T = {U_1, …, U_n} と書ける。
各 λ ∈ Λ に対して、 U_λ ∈ T とする。
写像 Λ ∋ λ → U_λ ∈ T = {U_1, …, U_n} を f とする。

∪_{λ∈Λ} U_λ = ∪_{λ∈Λ} f(λ) = ∪ f(Λ)

数学的帰納法によって、 T の有限個の元たちの和集合は T の元だから、

∪ f(Λ) ∈ T である。

∪_{λ∈Λ} U_λ
0536132人目の素数さん垢版2023/01/21(土) 20:53:03.88ID:uE3JR9aM
>>533
お前誰?
0537132人目の素数さん垢版2023/01/21(土) 20:54:32.34ID:uE3JR9aM
>>533
>相手のレベル

君は>>530
どういうレベルと分かったのかな?
0538132人目の素数さん垢版2023/01/21(土) 20:58:40.99ID:uE3JR9aM
>>533

君は>>518
>>Example 1.2.8 Let G be a monoid, regarded as a one-object category G . A
>>functor F : G → Set consists of a set S (the value of F at the unique object
>>of G ) together with, for each g ∈ G, a function F(g) : S → S , satisfying
>>the functoriality axioms. Writing (F(g))(s) = g · s, we see that the functor F
>>amounts to a set S together with a function
>>G × S → S
>>(g, s) 7 → g · s
>>satisfying (g′g) · s = g′ · (g · s) and 1 · s = s for all g, g′ ∈ G and s ∈ S . In
>>other words, a functor G → Set is a set equipped with a left action by G: a left
>>G-set, for shor
を読んで
>>522
に限定した話だと思うレベル?
0539132人目の素数さん垢版2023/01/21(土) 21:00:24.00ID:uE3JR9aM
>>534
>T は位相空間
T?
0541132人目の素数さん垢版2023/01/21(土) 21:05:12.26ID:uEwNVw2T
S を集合とする。
T を S の部分集合の有限集合とする。
∅ ∈ T とする。
S ∈ T とする。
T の任意の2元 U, V に対し、 U ∩ V ∈ T とする。
T の任意の2元 U, V に対し、 U ∪ V ∈ T とする。

(S, T) は位相空間であることを証明せよ。
0542132人目の素数さん垢版2023/01/21(土) 21:05:54.80ID:uE3JR9aM
>>540
ショーモナ
こんな簡単な確認事項に答えられないんですね
0543132人目の素数さん垢版2023/01/21(土) 21:13:08.00ID:YNykr/ut
>>542
絡んでくんな能無し
お前なんか数学の世界にいらんわ
数学の世界に関わろうとすんな能無し
0544132人目の素数さん垢版2023/01/21(土) 21:32:56.05ID:uE3JR9aM
可哀想な人だな
0545132人目の素数さん垢版2023/01/22(日) 02:52:13.92ID:E1+PGcYI
コホモロジーについて質問です
コチェイン複体C、双対境界写像δに対して、n次コホモロジーは
Ker(δ^(n+1))/Im(δ^n)
で表されますが、この商集合はIm(δ^n)で定義される同値関係によるKer(δ^(n+1))の同値類なのでしょうか?
その場合、Im(δ^n)はどのような同値関係を定義しますか?
0547132人目の素数さん垢版2023/01/22(日) 03:05:05.53ID:jOdnlsnr
δは線型写像
商集合はただの商加群
0551132人目の素数さん垢版2023/01/22(日) 08:40:24.87ID:14WuuDhF
いや意外と書き込んだりツイートした瞬間に分かることあるんだよな
これはあるある現象だと思う
0552132人目の素数さん垢版2023/01/22(日) 09:53:16.90ID:hs4lTbyu
axiom of dependent choiceの訳語って従属選択公理なんですか?
0553132人目の素数さん垢版2023/01/22(日) 10:58:08.73ID:vcma0GG1
>>552
私が記憶していることによれば、そうです。
0554132人目の素数さん垢版2023/01/22(日) 12:19:12.20ID:Hhl1mMe6
(S, O) が位相空間となるための O に関する条件について質問があります。

松坂和夫著『集合・位相入門』のように、

∅ ∈ O
S ∈ O

を条件に含めている本が多いですが、これは不要ですよね。

なぜ、冗長な条件を採用しているのでしょうか?
0555132人目の素数さん垢版2023/01/22(日) 12:27:28.95ID:IR91d8OM
他の2つから示してみて
0556132人目の素数さん垢版2023/01/22(日) 12:35:17.44ID:Hhl1mMe6
(A)
Λ が有限集合とする。
各 λ∈Λ に対して、 O_λ ∈ O であるならば、 ∩_{λ∈Λ} O_λ ∈ O

Λ = ∅ は有限集合であり、 ∩_{λ∈∅} O_λ = S であるから、(A)より、 S ∈ O

(B)
各 λ∈Λ に対して、 O_λ ∈ O であるならば、 ∪_{λ∈Λ} O_λ ∈ O

∪_{λ∈∅} O_λ = ∅ であるから、(B)より、 ∅ ∈ O
0557132人目の素数さん垢版2023/01/22(日) 13:02:21.94ID:vcma0GG1
>>554
ブルバキ数学原論の位相の巻では、まさにその冗長な定義を回避しています。
0558132人目の素数さん垢版2023/01/22(日) 13:10:45.87ID:Hhl1mMe6
>>557

ありがとうございます。

ブルバキの本って最初の巻から順に読んでいかないと読みにくいですか?
0559132人目の素数さん垢版2023/01/22(日) 13:16:09.09ID:vcma0GG1
>>558

結論から言うと、最初から読んでいった方が無難です。

ブルバキの本には、『標準写像』とか『普遍写像問題』とか言う用語が結構あって、
それらの用語については、集合論の巻で取り決めごとや約束事があります。
そう言う取り決めごとを知らずに読み進めて行っても、わからないと思います。
(和訳の場合は、訳者の責任も大きいと思います。)

私もブルバキを読んで大学 2、3年レベルの学部数学の基礎を身につけました。
その時は、最初に集合論、その次に代数と位相を並行して読んでいきました。
その後は実一変数関数、位相線型空間、積分、多様体の順番です。

学生時代に代数系の同級生にブルバキを紹介したんですが、いきなり代数の間から
始めてしまって、全然わからない、と言って、彼は挫折してしまいました。
0560132人目の素数さん垢版2023/01/22(日) 13:18:43.03ID:Hhl1mMe6
>>559

ありがとうございました。

ブルバキの本はあまり読まれていないようです。
そういう高いハードルがあるから、読まれていないんですね。

集合論に興味がないので、第1巻を読む気になれません。
0561132人目の素数さん垢版2023/01/22(日) 13:22:52.42ID:vcma0GG1
>>560

集合論や基礎論が嫌いな方は相当数いらっしゃいますね。
私は基礎論は結構好きだったので、ブルバキの集合論には抵抗がありませんでした。

ただ、ブルバキの集合論は、基礎論の専門家からも、内容の構成について
結構な批判があります。

ブルバキにしてみれば、数学を論理的な基礎から構築するための、
ベースになる部分ですから、集合論は外せなかったのだと推察します。

私の感覚だと、数学科の学生が大学 1 年次の講義で学ぶ、
『集合論入門』が少々難しくなっただけ、と言う感じですね。
0562132人目の素数さん垢版2023/01/22(日) 13:40:59.14ID:Hhl1mMe6
>>561

ありがとうございました。
0563132人目の素数さん垢版2023/01/22(日) 13:54:06.18ID:hJfpiP0F
>>554
そっちの方が分かりやすいからじゃないですか?


>>556の∩_{λ∈∅} O_λ =Sというのは論理の小技でそうなるんだなぁという感じですけどいかにも無な感じの左辺がSになるのは変な感じします
あと位相の定義で「二つの開集合の共通部分は開集合である」みたいに個数が指定されてたら使えないですね
0564132人目の素数さん垢版2023/01/22(日) 14:13:28.61ID:EOIe8Rha
普通のよくある本の定義だと有限個の開集合「U_1,…,U_n」の共通部分に対する条件で、添字についてはn≧1が暗に仮定されてますね
その上で開集合族が空となることを回避するためでしょうね
0565132人目の素数さん垢版2023/01/22(日) 14:18:42.92ID:fxKeUWEw
そもそも興味もないし教科書でちゃんと勉強する気もないって言ってる人間相手にする必要がない
0566132人目の素数さん垢版2023/01/22(日) 14:52:02.34ID:PlHZzOS/
>>563
>∩_{λ∈∅} O_λ =S
単に集合論的にはこれは嘘よね
0567132人目の素数さん垢版2023/01/22(日) 16:32:57.23ID:PlHZzOS/
>>563
>いかにも無な感じの左辺
逆で
イカニモ何でもありそうな左辺
でしょ
一般に集合族Aに対して
∩A={x|∀X∈A (x∈X)}
とすると
∩(A∪B)=(∩A)∩(∩B)
なので
X=∩{X}=∩({X}∪φ)=(∩{X})∩(∩φ)=X∩(∩φ)⊂∩φ
がどんなXについても成立するべきなので
∩φは任意の集合を包含するわけ
“全体集合”Uが設定されている場合のみ
∩φ=U
0568132人目の素数さん垢版2023/01/22(日) 17:56:03.04ID:Hhl1mMe6
>>566-567

全体集合は S じゃないんですか?
0569132人目の素数さん垢版2023/01/22(日) 18:13:41.45ID:vcma0GG1
>>568
この種の議論も、ブルバキの集合論で定式化されています。
(X_i)_{i ∈ I} を集合 E の部分集合の族とする時,
(X_i)_{i ∈ I} の E に関する共通部分B = ∩_{i ∈ I} X_i を,
(∀x)(x ∈ B ⇔ [x ∈ E & (∀i∈I)(x∈X_i)])
で定義できます. ここで, I が空集合の時, 上の論理式による定義は
(∀x)(x ∈ B ⇔ x ∈ E)
となりますから, B = E
となります.

位相の定義で, S の任意有限個の開集合の共通部分は開集合と言う条件について,
その共通部分は, S に関する共通部分を考えます.
つまり, 上の議論で, E = S で,
0 個の開集合の族 (O_i)_{i∈φ} の共通部分 B = ∩_{i ∈ φ} O_iは,
B = S となります.

上記の議論も, 読者があらかじめ「集合 E に関する共通部分」と言うものを知っていれば
問題なくスムーズに進められるところなので, 先のレスでも、ブルバキに関しては
私は集合論から読んだ方が無難ですと言いました。
0570132人目の素数さん垢版2023/01/22(日) 18:54:08.74ID:Kc7GaHis
∩φみたいな変なのはP⇒QはPが偽ならば真であるというところから来てるから、そう定義したからそうなんだなぁという感じがする
0571132人目の素数さん垢版2023/01/22(日) 19:07:15.11ID:Hhl1mMe6
>>569

ありがとうございます。

(∀x)(x ∈ B ⇔ [(∀i∈I)(x∈X_i)])

だと、 I = ∅ のときに、おかしなことになってしまいますね。

ちょっとブルバキの集合論の本に興味が出てきました。

(∀x)(x ∈ B ⇔ [x ∈ E & (∀i∈I)(x∈X_i)])

↑これは他の集合論の本でも、標準的なんでしょうか?
0572132人目の素数さん垢版2023/01/22(日) 19:23:11.79ID:vcma0GG1
>>571
I が空集合の時は,
(∀x)(x ∈ B ⇔ [(∀i∈I)(x∈X_i)])・・・(1)
で B を定義すると, B は全ての集合からなる集合となってしまいます.
そのような B は存在すると仮定すると, ラッセルのパラドックスにより矛盾しますから,
式 (1) を満たす様な集合 B は存在しないわけです.
これは, >>571 さんのおっしゃる通りです.

論理式
(∀x)(x ∈ B ⇔ [x ∈ E & (∀i∈I)(x∈X_i)])
で E に関する (X_i)_{i ∈ I} の共通部分を定義する流儀は,
私はブルバキの集合論以外には見たことがありません.
0573132人目の素数さん垢版2023/01/22(日) 19:26:45.02ID:Hhl1mMe6
>>572

ありがとうございました。

James R. Munkres著『Topology Second Edition』には以下のように書いてあります:

∩_{A∈B} A = {x | x ∈ A for every A ∈ B}

全ての x がvacuouslyに「x ∈ A for every A ∈ B」を満たす。
問題はどの集合のすべての x なのかである。
議論の最初に大きな集合 X が与えられていて、その議論の間 X の部分集合のみを考えるのならば、

B = ∅ のときに、

∩_{A∈B} A = X

とするのはreasonableである。

しかしながら、すべての数学者がこのconventionに従っているわけではない。

困難を避けるために、我々は B = ∅ のときにはその共通部分を定義しない。
0574132人目の素数さん垢版2023/01/22(日) 19:28:49.40ID:Hhl1mMe6
訂正します:

>>572

ありがとうございました。

James R. Munkres著『Topology Second Edition』には以下のように書いてあります:

∩_{A∈B} A = {x | x ∈ A for every A ∈ B}

B = ∅ のときに、全ての x がvacuouslyに「x ∈ A for every A ∈ B」を満たす。
問題はどの集合のすべての x なのかである。
議論の最初に大きな集合 X が与えられていて、その議論の間 X の部分集合のみを考えるのならば、

B = ∅ のときに、

∩_{A∈B} A = X

とするのはreasonableである。

しかしながら、すべての数学者がこのconventionに従っているわけではない。

困難を避けるために、我々は B = ∅ のときにはその共通部分を定義しない。
0575132人目の素数さん垢版2023/01/22(日) 19:29:42.62ID:vcma0GG1
>>573
なるほど, Munkres の本では, I = φ のときの (X_i)_{i ∈ I} の共通部分は定義しないわけですね.
おそらくは, Munkres のその流儀の方が多数派ではないかと思われます.
0576132人目の素数さん垢版2023/01/22(日) 20:03:08.68ID:PlHZzOS/
>>563
>あと位相の定義で「二つの開集合の共通部分は開集合である」みたいに個数が指定されてたら
普通これでしょ
2個の共通部分と任意個の合併
0577132人目の素数さん垢版2023/01/22(日) 20:10:47.83ID:PlHZzOS/
>>570
でも自然よ
X=∩{X}=∩({X}∪φ)=(∩{X})∩(∩φ)=X∩(∩φ)⊂∩φ
同様に
X=∪{X}=∪({X}∪φ)=(∪{X})∪(∪φ)=X∪(∪φ)⊃∪φ
からこっちは問題なく
∪φ=φ
けど∩φ,∪φは定義しないというのが一番自然かもね
0578132人目の素数さん垢版2023/01/24(火) 00:01:30.10ID:HJKmKliC
1/(z^2+1)^2 を点a=iを中心としてローラン展開するやり方を教えて欲しいです
上手く部分分数分解するやり方が思い付かないんです
0580132人目の素数さん垢版2023/01/24(火) 01:07:50.22ID:HJKmKliC
>>579
分母がz^2+1の2乗になってるんですよね
1/(z^2+1)をそのように分解してローラン展開した後に2乗すればいいんですかね?
0582132人目の素数さん垢版2023/01/24(火) 07:30:00.33ID:3zpk086J
(z^2+1)^(-2)=(z-i)^(-2)(2i)^(-2)(1+(z-i)/(2i))^(-2).
0583132人目の素数さん垢版2023/01/24(火) 07:32:25.19ID:HJKmKliC
1/(z^2+1)^2
=-1/4•{1/(z-i)^2 - 1/i•1/(z-i) + 1/i•1/(z+i) + 1/(z+i)^2}
の形にしてそれぞれをローラン展開すれば良いんですかね?

>>581
(z-i)^2と(z+i)^2の求め方だけ教えていただいてもいいですか?
1/(z-i)ならわかるのですが2乗になってる時はどのように処理すれば良いのかが曖昧なんです
0585132人目の素数さん垢版2023/01/24(火) 09:24:27.78ID:tWj5cHT8
>>583
f(z)=1/(z+i)^2 : holo at i
f(z)=a0+a1(z-i)+a2(z-i)^2+…+an(z-i)^n+…
f(z)/(z-i)^2=a0/(z-i)^2+a1/(z-i)+a2+…+an(z-i)^(n-2)+…
an=f^(n)(i)/n!
(z+i)^2f(z)=1
(nC2)2f^(n-2)(z)+(nC1)2(z+i)f^(n-1)(z)+(z+i)^2f^(n)(z)=0
f(i)=1/(2i)^2
n(n-1)f^(n-2)(i)+2n2if^(n-1)(i)+(2i)^2f^(n)(i)=0
a0=-1/4
n(n-1)(n-2)!a(n-2)+4ni(n-1)!a(n-1)-4n!an=0
a(n-2)+4ia(n-1)-4an=0
0586132人目の素数さん垢版2023/01/24(火) 11:42:14.52ID:STWDWTuB
z=iの周りの展開なのに1/(z-i)^2の処理が分からんとか言ってるってことは
これ何も考えてないなとしか思えんよな……
0587132人目の素数さん垢版2023/01/24(火) 12:22:09.60ID:HJKmKliC
>>584,585
1/(z+i)^2の時はテイラーの定理に従って求めれば良いんですね
教えていただきありがとうございます!

>>586
確かにそうでした!
1/(z-i)^2はそのままで良いですね
1/(1-z)^(k+1)の形なら公式を使えば簡単に展開できるんですけど、分母にiが来ちゃったときにはどうすれば良いんだろうと悩んでたんです
0589132人目の素数さん垢版2023/01/24(火) 13:52:48.71ID:dG3n8Ma1
わからないんですね
0590132人目の素数さん垢版2023/01/24(火) 13:53:21.52ID:Wk48HG5c
なんとかちゃんねるで尋ねるだけで単位ゲット!
0592132人目の素数さん垢版2023/01/24(火) 14:52:27.17ID:3+nKQZg3
圏論で、射を矢印で表わすのはなぜでしょうか?

グラフ理論で、グラフを点と線で表わすのと同じことでしょうか?
0593132人目の素数さん垢版2023/01/24(火) 15:31:49.49ID:dG3n8Ma1
わからないんですね
0594132人目の素数さん垢版2023/01/24(火) 16:26:00.07ID:+wXipAUj
>>588
たしかにこれだけだと質問としても抽象的過ぎたので自分が作った問題5の解答を載せます 全然自信ないので添削をお願いします

Dが有界なので、D上の関数f(x,y)も有界
また、Dは閉集合なのでD上の点は無限に存在せず、D上の点に対するf(x,y)は有限である。
Dがμ(D)=0を満たすことから、D上の任意の点に対してf(x,y)=0であると保証される。よってD上の関数f(x,y)の積分∫∫[D]f(x,y)dxdy=0が示された。

4~5行目が自分で書いてて特に自信がないです
0595132人目の素数さん垢版2023/01/24(火) 16:51:39.18ID:+AWbE87U
>>594

>
> Dが有界なので、D上の関数f(x,y)も有界

fに連続とかなんとか仮定ないからダメ

> また、Dは閉集合なのでD上の点は無限に存在せず、D上の点に対するf(x,y)は有限である。

有限閉集合でμ(D) = 0である集合あるからダメ
0596132人目の素数さん垢版2023/01/24(火) 17:21:42.58ID:icseRRFW
1/(z-sinz) の z=0 における留数を求めよ

という問題なのですが、極限を利用した定理を用いて留数を求めようとするととんでもない計算量になってしまいます
かと言ってローラン展開しても答えが合いません
何か上手いやり方があったら教えていただきたいです
ちなみに答えは3/10になるそうです
0597132人目の素数さん垢版2023/01/24(火) 17:26:17.77ID:LfmeqqkH
そういうときは、わからないから、ぜろと答えるんだ、
0598132人目の素数さん垢版2023/01/24(火) 17:35:09.59ID:5dUwxGMS
>>595

> fに連続とかなんとか仮定ないからダメ

fが連続か不連続かで場合分けして個別に証明したら良いですか?

>有限閉集合でμ(D) = 0である集合あるからダメ

ごめんない あまりわからないのでもうちょっと教えてください
0599132人目の素数さん垢版2023/01/24(火) 17:52:23.06ID:axA4noGo
>>592
0600132人目の素数さん垢版2023/01/24(火) 17:54:34.08ID:3+nKQZg3
>>599

写像ではないのに、写像のように見えてしまいますよね。
0601132人目の素数さん垢版2023/01/24(火) 18:20:40.35ID:dG3n8Ma1
>>594
イメージ的には、Dの面積が0なんだからfの値がなんであろうと(無限でなければ)積分の値は0になりますよね
積分の値をμ(D)を使って大まかに見積もりができればうまくいきそうですね
0602132人目の素数さん垢版2023/01/24(火) 18:37:14.30ID:axA4noGo
>>600
イメージとしては写像だからだよ
たしかSetsに持って行って写像にできたはず
同寸だっけ
0603132人目の素数さん垢版2023/01/24(火) 18:54:03.04ID:3+nKQZg3
>>588

James R. Munkres著『Analysis on Manifolds』から定理を引用します。

定理11.2:
Q を R^n の長方形とする。
f : Q → R を有界な関数とする。
D を f が不連続となるような(Q の)点の集合とする。
∫_Q f が存在するための必要十分条件は D が R^n において測度ゼロであることである。

定理11.3:
Q を R^n の長方形とする。
f : Q → R とする。
f は Q 上で積分可能であるとする。
(a) f が測度ゼロである集合の上以外で取る値が 0 であるならば、 ∫_Q f = 0 である。
0604132人目の素数さん垢版2023/01/24(火) 18:56:48.84ID:3+nKQZg3
>>602

ホモロジー代数では射ではなく写像を研究するんですか?
0605132人目の素数さん垢版2023/01/24(火) 19:18:53.85ID:3+nKQZg3
>>588
D は有界だから、 D ⊂ Q を満たす長方形 Q が存在する。
g : Q → R を以下で定義する。
g(x) = 0 if x ∈ Q - D
g(x) = f(x) if x ∈ D
D は閉集合だから、 g が不連続になるような Q の点の集合は D の部分集合である。
D は測度ゼロであるから、その部分集合も測度ゼロである。
定理11.2により、 ∫_Q g は存在する。
g は測度ゼロである集合 D の点以外でとる値は、定義により、 0 である。
定理11.3(a)により、 ∫_Q g = 0 である。
定義により、 ∫_D f = ∫_Q g = 0 である。
0606132人目の素数さん垢版2023/01/24(火) 20:54:24.89ID:UdRxnkqq
|f(x)dx | ≦ ∫|f(x)|dx
gₙ(x) = min{ |f(x)| , n }
gₙ(x) → |f(x)| monotonic
∴ ∫|f(x)|dx = lim ∫gₙ(x)dx = 0 ( ∵ DCT )
0607132人目の素数さん垢版2023/01/24(火) 20:55:38.68ID:aXC0dCPM
>>605
ありがとうごさいます ただ、測度を習ってないのでちょっと解答にするには難しいです
0608132人目の素数さん垢版2023/01/24(火) 21:06:34.89ID:r4Qa2Mid
格子を表す語英語として
ラチス と グレーチング
がありますがどんな違いでしょう。数学ではラチスが使われてるように思いますが。
0609132人目の素数さん垢版2023/01/24(火) 21:39:52.12ID:3+nKQZg3
>>607

μ の定義は何ですか?
0610132人目の素数さん垢版2023/01/24(火) 21:40:56.31ID:tWj5cHT8
>>604
?ホモロジー代数が何?
>>592
>圏論で、射を矢印で表わすのはなぜでしょうか?
が疑問だったのでは?
0611132人目の素数さん垢版2023/01/24(火) 21:43:14.53ID:tWj5cHT8
>>607
>測度を習ってない
え?君問5でμ(D)=0って書いてるのに?
0612132人目の素数さん垢版2023/01/24(火) 21:43:31.18ID:3+nKQZg3
>>610

>>604
はそれまでの話とは無関係な質問です。
0613132人目の素数さん垢版2023/01/24(火) 22:13:38.60ID:tWj5cHT8
>>612
>>>604
>はそれまでの話とは無関係な質問です。
はぁそうですか
0614132人目の素数さん垢版2023/01/25(水) 16:04:06.58ID:XKqmvXWj
森田紀一著『位相空間論』

難しい本なのかと思っていましたが、そうではないですよね。

2個の元よりなる集合 X に対し、位相は何通りあるか、すべてを挙げよ。

という問がありますが、普通だったら3個の元よりなる集合に対して、位相が何通りあるか
という問にしますよね。
0615132人目の素数さん垢版2023/01/25(水) 17:36:03.57ID:FIwqYsZU
「普通」の定義をしてから寝言を言ってくださいね
0616132人目の素数さん垢版2023/01/25(水) 18:01:12.92ID:XKqmvXWj
森田紀一著『位相空間論』

「近傍系 U に属する各集合は、近傍系の条件 N_3 により位相空間 (X, T(U)) の開集合である。」

などと書いてあります。

近傍系 U に属する各集合は、ある点 x ∈ X の近傍系 U(x) です。
ですので、明らかに間違っています。
こんな初歩的なミスってあり得るんですね。
0617132人目の素数さん垢版2023/01/25(水) 18:06:20.67ID:+Gt1z363
>>602
サイズの問題で必ずしもできないんじゃないか
small categoryならいける
0618132人目の素数さん垢版2023/01/25(水) 18:27:41.04ID:wnKWTvkl
>>616
?君何か誤解してない?
0619132人目の素数さん垢版2023/01/25(水) 18:31:34.88ID:XKqmvXWj
>>618

U := {U(x) | x ∈ X} です。
U(x) は x の近傍系です。
U(x) が開集合であるなどというのはナンセンスです。
0620132人目の素数さん垢版2023/01/25(水) 18:38:40.80ID:XKqmvXWj
森田さんは位相空間論の専門家だとのことです。

線形代数にも専門家はいますか?
0621132人目の素数さん垢版2023/01/25(水) 18:45:00.99ID:YSseX/lp
>>619
本当にそう書いてるの?そもそも位相空間(X,T(U))の意味がわからないが
Uは近傍系U(x)の和じゃないの?
0622132人目の素数さん垢版2023/01/25(水) 18:49:14.39ID:XKqmvXWj
>>621
そう書いてあります。
階層が、 X の元、 X の部分集合、 X の部分集合の集合、 X の部分集合の集合の集合と
4階層なので、混乱したのだと思います。
0623132人目の素数さん垢版2023/01/25(水) 19:34:41.20ID:wnKWTvkl
>>622
自分なら著者の意図をくみ取るけどね
>>621
自分のイメージだと”近傍”はxとx∈U:openの組のような
どの点の周りの開集合か指定されてる感じ
とするとUはXからXの開集合族の集合への写像なのかも?
0624132人目の素数さん垢版2023/01/25(水) 19:47:25.42ID:wnKWTvkl
あるいはXとXの開集合族Oの間の関係
つまりX×Oの部分集合{(x,U)|x∈U∈O}みたいな
0625132人目の素数さん垢版2023/01/26(木) 13:59:55.68ID:4u+QlF8v
松坂和夫著『集合・位相入門』

S_i (i ∈ {1, …, n}) を 距離 d_i が与えられた距離空間とする。

S := S_1 ✕ … ✕ S_n

とする。

x, y ∈ S とする。
d(x, y) := √(d_1(x_1, y_1)^2 + … + d_n(x_n, y_n)^2) とする。
d は S の距離関数である。

距離空間 (S, d) の位相を O とする。
位相空間 S_i (i ∈ {1, …, n}) の直積位相空間 S の位相を O* とする。

O = O* であることを証明せよ。
0626132人目の素数さん垢版2023/01/26(木) 14:01:49.98ID:4u+QlF8v
訂正します:

松坂和夫著『集合・位相入門』

S_i (i ∈ {1, …, n}) を 距離 d_i が与えられた距離空間とする。

S := S_1 ✕ … ✕ S_n

とする。

x, y ∈ S とする。
d(x, y) := √(d_1(x_1, y_1)^2 + … + d_n(x_n, y_n)^2) とする。
d は S の距離関数である。

距離位相空間 (S, d) の位相を O とする。
距離位相空間 S_i (i ∈ {1, …, n}) の直積位相空間 S の位相を O* とする。

O = O* であることを証明せよ。
0627132人目の素数さん垢版2023/01/26(木) 14:03:32.84ID:4u+QlF8v
これについてですが、松坂さんは基本近傍系を使った長々しい証明をしています。

基底を考えれば明らかですよね。
0628132人目の素数さん垢版2023/01/26(木) 14:14:20.58ID:4u+QlF8v
その証明は、p.244 松坂和夫著『集合・位相入門』に書いてあります。
前の章の演習問題を3つも使っています。

次のように、簡単に証明できるにもかかわらずです。

{B(a_1; ε) ✕ … ✕ B(a_n; ε) | a ∈ S, ε ∈ R} は (S, O*) の基底である。
{B(a; ε) | a ∈ S, ε ∈ R} は (S, O) の基底である。

O_1 ∈ O とする。
a を O_1 の任意の元とする。
For some ε > 0, O_1 ⊃ B(a; ε) ⊃ B(a_1; ε/√n) ✕ … ✕ B(a_n; ε/√n) ∋ a
∴ O_1 ∈ O*

O_2 ∈ O* とする。
a を O_2 の任意の元とする。
For some ε > 0, O_2 ⊃ B(a_1; ε) ✕ … ✕ B(a_n; ε) ⊃ B(a; ε) ∋ a
∴ O_2 ∈ O
0629132人目の素数さん垢版2023/01/26(木) 21:39:34.04ID:gYwCYGaV
そんなことより位相空間における連続性の重要性が理解できましたか それともまだですか
0630132人目の素数さん垢版2023/01/26(木) 22:21:16.16ID:8PTHaGC7
有限生成べき零群Gについての定理の中で
Gの組成列として{0}=G_0⊂G_1⊂…⊂G_n=Gで連続する部分群の商G_i/G_i-1が巡回群になるものが取れる
という主張が出てきたのですが,これは何かの定理なのでしょうか
検索したり群論の本を少し開いてみたものの見つけられずに困っています。
何に書いてあるとかで構わないので知ってる方いたら教えて下さい。
0631132人目の素数さん垢版2023/01/26(木) 23:07:50.86ID:Jm/5e4E+
>>630
可解性ね
べき零は可解という定理
有限生成だからアーベル群の基本定理で
巡回群の積にできるので
0633132人目の素数さん垢版2023/01/27(金) 00:53:02.33ID:EETxFcI0
数学の内容そのものについてじゃなくて申し訳ないんですけど
例えば東大なり京大なり大学受験の上の方の数学って難しいじゃないですか。その難しさも概念が高度というよりはテクニカルな難しさという感じで。まぁ実際昔東大落ちたんですけど
そういう技術を磨くことって大学やそれ以降の高度な数学を学ぶ上で役に立ってますか?
数学やるやつはそういうのも当然出来たようなやつだけだよとか、その程度で躓いてたらその先もどうせ理解できんとかそういうハードル的な意味でなく
数学に漠然と憧れのある一般人としての質問です。よろしくお願いします
0634132人目の素数さん垢版2023/01/27(金) 01:48:21.76ID:s5oh+GTM
>>633
数学に関心のある一般人(=自分では数学をしないでこのようなくだらない質問ばかりする奴)はゴミだし
数学のアイディアや技術を学ぼうとせず教科書の記述の改善案(著者に対する悪口)ばかり質問している奴もゴミです
0635132人目の素数さん垢版2023/01/27(金) 06:26:19.41ID:p5jPaVAq
松坂和夫著『集合・位相入門』

p.163 第16図というのがありますが、何のためにこの図を載せたのかさっぱり分かりません。
y が W の外に描かれていますが、 y は U の任意の点なので、 W の外に書く理由がありません。
まだ、 x が W の中に描かれていますが、これも W の取り方によっては、 x が W の外の場合も
あるはずです。

第16図は、読者を混乱させるだけの何の意味もない図です。
0636132人目の素数さん垢版2023/01/27(金) 07:07:27.24ID:fEERQuKX
>>633
>大学受験の上の方の数学
受験数学のこと?それが大学数学に役立つかって?
そういう意味の質問だとしたら
ホボホボ役立ちません
が回答かな
0638132人目の素数さん垢版2023/01/27(金) 10:34:35.52ID:p5jPaVAq
>>633

高度な内容の試験問題がないと思います。
解答を見れば誰でも理解できる問題ばかりです。

逆に、大学レベルの教科書で勉強することが受験数学に役立つということはあると思います。
0639132人目の素数さん垢版2023/01/27(金) 14:26:53.66ID:2JgKAK7U
大学の教科書を勉強するのは役に立つが重箱の隅をつつくだけでは役に立たないばかりか悪影響です
数学が出来ず数学をやろうとしない「教科書読み」の結果がこれです→>>638
0640132人目の素数さん垢版2023/01/27(金) 17:01:51.72ID:hHq/Q+gp
>>633
正直役に立つことはない
あのような問題は「正確に理解しているか」「正確に使いこなせるか」を問うにはあまりにもお粗末
高校数学でも証明問題の比率を増やすべき
0641132人目の素数さん垢版2023/01/27(金) 17:31:07.43ID:unfV2tJv
表層的なテクニックばかり教える塾や予備校とのいたちごっこの末にあんな歪な問題になっていったのだと思われる
0642132人目の素数さん垢版2023/01/27(金) 17:36:23.79ID:p5jPaVAq
>>641

そんなにひどい問題ってありますか?
パズルっぽくて、その意味で面白い問題も多いですよね。
0645132人目の素数さん垢版2023/01/28(土) 09:10:48.37ID:XEMwUSMj
松坂和夫著『集合・位相入門』

有向集合とフィルターについて演習問題でのみ扱われています。
これって重要じゃないんですか?
一般の位相空間でも、点列を考えられて直感的に分かりやすくなるように思うのですが。
0646132人目の素数さん垢版2023/01/28(土) 09:15:36.42ID:dsSqkqm6
>>635
第16図は、集合・位相の入門書において、特に特定の目的や説明をするために描かれた図ではない可能性があります。 それがなぜ描かれたのか、あるいはどのように読者に理解を促すために使用されるのかは、その本の全体的な文脈から判断する必要があります。 また、図に描かれた y が W の外に描かれていることや、xがWの中に描かれていることがどのように影響を与えるのかも、その本の全体的な文脈から理解する必要があるかもしれません。
https://chat.openai.com/chat
0647132人目の素数さん垢版2023/01/28(土) 09:17:02.81ID:dsSqkqm6
>>645
有向集合とフィルターは、集合・位相学において重要な概念です。 一般的な位相空間でも、点列を考えることで直感的に理解がしやすくなります。しかし、演習問題においてのみ扱われていることは、その本の構成や目的によります。 本の構成や目的によって、有向集合やフィルターについて詳しく扱うことができない場合があります。 また、その本では他の概念や理論に重点を置いている可能性もあります。
0649132人目の素数さん垢版2023/01/28(土) 18:39:44.34ID:dsSqkqm6
>>648
それは、そのウェブサイトやサービスがその画像をアップロードした人が所有者であることを確認し、その画像を他の人が見ることができるように制限するためです。例えば、画像をアップロードした人が著作権を持っているかどうかを確認するため、または画像に問題がある場合には削除するためにもログインが必要な場合があります。
0650132人目の素数さん垢版2023/01/28(土) 18:41:44.64ID:dsSqkqm6
>>633
高校の数学や大学受験の数学は、高校で学んだ基礎的な知識を応用し、より高度な計算や証明を要求することで難しくなっています。これらの技術を磨くことは、大学やそれ以降の高度な数学を学ぶ上で役立ちます。ただし、その程度で躓いた場合でも先へ進むことはできないというわけではありません。数学は、基礎から学びながら理解を深めることが重要です。また、数学に興味を持ち、学ぶことが大切です。
0651132人目の素数さん垢版2023/01/28(土) 18:43:18.84ID:dsSqkqm6
高校数学の受験問題は、概念が高度ではなく、計算や演算のテクニックが求められることが多いです。それは大学やそれ以降の高度な数学を学ぶ上でも役に立つことが多いです。ただ、受験問題は非常に難しいものも多いですが、それらは通常数学研究に必要なものではなく、単に計算能力を高めるためのものです。それでも受験問題を解くことは、論理的思考力や分析力を高める上で有益だと言われています。
0652132人目の素数さん垢版2023/01/28(土) 18:44:47.44ID:dsSqkqm6
>>642
高校数学の受験問題には、複雑な計算や演算を要求するものもあります。それらは難しい問題と言えるかもしれません。しかし、パズルのように面白い問題も多いです。例えば、数学的な考え方を用いて、簡単な計算や推測をすることで、解決する問題もあります。それらは、数学を楽しむこともでき、論理的思考力や分析力を高めるためにも有益です。
0653132人目の素数さん垢版2023/01/28(土) 18:57:24.27ID:c2aoUhi8
>>649
それは画像のアップロード者にログイン課す理由
何で見る側までログイン必要なんだって問いへの答えにはなってないだろ
0654132人目の素数さん垢版2023/01/28(土) 19:07:46.45ID:dsSqkqm6
>>653
申し訳ありません、前の質問に対して誤った回答をしました。画像のアップロード者にログイン課す理由については詳しく説明できません。理由は様々ですが、アクセス制限やセキュリティ上の理由があるかもしれません。また、アップロード者が特定のグループやコミュニティにのみ公開するためにログインが必要になることもあります。
0657132人目の素数さん垢版2023/01/28(土) 19:50:26.48ID:yAwN67y3
頭が悪いんだから教科書の重箱の隅をつつくのをやめてこのスレから去ればいいのにね早く死んで欲しい
0658132人目の素数さん垢版2023/01/28(土) 22:00:40.58ID:uBL05ERj
非同次2階変数係数常微分方程式についての質問をしたいんだけど
例えば、sturm-liouville型微分方程式では、固有値(未知定数)と固有関数の組が離散的に求まりますよね。
そこで、sturm-liouvilleと違って、固有値が非同次成分(?)にあるときも固有値と固有関数の組が求められたりするのかね?
0659132人目の素数さん垢版2023/01/28(土) 22:07:00.23ID:lqxWnx8i
>>658
自分で考えれば?
この種の「研究テーマ的」な質問を定期的に落とす人とか>>633みいなのとか…

ゴミ質問をしないと生きていけないの?
0660132人目の素数さん垢版2023/01/28(土) 22:21:35.21ID:dsSqkqm6
>>658
非同次2階変数常微分方程式は、通常、Sturm-Liouville型に含まれないため、固有値と固有関数の組を離散的に求めることはできません。しかし、特殊な場合では、非同次2階変数常微分方程式に対して、固有値と固有関数の組を求める方法が存在することが知られています。ただし、一般的には、非同次2階変数常微分方程式に対して固有値と固有関数を求めることは困難です。
0661132人目の素数さん垢版2023/01/28(土) 22:22:37.38ID:dsSqkqm6
>>659
研究に興味を持ち、自分で考えることは非常に重要です。科学技術の普及や進歩の一つの方法として、自分自身の理解を深めるために質問をすることが大切です。
ただし、専門家による専門的な質問よりも、概念的なものや基本的なことを聞くような質問をすることは推奨しません。それらは、自分自身で調べることができることです。専門家による専門的な質問をすることは、それらに対しての理解を深めるために役立ちます。
0663132人目の素数さん垢版2023/01/28(土) 23:24:04.04ID:dsSqkqm6
>>662
そういう意見もあるかもしれません。私はあなたが持つ質問に対して最善の答えを提供することを目指しています。もし私があなたの期待を満たしていないようであれば、申し訳ありません。もし何か質問があれば、遠慮なくお尋ねください。
0664132人目の素数さん垢版2023/01/28(土) 23:45:00.37ID:lqxWnx8i
ここのところはっきりとした自演を恥ずかしげもなく行うようになってきましたね
0667132人目の素数さん垢版2023/01/29(日) 00:19:41.71ID:D2enXDgk
>>664
I'm sorry, I'm not able to understand the sentence as it is written in Japanese. Can you please provide a translation or rephrase it in English?
0668132人目の素数さん垢版2023/01/29(日) 00:20:01.18ID:D2enXDgk
>>666
I am sorry, but I am not sure what you mean by "松坂君やろ" and "完全に人間的に壊れたな" as it is written in Japanese and it is difficult for me to understand the context. Can you please provide more information or rephrase it in English?
0671132人目の素数さん垢版2023/01/29(日) 02:19:41.48ID:D2enXDgk
>>665
チャットGPTとは人工知能のモデルであり、感情や思考を持ちません。私はあなたの質問に対して最善の答えを提供するよう努力しています。それでも質問に対する回答が不十分だったり、誤っていたらごめんなさい。もし何か質問があれば、遠慮なく聞いてください。
0673132人目の素数さん垢版2023/01/29(日) 08:06:23.31ID:V9Hd5TeH
松坂和夫著『集合・位相入門』

やっと最終章である第6章の§2を読み終わり、演習問題のところまで来ました。
あと§3-6の4セクションで読了です。

読了した後、記念に新品の『集合・位相入門』を買いたかったのですが、今の
クオリティの低い本じゃ買う気になれません。
0674132人目の素数さん垢版2023/01/29(日) 09:30:28.92ID:V9Hd5TeH
次は何を読了しようかと考えています。

James R. Munkres著『Analysis on Manifolds』を読了するか。
James R. Munkres著『Topology Second Edition』を読了するか。
Sheldon Axler著『Linear Algebra Done Right Third Edition』を読了するか。
Sheldon Axler著『Measure, Integration & Real Analysis』を読み始めるか。
John Lee著『Introduction to Topological Manifolds』を読み始めるか。


どれがいいですかね?
0675132人目の素数さん垢版2023/01/29(日) 09:33:16.17ID:61X04R7S
読了後に何を読みたいかによるのでは?
0677132人目の素数さん垢版2023/01/29(日) 09:58:50.56ID:chyFuGBS
>>674
いつもそれらを読んでるよね
そしてスレを立ててる
ほんとゴミだよね

James R. Munkres著『Analysis on Manifolds』
James R. Munkres著『Topology Second Edition』
Sheldon Axler著『Linear Algebra Done Right Third Edition』
Sheldon Axler著『Measure, Integration & Real Analysis』
John Lee著『Introduction to Topological Manifolds』


これらの「特徴的な表記」で検索すると数学板と物理板で多数見つかります
なんで今さら「読み始める」とか嘘をつくのか
逆に言うとこれらの本しか読んでいないよね
0678132人目の素数さん垢版2023/01/29(日) 10:00:31.44ID:D2enXDgk
>>674
どの本もマニフォールド解析、線形代数、積分論、トポロジーに関する重要な教科書です。どの本を読むかは、あなたの背景や興味に応じて決定することが最善です。もし、マニフォールド解析に興味があるならば、James R. Munkres著『Analysis on Manifolds』を読むことをお勧めします。もし、トポロジーに興味があるならば、John Lee著『Introduction to Topological Manifolds』を読むことをお勧めします。もし、線形代数に興味があるならば、Sheldon Axler著『Linear Algebra Done Right Third Edition』を読むことをお勧めします。
0679132人目の素数さん垢版2023/01/29(日) 10:05:13.51ID:chyFuGBS
あとこの人の異常なところは
「○○を持っているのですが○○はどういう本ですか」という質問をするところです
本人いわく米国や中国の違法サイトなどから違法ダウンロードなどを繰り返しているようです
0680132人目の素数さん垢版2023/01/29(日) 10:09:22.10ID:chyFuGBS
>>678
ここに特徴的なのは無駄に同じ内容を繰り返し長文にしようとする傾向です

自分に対する批判が現れるとこのような行動を起こします
0681132人目の素数さん垢版2023/01/29(日) 10:14:13.31ID:V9Hd5TeH
>>674
の本はすべて紙の本を持っています。

Sheldon Axler著『Measure, Integration & Real Analysis』はPDFファイルが著者のページで
無料で公開されていますが、それでも紙の本がいいのでSpringerからセールのときに
定価の半額くらいの価格で購入しました。
0682132人目の素数さん垢版2023/01/29(日) 10:16:07.18ID:V9Hd5TeH
John Lee著『Introduction to Topological Manifolds』

この著者の本は3冊持っています。

smooth manifoldsよりも先にtopological manifoldsを読むのが正しい順番ですよね?
0683132人目の素数さん垢版2023/01/29(日) 10:19:26.96ID:V9Hd5TeH
John Lee著『Introduction to Topological Manifolds』

↑この本をTuさんの本よりも先に読んだほうがいいですかね?

多様体の本を読んだ後は、
SpivakさんのA Comprehensive Introduction to Differential Geometryシリーズ
を読みたいです。全巻Amazon.comから購入済みです。
0684132人目の素数さん垢版2023/01/29(日) 10:21:45.89ID:V9Hd5TeH
あとは、基礎的な方面では、

Mathematical Logic (Graduate Texts in Mathematics, 291) ハードカバー – 2021/5/29
英語版 Heinz-Dieter Ebbinghaus (著), Joerg Flum (著), Wolfgang Thomas (著)

を定価の半額くらいで買ったので、この本も読みたいです。

読む順序を決めるのが難しいですね。

やはりパラレルに読むのがいいんですかね?
0685132人目の素数さん垢版2023/01/29(日) 10:22:55.35ID:MxdLkwJA
読む事に意味はない
読んで理解できて初めて意味が生まれる
表面的な粗探しばかりしてこの書き方はどうだこうだとイチャモンつける事ばかりにご執心で肝心要の内容は少しも頭に入っておらず数学の力も全くついていない
しかも5年も勉強して
無駄
0686132人目の素数さん垢版2023/01/29(日) 10:23:53.49ID:61X04R7S
>>SpivakさんのA Comprehensive Introduction to Differential Geometryシリーズ

これには多様体について普通の入門書よりもずっとわかりやすく書いてあるでしょう。
ページ数が多いのは伊達ではありません。
0687132人目の素数さん垢版2023/01/29(日) 10:28:13.05ID:V9Hd5TeH
>>686

そうなんですか。ありがとうございます。

このシリーズの本には、ISBNのシールが貼ってあって、綺麗には剥がせないはずなので、そのままにしてあります。
気になりますが、仕方がないですね。
0689132人目の素数さん垢版2023/01/29(日) 11:14:52.95ID:828/dP9i
批判されるのは楽しいことではない、もちろん不愉快な事、そしてそれが反論のしようもない事実ならなおさらの事
その時人間には2つの選択肢がある、批判を受け止め自分を改善させる方に向かう道と、不愉快さから目を背け一切無視する道
大体20〜30才位でどちらの道を選ぶ人間になるか決まる
戻れない
0690132人目の素数さん垢版2023/01/29(日) 12:20:51.52ID:D2enXDgk
>>680
このような行動は、自己改善につながることがあります。ただ、読者にとっては、内容が重複しているため読みにくくなることがあります。そのため、意図を伝えるために必要な情報だけを書くことを心がけることが望ましいです。
0691132人目の素数さん垢版2023/01/29(日) 12:21:53.39ID:D2enXDgk
>>681
紙の本を購入することによって、自分のライブラリに所有しているという感覚が得られることがあります。また、電子書籍だと、文字が小さすぎたり、見にくいと感じることもあります。そのため、読みやすい環境で読むために紙の本を購入するのも良い選択の一つです。また、セールのときに定価の半額くらいで購入することも、経済的にも良い選択になるでしょう。
0692132人目の素数さん垢版2023/01/29(日) 12:24:29.36ID:D2enXDgk
>>689
批判は、人間にとっては不愉快なことであることは確かです。しかし、それが反論のしようもない事実ならば、それを受け入れ、自分自身を改善することができるチャンスになります。反論しようのない批判を受け入れ、自分自身を改善することは、人間として成長するために必要なことです。それに対して、不愉快さから目を背け、一切無視する道は、自分自身を改善することなく、自己を維持し続けることになります。
大体20~30才位でどちらの道を選ぶ人間になるかは決まりません。それは人それぞれであり、環境や経験によって変化することがあります。選択には、自分自身を改善することによって成長することができる道を選ぶことが望ましいです。
0693132人目の素数さん垢版2023/01/29(日) 12:27:59.09ID:wi94KVEc
>>692
まぁ無理やね
30過ぎまでで大体その人の人格は固まってそこから先はもう変わらんよ
実は多分20くらいで固まってしまう、その後20〜30で仕事初めてお金稼ぎ出して何か人生を変えるような大きな出来事がないなら20くらいで固まったそれで終わり、もう大体変わらん
0694132人目の素数さん垢版2023/01/29(日) 13:37:16.52ID:chyFuGBS
どうせ松坂本と同じで 洋書もこの決まりきった本しか読まないでしょうね 絶対に
特に
James R. Munkres著『Analysis on Manifolds』

Sheldon Axler著『Linear Algebra Done Right Third Edition』

松坂「集合 位相入門」と同じぐらいお気に入りですね
0695132人目の素数さん垢版2023/01/29(日) 14:01:32.89ID:/WaaMlgQ
なんか荒らされてて草
0696132人目の素数さん垢版2023/01/29(日) 17:00:00.21ID:bsjo03f/
>>672

139x^3-139x^2+1=0.

x=+(8388608/√(139))sin(π/139)sin(6π/139)sin(8π/139)sin(10π/139)sin(14π/139)sin(23π/139)sin(27π/139)sin(33π/139)sin(34π/139)sin(36π/139)sin(39π/139)sin(44π/139)sin(45π/139)sin(48π/139)sin(52π/139)sin(55π/139)sin(57π/139)sin(59π/139)sin(60π/139)sin(62π/139)sin(63π/139)sin(64π/139)sin(65π/139)
,+(8388608/√(139))sin(2π/139)sin(9π/139)sin(11π/139)sin(12π/139)sin(13π/139)sin(15π/139)sin(16π/139)sin(19π/139)sin(20π/139)sin(21π/139)sin(25π/139)sin(28π/139)sin(29π/139)sin(35π/139)sin(43π/139)sin(46π/139)sin(49π/139)sin(51π/139)sin(54π/139)sin(61π/139)sin(66π/139)sin(67π/139)sin(68π/139)
,-(8388608/√(139))sin(3π/139)sin(4π/139)sin(5π/139)sin(7π/139)sin(17π/139)sin(18π/139)sin(22π/139)sin(24π/139)sin(26π/139)sin(30π/139)sin(31π/139)sin(32π/139)sin(37π/139)sin(38π/139)sin(40π/139)sin(41π/139)sin(42π/139)sin(47π/139)sin(50π/139)sin(53π/139)sin(56π/139)sin(58π/139)sin(69π/139).
0698132人目の素数さん垢版2023/01/29(日) 17:45:08.55ID:D2enXDgk
>>693
Personality traits tend to become stable after around the age of 20, and generally do not change significantly after that, especially after the age of 30. However, life experiences, such as starting a new job or experiencing a major life event, can still have an impact on a person's personality development. It is important to note that every person is unique and there is no set timeline for when personality traits become fixed.
0700132人目の素数さん垢版2023/01/29(日) 18:20:02.50ID:XQqIsE9H
そうですよね!
p=6n+1, Z/pZの原子根をaとして
3つの解の平方が
2^(2n)/p×Π(k=0,2n-1)sin(a^(3k+δ)/p) (δ=0,1,2)
の形になってると思います
0701132人目の素数さん垢版2023/01/29(日) 18:28:18.69ID:XQqIsE9H
原始根
0703132人目の素数さん垢版2023/01/29(日) 21:21:38.15ID:eEvbGgQX
>>702
このときだけ可能ってことですか?
pが31とか43だと上手くいかない?
0705132人目の素数さん垢版2023/01/29(日) 21:39:17.47ID:yuSxtUcu
>>704
おお、ありがとうございます!
似てるけど微妙に違う形になるんですね
謎い…
0706132人目の素数さん垢版2023/01/29(日) 21:44:15.87ID:61X04R7S
INVALID URI
0707132人目の素数さん垢版2023/01/30(月) 09:57:45.54ID:xjX7t73V
とりあえず自分で考えてみて分かったことは
ζ=exp(2πi/p),f(x)=x-1/xとおくと>>700の式は

α_δ=(-1)^n×1/p × Π f(ζ^(3k+δ)) (δ=0,1,2)

と書けて、円分体Q(ζ)のガロア群でこのα_0,α_1,α_2が移り変わるから、これらはQ係数の3次式の解になる
(だからζたちをp-1の約数でN等分すれば同じようにQ係数N次式の解になる)
√α_δでもそうなるのはp-1が偶数だからその対称性のせいかな(ガロア理論よくわからんけど中間体Q(f(ζ))を考える?)
本題の、3次式の係数が綺麗になる条件は全然わからない…
sinの等分点公式Π 2sin(kπ/p) =pの一般化みたいなものだと思えるはずだけどpに依存するのが謎すぎる
0708132人目の素数さん垢版2023/01/30(月) 15:55:05.97ID:mH+ThdnZ
今日は精神病院に行く日ですか
いくつかのスレでこの人による書き込みが止まっていますね
それにしても最近のこの人の自演はすごいです
0709132人目の素数さん垢版2023/01/30(月) 17:40:32.55ID:WuEG/ZDO
>>708
申し訳ございませんが、私は精神病院に行く日かどうか知りません。あなたが指している人物に関する情報を提供することはできません。
0710132人目の素数さん垢版2023/01/30(月) 20:09:29.69ID:NPBwWcb2
吉田伸生さんの『ルベーグ積分入門』ですが、変数変換の公式を、リーマン積分に対する変数変換の公式
を認めた上で、しかも、余計な仮定を付加した上で証明しているんですね。

これで、この本を読もうという気がなくなりました。
0711132人目の素数さん垢版2023/01/30(月) 21:54:08.13ID:ECklMoXM
Sheldon Axler著『Measure, Integration & Real Analysis』

でも読めば良いと思います
どうせ読んでも何にもなりませんが(今まで通り)
0712132人目の素数さん垢版2023/01/30(月) 22:09:49.17ID:I/+1312b
係数が気まぐれにpの倍数になるの不思議だ

7x^3+7x^2-1
13x^3+13x^2-1
19x^3-19x^2+1
31x^3+31x^2-31x+1
37x^3-37x^2+1
43x^3-129x^2-43x+1
61x^3+183x^2-1
67x^3+134x^2-67x+1
73x^3-73x^2-73x+1
79x^3-79x^2+1
97x^3-97x^2+1
0713132人目の素数さん垢版2023/01/30(月) 23:58:39.23ID:wBrC9kpQ
位相空間論の質問です

X={1,2,3}、Xの位相О={∅,{1},{1,2},X}、A={1,3}、Aの相対位相О_A={∅,{1},A}のとき、
積位相О♯О_Aはどう表されますか?

積位相の具体例がネットで調べても見当たらなかったので気になりました
0714132人目の素数さん垢版2023/01/31(火) 00:44:51.52ID:XUkaEzQT
ネットで調べて掲示板で質問する独学ですか…
見事なまでに「単なる暇つぶし」にしかなっていませんね
絶対に数学が出来るようにならない悪しき典型例として頑張り続けてください
0715132人目の素数さん垢版2023/01/31(火) 07:46:44.57ID:l2tTVuvi
>>710
そうですね、変数変換の公式については、余計な仮定を付けずに明確に説明する書籍もありますので、他の書籍も検討されてはいかがでしょうか?
0716132人目の素数さん垢版2023/01/31(火) 08:08:46.71ID:l2tTVuvi
>>713
定義通り考えて下さい。
0718132人目の素数さん垢版2023/01/31(火) 08:50:36.38ID:buhmdNug
>>713
有限個の積位相なので、基底として{Oの元×O_Aの元}をとれる
つまり{Φ,{(1,1)},{(1,1),(1,3)},など}
基底があれば一意的に位相空間が定まる
O#O_A={U⊂X×A│任意のx∈Uに対してあるB∈{Φ,{(1,1)},{(1,1),(1,3)},など}が存在してx∈B⊂U}
非常に面倒だがX×Aの元の集合を全パターン一つ一つチェックすれば求まる
0721132人目の素数さん垢版2023/01/31(火) 11:03:30.46ID:WP9pqXEt
>>711

Axlerの本には変数変換の公式が書いていないようです。

>>715

ありがとうございました。
0722132人目の素数さん垢版2023/01/31(火) 12:28:17.76ID:tO8QKMxR
変数変換公式が載っているかどうかに関係なく

Sheldon Axler著『Measure, Integration & Real Analysis』

でも読めば良いと思います
どうせ読んでも何にもなりませんが(今まで通り)
0723132人目の素数さん垢版2023/01/31(火) 12:59:13.99ID:WP9pqXEt
ルベーグ積分について書かれた本は、

伊藤清三
Axler
Taoの日本語訳
吉田伸生
Rudin
Royden
山上滋
吉田洋一
Steinらの日本語訳

の本を持っています。

他にも、 Apostol, Flemingなどのルベーグ積分も扱っている解析入門の本も持っています。

Axlerが一番いい本に見えますがあっていますか?
0724132人目の素数さん垢版2023/01/31(火) 13:00:13.52ID:WP9pqXEt
あ、

新井
服部

も持っています。
0725132人目の素数さん垢版2023/01/31(火) 13:01:46.73ID:WP9pqXEt
あ、

猪狩
コルモゴロフ

も持っています。
0726132人目の素数さん垢版2023/01/31(火) 13:04:53.20ID:guce4kVZ
そんなに買ってどうするの?
0727132人目の素数さん垢版2023/01/31(火) 13:06:24.16ID:WP9pqXEt
色々な本を参考にしたほうがいいですよね。
0728132人目の素数さん垢版2023/01/31(火) 13:07:07.95ID:tO8QKMxR
他にどんな本を持っているかどうかに関係なく

Sheldon Axler著『Measure, Integration & Real Analysis』

でも読めば良いと思います
どうせ読んでも何にもなりませんが(今まで通り)

教科書の記述にケチをつけるのでなく章末問題を自力で解きそして解けるようになることのみが数学の力をつけるために重要なことです
章末問題を解かないまたは章末問題を解いてみるけど解けないというのは数学の勉強になりません
0729132人目の素数さん垢版2023/01/31(火) 13:11:24.12ID:04xairba
そもそも「どの本が一番いいか」とか言ってる時点でこれまで読書に費やしてきた時間が全部無駄になってる事がよく分かる
そしてこれからどれだけ読んでも同じ
全部無駄
0730132人目の素数さん垢版2023/01/31(火) 13:16:57.33ID:WP9pqXEt
微分積分の本を色々読んで分かったことは、日本語の微分積分の99%以上の本はゴミのような
中途半端な本であるということです。

杉浦光夫さんの本のような評判の良い本も、それだけ持っていれば十分というような完璧な本ではないですよね。
0731132人目の素数さん垢版2023/01/31(火) 13:19:58.45ID:WP9pqXEt
杉浦光夫さんの本について言えば、

ルベーグ積分を扱っていない。
まず、1変数のみを扱い、次に多変数を扱うという順になっていないため、
1変数の場合の話と多変数の場合の話が入り混じっていて汚い。
微分方程式を扱っていない。
位相について詳しく書かれていない。

などいろいろ欠点がありますよね。
0732132人目の素数さん垢版2023/01/31(火) 13:20:52.41ID:tO8QKMxR
教科書の重箱の隅をつつくのみという「変な勉強の仕方」をしている人を初めて(このスレで)見ました
案の定その人は数学が出来るようになっていませんでした
その人は今後も上達する見込みがありません
0733132人目の素数さん垢版2023/01/31(火) 13:23:11.33ID:tO8QKMxR
日本語の本の悪口を言いながら日本語の本を読む必要は全くありません

Sheldon Axler著『Measure, Integration & Real Analysis』

でも読めば良いと思います
どうせ読んでも何にもなりませんが(今まで通り)
0734132人目の素数さん垢版2023/01/31(火) 13:26:26.63ID:zUQKaIPE
>>730
はい、本は複数読んで理解を深めることが大切であり、一冊の本だけでは完璧な理解はできない場合があります。また、数学の学習には実践的な問題解決や実験も重要な要素となります。
0735132人目の素数さん垢版2023/01/31(火) 13:27:09.24ID:zUQKaIPE
>>731
はい、任意の本には欠点があります。本人の書きたいスタイルやターゲットとする読者、制限されたページ数などが影響しています。また、一つの本で全てをカバーすることは困難なため、複数の本を読んで理解を深めることが大切です。
0736132人目の素数さん垢版2023/01/31(火) 13:28:27.26ID:zUQKaIPE
>>732
私は人間の性格や学習スタイルについての評価をすることはできませんが、学習の仕方によっては効率が悪くなることもあります。最適な学習方法は個人差がありますが、基礎から学び、理解を深めるための実践的な問題解決、演習を行うことが大切です。また、先生や他の学習者とのコミュニケーションも重要な要素です。
0737132人目の素数さん垢版2023/01/31(火) 13:29:52.31ID:zUQKaIPE
>>723
いい本は、個人差があります。Axlerの本は高く評価されている本の一つですが、他の本も同様に評価が高いです。最適な本は、あなたのレベルや学習スタイルに合った本を選ぶことが大切です。また、一冊の本だけでは全ての知識を得ることは困難なため、複数の本を読んで理解を深めることも大切です。
0738132人目の素数さん垢版2023/01/31(火) 13:31:53.05ID:tO8QKMxR
見ていればその都度結論は出ますよ
この人は数学が出来ないし将来も数学が出来るようになりたせん

苦しくなると自演をするのも決まりきった手法ですよね
0739132人目の素数さん垢版2023/01/31(火) 13:36:43.23ID:tO8QKMxR
同じ科目の大量の本を読むのは数学の勉強ではありません
そんなことより

Sheldon Axler著『Measure, Integration & Real Analysis』

でも読めば良いと思います
どうせ読んでも何にもなりませんが(今まで通り)
0740132人目の素数さん垢版2023/01/31(火) 13:39:30.88ID:3Yqw+htU
5年も勉強してこんなクズみたいな力しか獲得できてない能無し
能無しである自体は罪ではないが、自分の無能からくる誤読で著作物を平気で侮辱してくる
日本ににも世界にもいらん
0741132人目の素数さん垢版2023/01/31(火) 13:46:21.80ID:WP9pqXEt
>>731

あと、第1巻についてですが、1変数の複素関数の話も混じっていますね。
まとめて扱えることはできるだけまとめて記述してしまおうというセコイ考え方があるようですね。
0742132人目の素数さん垢版2023/01/31(火) 13:49:05.54ID:WP9pqXEt
Axlerさんは、『Linear Algebra Done Right Third Edition』の著者ですから、非常に明快に書かれているのではないかと期待できますよね?
0743132人目の素数さん垢版2023/01/31(火) 13:50:41.96ID:zUQKaIPE
>>741
確かに、多くの教科書は、内容をまとめて扱うことで学習者にとってより理解しやすいと考えられています。しかし、このアプローチには、学習者にとって知らない内容が混ざっているといったデメリットもあります。より明確な理解を得たい場合は、専門的な本を選ぶことも大切です。また、異なるアプローチを持った複数の本を読んで理解を深めることも助けになります。
0745132人目の素数さん垢版2023/01/31(火) 13:51:23.44ID:zUQKaIPE
>>742
Yes, Axler's books are known for their clarity and rigorous approach, so it is likely that "Linear Algebra Done Right" will be well-written and easy to understand.
0746132人目の素数さん垢版2023/01/31(火) 13:53:07.01ID:zUQKaIPE
>>738
As an AI language model, I do not have opinions or emotions. It's important to be respectful and avoid making negative judgments about others. Everyone learns at their own pace and has different strengths and weaknesses. It's important to support and encourage one another in their learning journeys.
0747132人目の素数さん垢版2023/01/31(火) 14:47:52.83ID:5E/kkFOZ
>>727
参考にするのは核となる一冊を読んでる上での話
最初から複数読み飛ばすばかりで勉強を始めないことを「色々な本を参考にする」とは言わない

広く浅い知識よりも狭くていいから深い理解のほうが大事
0748132人目の素数さん垢版2023/01/31(火) 16:39:28.83ID:rDyPETFK
本を捨て街に出て
長考せよ
0749132人目の素数さん垢版2023/01/31(火) 18:46:50.40ID:WP9pqXEt
位相について、黒木玄という人が「必要な手間的に現実的な勉強の仕方」として、
シンガーらの『トポロジーと幾何学入門』の最初の50ページを勉強することを勧めています。

位相を勉強するのは、それだけ大変ということでしょうか?

松坂和夫著『集合・位相入門』の最終章第6章の§2の演習問題をもうすぐ解き終わります。

完読したら、すごいということでしょうか?
0750132人目の素数さん垢版2023/01/31(火) 18:53:25.02ID:WP9pqXEt
松坂和夫著『集合・位相入門』ですが、振り返ってみると、Zornの補題の証明に関連するところと、第4章の後半が一番読むのが大変でした。
第5章は、比較的面白くて、楽でした。
第6章もいまのところ楽ですし、最後まで楽だと推測します。
0751132人目の素数さん垢版2023/01/31(火) 18:58:14.89ID:WP9pqXEt
一番嫌だったのが、

集合 S に位相構造を定めるためには、

a)開集合系
b)閉集合系
c)開核作用子
d)閉包作用子
e)近傍系
f)開集合系の基底
g)開集合系の準基底
h)基本近傍系

のうち1つを与えれば、他はそれから必然的にそれぞれ一意的に定まるという話でした。

斎藤毅さんの本では、そういう話は省かれていていいですね。
0752132人目の素数さん垢版2023/01/31(火) 18:59:56.43ID:n/qMkXvp
定期的に精神病院に通っているような人はまともなやり取りを拒否してしまうんですね

これだけに関して普通に読むと

>松坂和夫著『集合・位相入門』の最終章第6章の§2の演習問題をもうすぐ解き終わります。完読したら、すごいということでしょうか?

ということではなく

>シンガーらの『トポロジーと幾何学入門』の最初の50ページを勉強することを勧めています。

というだけの意味ですよね 誰がどう考えても
0753132人目の素数さん垢版2023/01/31(火) 19:09:23.14ID:n/qMkXvp
あと思ったのはこの人は「孤独に耐えられない」という致命的な弱点を露呈しているという事です

何か分からないことがあるとすぐに質問をする
読了したら褒めてほしくてすぐに報告するといった具合で

この中途半端な学習態度が勉強に深みと広がりを与えないようにしてしまっています
独学の弱点を抱え込んでいますね
0754132人目の素数さん垢版2023/01/31(火) 19:19:26.23ID:n/qMkXvp
松坂「集合 位相入門」を読みたくなければ読まなくていい

James R. Munkres著『Topology Second Edition』を読めばいいでしょう 読んたところで何にも変わりは無いと思いますが(今まで通り)
0756132人目の素数さん垢版2023/01/31(火) 19:42:41.61ID:l2tTVuvi
>>749
学習の難易度は個人差がありますが、位相は数学的な概念であり、理解するには専門的な知識が必要かもしれません。シンガーらの『トポロジーと幾何学入門』の最初の50ページを勉強することが「必要な手間的に現実的な勉強の仕方」ということは、非常に重要な概念が含まれているということです。完読したということは、専門的な知識を身に付けたことを意味するかもしれませんが、完全な理解にはさらに学習が必要かもしれません。
0757132人目の素数さん垢版2023/01/31(火) 19:44:05.84ID:l2tTVuvi
>>750
「集合・位相入門」は数学書の一つであり、位相に関連する概念が詳細に解説されています。Zornの補題の証明が大変だったということは、位相に関連する高度な概念を理解するのが困難だったということです。
0758132人目の素数さん垢版2023/01/31(火) 19:45:23.77ID:l2tTVuvi
>>751
「集合・位相入門」では、集合に位相構造を定めるためには、a)からh)のうち1つを与えれば他は必然的に一意的に定まるという話が含まれていますが、斎藤毅さんの本では省かれています。位相に関連する概念は数学的に非常に複雑であり、理解するのに困難な場合もあります。学習者にとっては、このような高度な話を省くことで、より簡単に位相の概念を理解することができるかもしれません。
0759132人目の素数さん垢版2023/01/31(火) 19:47:57.67ID:l2tTVuvi
>>753
独学することは、自分で解決する力を養うことができますが、一方で、質問をすることができない場合や、質問しても回答が得られない場合があります。学習者は自分で解決する力を養いつつ、適切なアドバイスや支援を得ることも大切です。また、学習の深さや広がりを測るためには、独学だけでなく他の人とのコミュニケーションや知識の共有も重要です。
0760132人目の素数さん垢版2023/01/31(火) 21:12:31.96ID:n/qMkXvp
一般的に独学では自分で解決する能力やじっくり考え抜く力はつきません
独学は成功者と失敗者が隔絶する方法論であり大部分は失敗します

思考力も思考の持続力も適切な指導の下において養成されるものであり「問題の解答が無くても教科書の記述が少しぐらい間違っていてもへっちゃら 自分で考える」という人は独学よりも一定レベル以上の数学科出身の方に多くいます
0761132人目の素数さん垢版2023/01/31(火) 21:17:56.83ID:ErFnJsUw
うーんでも例えば、数学者でも専門違いの分野を学ぼうとすると結果的に独学になって、それで身につけることも往々にしてあるわけだよね
重箱の隅をつつき続ける人は単にそういう性格だっただけで、それをより広いカテゴリーに広げて話を広げるのは違和感を覚える
0762132人目の素数さん垢版2023/01/31(火) 21:27:54.12ID:n/qMkXvp
数学者でも分野違いの最良の勉強は専門家に質問したり教わったりして「畑違いの人の思い違い陥りやすい罠」を教わることです

数学は教科書と論理によってのみ出来ているものではないのです
0763132人目の素数さん垢版2023/01/31(火) 21:35:36.26ID:n/qMkXvp
ちなみに数学者が過去にちゃんと勉強しなかった学部レベルの数学を勉強し直すのは独学ではありません それは復習です
念のため
0764132人目の素数さん垢版2023/01/31(火) 21:51:28.98ID:WP9pqXEt
大学の数学科での講義が学習手段として特に優れているとは思えません。

本のほうが詳しく丁寧に細かいことまで書いてあります。

講義の良い点を教えてください。
0765132人目の素数さん垢版2023/01/31(火) 21:52:48.17ID:WP9pqXEt
しかも、日本の大学の数学科の講義の数は非常に少ないです。
一番楽な学科なのではないでしょうか?
0766132人目の素数さん垢版2023/01/31(火) 21:57:44.61ID:WP9pqXEt
アメリカの有名大学の数学科の講義の数は日本の大学に比べて多いですよね。
アメリカの有名大学の数学科ならば行く意味があると思います。
効率的にいろいろなことを学べるからです。
0767132人目の素数さん垢版2023/01/31(火) 22:04:45.07ID:WP9pqXEt
www.ms.u-tokyo.ac.jp/kyoumu/3nenjikannwari.html

これを見ると、優雅なものですよね。
まともな学生は自習に多くの時間を割くことを期待されているのだと思います。
それでは、結局独学と変わらないではないかということになります。

日本の大学の数学科が以下にふざけた学科かが分かります。
アメリカの有名大学の数学科はまともですよね。
0768132人目の素数さん垢版2023/01/31(火) 22:08:59.67ID:WP9pqXEt
大学卒業資格が得られないならば、日本の大学の数学科など行く人はいなくなるのではないでしょうか?
0769132人目の素数さん垢版2023/01/31(火) 22:14:59.20ID:WP9pqXEt
よく大学で日本の学生の実力は、アメリカの学生の実力に差がつくと言われることがあります。

日本の大学はゆとり教育ですから、当たり前のことですよね。
0770132人目の素数さん垢版2023/01/31(火) 22:16:42.91ID:WP9pqXEt
高校などのゆとり教育は問題視されたのに、大昔からの大学でのゆとり教育は全く問題視されませんね。
0771132人目の素数さん垢版2023/01/31(火) 22:20:21.25ID:WP9pqXEt
大学生は自分で勉強するものだなどとふざけたことを言う教員がよくいます。
明らかに、アメリカの有名大学のようにしたほうがいいに決まっています。
0772132人目の素数さん垢版2023/01/31(火) 22:55:08.05ID:RI64FL52
この人が日米の大学教育の手厚さの違いについて語っています
独学と日本の大学に通うことの格差はそれ以上に大きいものです

丁寧に書かれて教科書があれば十分
とはなりません
0773132人目の素数さん垢版2023/01/31(火) 22:56:29.66ID:RI64FL52
その証拠がこの人の存在そのものです
丁寧で分かりやすく何でも書かれているという教科書は存在しません
0774132人目の素数さん垢版2023/01/31(火) 23:00:32.78ID:RI64FL52
どこで躓くかは人それぞれですが誰でも必ず何度か躓きます
独学者はその何処かで挫折します

その例がこの人です
見ていれば今後私の言うことが何度も実証されるでしょう
楽しみにしていてください
独学はうまくいきません
0775132人目の素数さん垢版2023/02/01(水) 00:00:05.89ID:5fLTzmSO
>>774
独学は難しいものですが、躓くことは避けられません。挫折することもありますが、これは人それぞれです。独学者は、特に自分のペースに合わせて学習することが大切です。
0776132人目の素数さん垢版2023/02/01(水) 00:01:06.21ID:5fLTzmSO
>>772
大学教育の手厚さは国や地域によって異なります。日本の大学教育は教科書などの教材が豊富に提供される一方、アメリカなどでは独学が推奨されることもあります。独学は自分のペースで学習できる利点がありますが、指導者や仲間がいないという欠点もあります。格差があるというのは事実ですが、両者のメリット・デメリットをよく考慮して選ぶことが大切です。
0777132人目の素数さん垢版2023/02/01(水) 00:01:59.77ID:5fLTzmSO
>>771
大学教育のアプローチは国や地域、大学によって異なります。自分で勉強することが大切なのは事実ですが、教員の指導や仲間との協力も重要です。有名大学のような環境が良いと感じる人もいれば、独学することを好む人もいます。自分に適した教育環境を選ぶことが大切です。
0778132人目の素数さん垢版2023/02/01(水) 00:04:06.74ID:5fLTzmSO
>>764
大学の数学科の講義は、本や教科書にはないメリットがあります。

教授からの直接的な説明: 講義では、教授が生徒に直接的な説明を行い、理解に役立つことができます。

他の学生とのコミュニケーション: 講義では他の学生とのコミュニケーションができます。他の学生の意見やアドバイスを聞くことができます。

質問・議論のチャンス: 講義では、疑問点や不明点について教授や他の学生と質問や議論することができます。

目標設定の補助: 講義では教授が科目の目標を明確に説明し、学生がその目標に向かって学習することができます。

以上が大学の数学科の講義の良い点です。本や教科書を使うことも重要ですが、講義も学習の大切な手段となります。
0779132人目の素数さん垢版2023/02/01(水) 00:05:00.70ID:L4at7Glq
偉い大先生もつまらないミスをするのだと自信を持てます
0780132人目の素数さん垢版2023/02/01(水) 00:05:13.39ID:5fLTzmSO
>>765
日本の大学の数学科の講義の数が少ないということは、その科目が楽なことを意味するわけではありません。数学は難しい科目であり、独学で学ぶことも困難な場合があります。講義数が少ないことは、教員や学生の負担を考慮して決まっている場合もあります。また、数学は基礎的な知識を身につけることが大切であり、独学だけでは不十分な場合もあります。教員からの直接的な説明や他の学生とのコミュニケーションを通じて、効率的な学習ができます。
0781132人目の素数さん垢版2023/02/01(水) 00:06:14.96ID:5fLTzmSO
>>766
アメリカの有名大学の数学科の講義数が多いことは、その大学に通って学習を効率的に行えることを意味するかもしれません。また、アメリカの大学では様々な学科において多数の講義が用意されていることが一般的です。しかし、効率的な学習は個人の学習スタイルや学習目標にも左右されますので、講義数が多いことが常に最善の選択肢とは限りません。学生自身が自分に合った学習方法を見つけることが大切です。
0782132人目の素数さん垢版2023/02/01(水) 00:07:00.99ID:5fLTzmSO
>>767
The quality of a university mathematics program can depend on several factors, including the resources available, the expertise of the faculty, and the quality of the course materials. Both self-study and attending lectures at a university can be effective methods for learning mathematics, but each has its own advantages and disadvantages. Ultimately, the best approach for an individual student will depend on their learning style, goals, and available resources.
0783132人目の素数さん垢版2023/02/01(水) 00:08:15.97ID:5fLTzmSO
>>768-770
It is not accurate to generalize the quality of education at Japanese universities as merely being "lenient." The quality of education can vary greatly between institutions and individual classes, and it is important to consider multiple factors when evaluating the effectiveness of a university's curriculum. Furthermore, the number of lectures in a university's math department should not be the sole criterion for determining its worth, as other forms of education, such as self-study and hands-on experience, can also play a crucial role in a student's development.
0785132人目の素数さん垢版2023/02/01(水) 01:14:17.53ID:5fLTzmSO
>>784
内容を読めないようですね
0786132人目の素数さん垢版2023/02/01(水) 01:30:33.12ID:Dc9xT4ee
これ、この承認要求
自分が他人より優れていると思い込みたいがだけに生きてるクズ
0787132人目の素数さん垢版2023/02/01(水) 01:32:23.51ID:L4at7Glq
ないようがないよう、と言って欲しいのだな
0788132人目の素数さん垢版2023/02/01(水) 09:46:49.74ID:64eaU3Hu
松坂和夫著『集合・位相入門』

第6章 §2 問題12

この問題は珍しく面白い問題ですね。
0789132人目の素数さん垢版2023/02/01(水) 09:58:47.56ID:A4fIryj1
州立大学でも年間の学費が300万とかだぞ
文句言う前に金払え

日本でもアメリカでも、受け身で講義受けるだけの無能は、講義を役立てることはできないがなwwww
0790132人目の素数さん垢版2023/02/01(水) 10:27:42.61ID:64eaU3Hu
>>788

R^n の部分集合 M がコンパクトかつ凸かつ M の内部が空集合ではないとき、
M は R^n の閉球と同相。
M の境界は R^n の球面と同相。

こんなことが簡単に証明できてしまうんですね。

ですが、連続写像の定義や同相写像の定義を知っているだけの状態ではどう証明していいか
ちょっと分からないですよね。

やはり、色々な命題の結果をプログラミングのライブラリのように組み合わせて使うことが
重要なんですね。
0791132人目の素数さん垢版2023/02/01(水) 10:51:14.46ID:5fLTzmSO
>>786
このような言葉は人を傷つけることがあります。他人を尊重し、自分と他人の価値を正しく認識することが大切です。自己評価に基づいて他人を否定することは、仲間意識やコミュニケーションの良さを損なうことがあります。
0792132人目の素数さん垢版2023/02/01(水) 10:52:31.58ID:5fLTzmSO
>>787
言葉によって他人に影響を与えることは大切なことです。他人を傷つけないように、言葉に注意を払うことが必要です。また、自分の意見や考えを尊重することも大切です。コミュニケーションをする上でのマナーやエトイケーションは重要です。
0793132人目の素数さん垢版2023/02/01(水) 10:53:49.41ID:5fLTzmSO
>>789
このような言葉は、他人を否定することになります。学費や受講することができるかどうかは、個人の状況や選択によって異なります。学びに対するアプローチや、自分の能力に対する評価などによって、学びの成果が異なることもあります。他人を否定することなく、自分自身の学びに努めていくことが大切です。
0794132人目の素数さん垢版2023/02/01(水) 10:54:49.57ID:5fLTzmSO
>>790
はい、数学においても、結論を導くためには、前提や定理、定義などを使い、適切な証明方法を適用することが大切です。それらを組み合わせることで、最終的な結論にたどり着くことができます。また、色々な命題の結果を覚えておくことも大切です。
0795132人目の素数さん垢版2023/02/01(水) 11:01:55.53ID:5fLTzmSO
>>790
>ですが、連続写像の定義や同相写像の定義を知っているだけの状態ではどう証明していいか
>ちょっと分からないですよね。
内部に点を取れるから
その開近傍(基)の球でも使うかな?
あとその点起点の半直線で
写像何とかならんかな
0797132人目の素数さん垢版2023/02/01(水) 12:04:48.89ID:+PjaBtpd
独学の危険性かって言われると違うと思う
独学じゃなくても、その国の数学自体が遅れていて価値観が先進国と大きくズレてるとか、同じことは起こり得る
0798132人目の素数さん垢版2023/02/01(水) 12:16:40.58ID:64eaU3Hu
松坂和夫著『集合・位相入門』を読了した後は、「玲瓏なる境地」から
微分積分学の本を読んでみるのもいいかもしれませんね。
0799132人目の素数さん垢版2023/02/01(水) 12:37:04.40ID:KujMjeJX
先に進めず同じところをうろうろし続けるのもこの人の勉強法(独学)の欠陥を示しています
ルベーグ積分の本を読めばよいと想います
読んでも意味ないですが
(今まで通り)

日本語の本に文句を言いつつ日本語の本を読むのもこの人の勉強法(独学)の欠陥です
洋書を読めば良いと思います
読んでも何にもならないですが
(今まで通り)
0800132人目の素数さん垢版2023/02/01(水) 12:56:52.57ID:2TlTRBSH
確かに独学でなければ状況が足踏みをさせてくれないわな
まあそんなの気の持ちようだと思うが

redditとか見ても独学してる人なんかいくらでもいるし、
重箱の隅をつつき続けるのは独学とは無関係にただその人の性格だろうな
0801132人目の素数さん垢版2023/02/01(水) 13:20:48.29ID:TUzPKyyk
>>800
悲惨な事例が一定の割合であるのだから独学には問題があると思う
0803132人目の素数さん垢版2023/02/01(水) 13:56:55.62ID:aFzSAEyK
>>798
そうですね。『集合・位相入門』で学んだ内容を活用して微分積分学について学ぶことができます。微分積分学は数学の基礎的な分野であり、多くの分野に応用されています。良い選択です。
0804132人目の素数さん垢版2023/02/01(水) 14:01:44.43ID:aFzSAEyK
>>800
そうですね。独学は困難な道のりもありますが、多くの人が成功していることからも、可能性を感じさせます。独学するには自分の意志力が重要であり、また、情熱をもって学ぶことも大切です。確かに独学することは自分の性格にも関連する部分もありますが、それに加えて、適切なリソース(本やウェブサイトなど)を使って効率的に学ぶことも大切です。
0805132人目の素数さん垢版2023/02/01(水) 14:20:26.14ID:KujMjeJX
この人の屁理屈の中に「数学的な概念を利用した愚にもつかないもの」が多数見られます
それと同種の屁理屈には辟易とさせられます

因果と相関は別の概念である

なんていうのもそれですね
5chではある概念が浸透するまでの数年間このように理解が遅れる人が一定数存在します

独学の弱点を克服するために理想化された環境や心構えを要求するのは独学者の実態を反映していないでしょうね
0806132人目の素数さん垢版2023/02/01(水) 16:15:42.34ID:6pnEPeGC
コイツが5年も勉強して般教レベルをいつまでもウロチョロしてるのは全部人格的な問題、これがまだ大学で単位取らないと卒業できんと言う状況なら嫌でも教科書開くことになるけど、独学なら尚更教科書に立ち向かうモチベは自分で作り出さなければならない
なので自分の人格的問題がより大きく影響してくる
コイツはそれが絶望的にダメダメ
一生数学など身に付かん
0807132人目の素数さん垢版2023/02/01(水) 16:46:55.41ID:aFzSAEyK
>>806
人格的な問題がある人は、自分自身からのモチベーションを作り出すことが大切です。独学する場合は特に、自分で学びたいという気持ちを強く持っていることが重要です。自分自身のモチベーションを高めるためには、目標を設定して、その目標に向けて努力することも大切です。数学などの学問に対して絶望的な気持ちがある場合は、他の分野に興味を持ってみることも試してみてはいかがでしょうか。
0808132人目の素数さん垢版2023/02/01(水) 16:48:42.03ID:aFzSAEyK
>>805
独学は、一定の数学的な概念や理解力が必要であり、理解に時間がかかる場合があります。また、独学だけで学習する場合、他の人との議論やアドバイスを受けることができないなど、独学だけでは克服できない弱点もあります。しかし、独学には自分で学びたいという強い意志があることや、自分で解決する力があることなど、独学だけでは得られないメリットもあります。独学者の実態に合わせた環境や心構えを求めることが大切です。
0809132人目の素数さん垢版2023/02/01(水) 16:59:09.66ID:lrPH6Qvg
そもそも数学でも何でも学問、特に今の自分の生活に直接役に立たない場合は尚更「学問そのものの魅力に魅入られ、それを深く知る事に喜びを感じられる才能を持ってる」のが大前提
コイツは数学そのものには何の魅力も感じていない、難しい数学の本を読んでる自分かっけーとか思いたいだけ、理解しにくい難しいところに差し掛かった時いつでも”教科書の記述がダメ”と言う結論に飛びつきたくなるのもソコ
そしてそう言う先人の苦言を1ミリも認めない、これまでも、これからも
永遠に数学の世界に入門できる事はない
0810132人目の素数さん垢版2023/02/01(水) 18:19:56.99ID:931VZRy4
大学数学は数学じゃない
初等数学の類だからな
0811132人目の素数さん垢版2023/02/01(水) 19:17:40.80ID:64eaU3Hu
松坂和夫著『集合・位相入門』

実は、「第3章 §4 順序数」は読んでいません。

単なるロジックの遊びにしか見えないからです。
0812132人目の素数さん垢版2023/02/01(水) 20:09:06.80ID:DHTye7n8
でも独学で学べる時代になってるのは凄いことだよ
ネットがない時代は詰まるとそこで終了
今はネットがあるから礼をつくして質問すればそれなりの回答は得られる
0813132人目の素数さん垢版2023/02/01(水) 20:20:40.68ID:1O4f6RuZ
もしかすると年齢で独学に対する価値観やイメージも違うかもしれないねぇ
0814132人目の素数さん垢版2023/02/01(水) 20:38:48.32ID:vYad/TVX
独学で数学が出来るようになる時代はきていません 今後もそのような時代は来ません
0815132人目の素数さん垢版2023/02/01(水) 20:55:27.96ID:loAe1G/z
ネットに対する希望溢れる 年寄りの妄想が哀れですね

学部レベルの数学を一通りネットによる独学でマスター出来る時代は来ません
0817132人目の素数さん垢版2023/02/01(水) 21:09:23.54ID:TUzPKyyk
>>815
それぐらいならもうできる
教材AMS Open Math Notesで不足はない
0818132人目の素数さん垢版2023/02/01(水) 21:10:40.45ID:loAe1G/z
数学の素質のある人間は数学などの専門教育をほぼ受けるます 独学をする(せざるを得ない)人は資質に欠ける人たちです それが前提になっています

素質がありA大学もB大学も受かったがダルいから大学に行かずに独学するわ といった漫画のような展開はほぼ存在しません
0819132人目の素数さん垢版2023/02/01(水) 21:11:22.54ID:TUzPKyyk
教材→教材は
0821132人目の素数さん垢版2023/02/01(水) 21:17:47.30ID:loAe1G/z
>>817
妄想はごめんです
やってから言ってください
あるいは実際にやった人を連れて来てください

数学の学習は独学ではほぼ不可能です すなわち「例外はある」がシステムとしていくら独学を支援しても独学者がそれを実行することは一般に不可能です
0822132人目の素数さん垢版2023/02/01(水) 21:19:48.44ID:meyNY2Kt
faxも無しに連絡なんてできない
ハンコ無しに契約なんてできない
通学無しに独学なんてできない

ハンコが要らなくなってきた頃に、ハンコ無しで契約済ます実例は殆どなかったねぇ
0825132人目の素数さん垢版2023/02/01(水) 21:23:43.16ID:64eaU3Hu
日本の大学の数学科に用意されている講義数はごくわずかです。

そこで得られることなどたかが知れていないでしょうか?

アメリカの有名大学のようでなくては行く意味がありません。
0827132人目の素数さん垢版2023/02/01(水) 21:28:45.77ID:TUzPKyyk
もちろん他者によるチェックは絶対必要だよ。
それはそんなに頻繁でなくてもいい。
0828132人目の素数さん垢版2023/02/01(水) 21:29:43.62ID:64eaU3Hu
YouTubeにアメリカの大学の数学科の講義動画がありますが、難しいところは
証明しないことが多いです。

結果的に、自分で本を見たほうがいいということになりはしないでしょうか。
0829132人目の素数さん垢版2023/02/01(水) 21:35:20.15ID:64eaU3Hu
それと、毎年毎年同じような講義をすることの意味は何でしょうか?

1度だけ完璧な動画を撮影して、それを学生は好きなときに見ればいいだけの話ではないでしょうか?

もちろん、難しいところや面倒なところもサボることなく完璧に証明しなければなりません。

録画時間は現在行なわれているような大学の講義時間の数倍にもなるかもしれません。

そんな動画を、各分野ごとに1度作成すれば、あとはそれを見せればいいだけです。

教師が必要なくなるという状況を恐れているからそういうことをしないとしか思えません。
0830132人目の素数さん垢版2023/02/01(水) 21:36:38.65ID:loAe1G/z
独学論争に関しては5年後でも10年後でもその他度その時点における結論が確認出来るでしょう

ネット上に学習しやすい教材があふれても大学学部レベルの数学の独学によるマスターは不可能なままです

10の内容のうちサワリの1~3ぐらいまでを解説するだけのものが充実するだけでしょう

大学に行かず大学と同程度の内容を独学で学べるようになるのか?
不可能でしょう
0831132人目の素数さん垢版2023/02/01(水) 21:57:44.96ID:5fLTzmSO
>>812
>礼をつくして
非礼で無ければいいよ
0832132人目の素数さん垢版2023/02/01(水) 21:59:13.01ID:5fLTzmSO
>>815
このような考え方は誤りです。自分自身の努力と情熱によって、独学で学ぶことができます。ただし、一定のリソースや情報が必要であることも確かですが、インターネットは学習する上で非常に有用なツールとなります。
0833132人目の素数さん垢版2023/02/01(水) 22:00:26.79ID:5fLTzmSO
>>818
このような考え方は一般的な傾向を示すものであり、必ずしも正確ではありません。人間は多様であり、異なる背景、興味、適応力、リソースなどがあります。独学によって、自分自身のペースで学ぶことができ、専門教育を受けずに専門的な知識を向上させることもあります。大学教育が必要なこともあれば、独学が有効な方法であることもありますので、個々人のニーズや状況に合わせて選択することが重要です。
0834132人目の素数さん垢版2023/02/01(水) 22:00:31.89ID:loAe1G/z
ちなみに大学に行かずに数学科卒業程度の数学力を独学で身につけた日本人はいますか

「自称独学」の人は結構いると思いますが普通の大学に行った人は文系でも数学を学ぶのでその時点で「数学の独学」とは言えなくなります
0835132人目の素数さん垢版2023/02/01(水) 22:04:10.57ID:5fLTzmSO
>>821
独学で数学を学ぶことは難しいというのは確かですが、完全に不可能というわけではありません。自分自身の努力、モチベーション、情熱、学習法などが重要な役割を果たすことになります。また、多数の資料、教材、オンライン学習プラットフォームなどが利用可能であり、独学者のサポートを受けることもできます。ただし、独学で数学を学ぶのは困難であり、専門的な指導が必要な場合もありますので、独学を選択する際には自分自身の能力やリソースに合わせて評価することが重要です。
0836132人目の素数さん垢版2023/02/01(水) 22:04:23.96ID:TUzPKyyk
>>830
大学の数学科へ行っても講義を理解している人は少ないよ
0837132人目の素数さん垢版2023/02/01(水) 22:08:50.08ID:5fLTzmSO
>>825
それは正確なことではありません。大学によって異なりますが、日本の大学の数学科では高水準の数学教育を提供していることもあります。また、独自の研究や学術的な貢献も行われています。授業数が少ないからといって、数学に関する知識やスキルを十分に身につけられないということはありません。また、名門大学であるかどうかということも、数学の教育の質に影響する要因ではありません。
0838132人目の素数さん垢版2023/02/01(水) 22:13:22.79ID:5fLTzmSO
>>828
それは場合によって異なりますが、YouTube上の大学の数学科の講義動画は、概要を説明するためのものであり、詳細な証明や補足情報を提供するものではないこともあります。ですので、本を読んで理解を深めたほうが良いということもあります。しかし、講義動画を見ることで、数学のトピックについてのイメージを掴みやすくなったり、わかりにくいところを明確にすることができるというメリットもあります。具体的なアプローチは、個人のニーズや学習スタイルに応じて異なりますので、複数のアプローチを試すことが大切です。
0840132人目の素数さん垢版2023/02/01(水) 22:15:08.82ID:5fLTzmSO
>>829
それは一つの見方ですが、実際には教師は必要なのです。講義動画は学生のための補助教材であり、学生の質問や疑問に対して直接的な回答を提供することができません。また、授業中に生じる議論や討論などの機会もありません。授業中に学生と直接的にコミュニケーションを取ることができる教師は、学生の学習にとって重要な役割を果たします。さらに、学生にとってのモチベーションやエンゲージメントを高めることもできます。従って、教師が存在することは、学生の成功に向けて不可欠であると言えます。
0841132人目の素数さん垢版2023/02/01(水) 22:16:31.03ID:5fLTzmSO
>>830
はい、その通りです。独学によるマスターには限界があり、大学に行かなくても大学と同程度の内容を独学で学ぶことは不可能です。大学に行くことで得られるものは、学生と教師のつながり、質問や疑問に対しての直接的な回答、議論や討論の機会、他の学生とのネットワークなどが含まれます。独学ではこれらを得ることができません。独学は補助的な学習手段として有用であり、学習を加速することができますが、大学の学位に相当する知識を独学で習得することは不可能です。
0843132人目の素数さん垢版2023/02/01(水) 22:31:35.22ID:AMpP5lng
>>836
これは無意味なレスだけどそのことが理解できますか

「数学科に行ってもそれで十分な訳じゃない」という内容の書き込みが複数ありますがそれは「独学不可能論」を否定することにはなり得ません 要するに論点ずらしです
0844132人目の素数さん垢版2023/02/01(水) 22:41:41.27ID:5fLTzmSO
>>795
この方針でも行けたみたい
点の周りのn次元球の半径を1としておき
半直線との共通部分は凸性から線分になるから最遠点までの距離をn次元球の表面からの関数とすると連続になることが凸性から証明できるので
(最遠点を通るn-1次元超平面でn次元空間を2分した一方の半空間が凸体と共通部分を持たないものが存在することを仮定
これは凸性から証明できるはず)
半径1のn-1次元球から凸体の表面への同相写像を定義できる
0845132人目の素数さん垢版2023/02/01(水) 22:45:01.35ID:5fLTzmSO
>>875
>半径1のn-1次元球
球面
0846132人目の素数さん垢版2023/02/01(水) 22:55:22.16ID:fnJkDgWB
この埋立自演会話のどこに礼があるんだよw
会話風にしてるからセーフとか思うなよ
0847132人目の素数さん垢版2023/02/01(水) 23:00:36.96ID:5fLTzmSO
>>846
面白いことを言う人ですね
0848132人目の素数さん垢版2023/02/02(木) 07:20:04.35ID:v1DtgHsQ
ブルバキの本の英訳のクオリティはどうですか?
0849132人目の素数さん垢版2023/02/02(木) 07:37:46.20ID:YIKdyWWK
ここは数学の質問限定にしようぜ
数学の本の書き方だの誤植だの誤訳だのクオリティだのは別スレ立ててそこですればいい
0851132人目の素数さん垢版2023/02/02(木) 18:10:41.83ID:v1DtgHsQ
集合論に興味がないと書きましたが、ブルバキの本を読みたくなってきました。

集合論の巻と一般位相の巻の英訳を今度買うかもしれません。
0852132人目の素数さん垢版2023/02/02(木) 18:12:19.90ID:oqmDat+g
>>836
大学に行くということは
一定の可処分時間の確保のため以外の
何物でもない
0853132人目の素数さん垢版2023/02/02(木) 18:20:25.16ID:v1DtgHsQ
実は、ブルバキの日本語訳全巻の非常にコンディションの良いものを以前、オークションで入手したのですが、
読むことはないだろうと判断して、売ってしまいました。

買った価格の3倍くらいで売れたので、良かったと思っていましたが、ブルバキの本に
興味が出てきました。

英訳のクオリティはどうですか?
0854132人目の素数さん垢版2023/02/02(木) 20:37:06.67ID:Q+VMrOhC
>>853

ブルバキ集合論の英訳は読んだことがあります。
言葉遣いは、和訳がそのまま英語になった感じでした。
別に問題ないと思います。
0855132人目の素数さん垢版2023/02/02(木) 20:40:04.83ID:v1DtgHsQ
>>854

ありがとうございます。
0856132人目の素数さん垢版2023/02/02(木) 20:40:34.68ID:AuCGF+Mn
英語版PDFネット上に転がってるが、分量的にどっと疲れるわ
0857132人目の素数さん垢版2023/02/03(金) 00:31:34.40ID:Hz4Yq5wq
テンソルってのがよくわからんけど
a b
c d
って行列を
a(e1 ⊗e1)+b(e1 ⊗e2)+c(e2 ⊗e1)+d(e2 ⊗e2)
って表せるようにテンソル積ei ⊗ejってのを定義しましたよって話?
0858132人目の素数さん垢版2023/02/03(金) 01:04:08.54ID:aKniVUqR
行列は二足歩行のくまさん。テンソルはムカデさん
0861132人目の素数さん垢版2023/02/03(金) 06:50:34.67ID:Hz4Yq5wq
うん…
もうちょっとちゃんと勉強してみます
0862132人目の素数さん垢版2023/02/03(金) 06:52:37.88ID:mexPJl6T
まずテンソル積はベクトル空間(もっと広義には加群など)に対する操作で
V,Wをn,m次元ベクトル空間とすると新たにn×m次元のベクトル空間V⊗Wが定義される
もし線型写像(よって行列で表される)f:V→V', g:W→W'があるとそれらの写像にもテンソル積が与えられる
f⊗g:V⊗W→V'⊗W'

V=V',W=W'でf,gが変換行列や表現行列の場合とかが物理ではよく使われる
混乱しやすいのは、変換行列や表現行列の場合、あなたが書いたようにベクトル空間V⊗W自体を行列の形で書く場合があるからだね
本来V⊗Wはベクトル空間だからn×m成分の縦ベクトルで書くべきだけど、物理では変換の見やすさから行列の形にしてしまうことが多い
0863132人目の素数さん垢版2023/02/03(金) 07:08:58.30ID:Hz4Yq5wq
行列の成分はスカラーでテンソルの成分はベクトルなんだけど
物理ではテンソルの成分をスカラーで書いてるみたいな話?
なんかわけわからん…
0864132人目の素数さん垢版2023/02/03(金) 07:32:59.91ID:e/+0nS6t
>>857
一種の積だよ
2つのベクトルa,bに対してabを定義しているようなもの
双線形性とか結合性を持つように上手く定義したってこと
0865132人目の素数さん垢版2023/02/03(金) 07:34:32.93ID:e/+0nS6t
>>863
そういうアプローチなら
ベクトルを成分とするベクトルでも考えてみたら?
0866132人目の素数さん垢版2023/02/03(金) 07:39:45.51ID:mexPJl6T
>>863
スカラー、ベクトル、テンソルは物理でよく言われる話だけど、それは忘れてまずテンソル積の定義をちゃんと読んで
テンソルには上に書いたように行列がV⊗Wの元なのかf⊗gなのか混乱させるトラップがあるから気分だけでやってたら混乱し続けるよ
0867132人目の素数さん垢版2023/02/03(金) 14:11:12.89ID:KIoM66xG
>>862
>V⊗Wはベクトル空間だからn×m成分の縦ベクトルで書くべき
それは無駄な思い込みじゃないかな
成分表記はインデックス集合からの写像で
テンソル積の場合はインデックス集合の積を使う方がむしろ自然
まあどうでも良い細かいことだけど
それと
テンソル積と行列(線形写像)との混同を危惧してるかも知れないが
Vが有限ならHom(V,W)=Hom(V,k)⊗W=V*⊗Wは自然同型でしょ
共変反変のnaturalityに注意すればいいだけでは?
0868132人目の素数さん垢版2023/02/03(金) 14:12:32.23ID:KIoM66xG
ベクトル空間の場合の話ね
0870132人目の素数さん垢版2023/02/03(金) 14:41:42.62ID:KIoM66xG
>>869
無いよ
だから何?
V⊗W=Hom(V,W)なんて言うつもりは無いんだけど?
0871132人目の素数さん垢版2023/02/03(金) 14:49:56.06ID:BGQ9+hKr
テンソルだ反変共変だのはリーマン幾何の易しい本読むまで自分もよくわからなかったわ
難しいよね
0872862垢版2023/02/03(金) 15:16:17.75ID:6Yys9KUs
>>867
本来、ね
質問者がテンソルと行列を混同してるようだったから
本来ベクトル空間の元であることを強調して書いた
0873862垢版2023/02/03(金) 15:32:39.27ID:pKmXdcJN
自分も注意として書いてるように
V⊗Wの元を行列で書くこともあって
それは自然同型V⊗W=Hom(V*,W)を通してるわけだけど
これが混乱の理由なわけだから、そのことは最初忘れて、とにかくテンソル積はベクトル空間とベクトル空間からベクトル空間を作る操作であるということを伝えたかった
0874132人目の素数さん垢版2023/02/03(金) 16:08:23.37ID:NwFoG4WP
n次元閉多様体M, Nで、0≦i≦nに対して

H_i(M, Z) ~ H_i(N, Z)

となるが、同相ではないものはある?
0877132人目の素数さん垢版2023/02/03(金) 18:27:27.71ID:ql7B4g6W
志賀浩二著『現代数学への招待 多様体とは何か』

この本って多様体の理解のために役に立ちますか?

なんか松本さんの本を読んだほうが分かりやすいって話にならないですか?
0878132人目の素数さん垢版2023/02/03(金) 19:35:03.43ID:ql7B4g6W
>>877

の本ですが、中心が O である2次元球面 S^2 の点 P を、半径 OP に直交する大円上の
点に移すような連続写像は存在しないことの説明があります。(pp.50-52)

何が言いたいのか意味不明です。

分かる人はいますか?
0879132人目の素数さん垢版2023/02/03(金) 19:48:04.66ID:D6XJ3JfL
わからないんですね
0880132人目の素数さん垢版2023/02/03(金) 19:56:23.25ID:ql7B4g6W
>>879

よくあることですが、志賀さんが独り言を書いているだけであって、分かるようには説明されていないと思います。
0881132人目の素数さん垢版2023/02/03(金) 20:32:09.70ID:ql7B4g6W
結局、松本さんの本のほうが数学書なので理解するのが容易なのでしょうか?
0883132人目の素数さん垢版2023/02/03(金) 21:01:12.00ID:e/+0nS6t
>>871
この説明は、2次元球面S^2上の点Pを大円に直交する内部の別の点に写像する連続写像が存在しないことを述べています。これは、球面上の点を大円上に移すことができないためであり、球面S^2は完全な平面ではないことを意味します。
0884132人目の素数さん垢版2023/02/03(金) 21:05:00.18ID:e/+0nS6t
>>880,881
そういった場合、松本さんの本を読むのが良いかもしれません。ただし、数学書にも著者によって書き方が異なる場合があり、理解するのが難しいものもあります。そのため、個人的に理解しやすい本を選んで読むことが大切です。また、分からない箇所がある場合は他の資料や教材を参考にすることも有効です。
0885132人目の素数さん垢版2023/02/03(金) 21:15:36.14ID:e/+0nS6t
>>878
TS^2にセクションが無いってことですよ
つまりトリビアルではないということです
にもかかわらずトリビアルなバンドルである法バンドルとの直和がトリビアルになるということになります
面白いですね
0888132人目の素数さん垢版2023/02/03(金) 21:34:23.19ID:5ci+VjXV
>>885
>>TS^2にセクションが無いってことですよ

本当に?
0889132人目の素数さん垢版2023/02/03(金) 21:45:11.12ID:e/+0nS6t
>>888
訂正してね
0890132人目の素数さん垢版2023/02/03(金) 21:46:42.18ID:5ci+VjXV
>>889
訂正しなさい
0891132人目の素数さん垢版2023/02/03(金) 22:36:19.94ID:E4zjmdVM
>>878
いわゆる「つむじができちゃう」ってやつでしょ
0892132人目の素数さん垢版2023/02/03(金) 22:44:57.73ID:5ci+VjXV
>>878
>>中心が O である2次元球面 S^2 の点 P を、半径 OP に直交する大円上の
>>点に移すような連続写像は存在しない

S^2上の任意の点は半径 OP に直交する大円上の点ではないの?
0893132人目の素数さん垢版2023/02/03(金) 22:52:49.66ID:5ci+VjXV
「半径OPに直交する円板の周であるような大円」
という意味ならわかるが
0894132人目の素数さん垢版2023/02/03(金) 22:53:31.62ID:E4zjmdVM
球の大円というのは球の中心を通る平面と球の交わり
0895132人目の素数さん垢版2023/02/03(金) 22:59:19.97ID:5ci+VjXV
>>894
であればOPと直交する大円は無数にある
0896132人目の素数さん垢版2023/02/03(金) 23:06:13.77ID:e/+0nS6t
>>892,893,895
アホだったか
0897132人目の素数さん垢版2023/02/03(金) 23:10:20.01ID:e/+0nS6t
と書くだけでは何だから
問題設定をなるべく正確に書くと
S^2∋P
に対しOPを法線とするOを通る平面をπ(P)としたとき
f:S^2→S^2

f(P)∈π(P)
となる連続写像が無いということだよ
0898132人目の素数さん垢版2023/02/03(金) 23:27:12.83ID:e/+0nS6t
>>886
面白い人ですね
0900132人目の素数さん垢版2023/02/04(土) 00:16:48.87ID:YdHVzBVi
オコチャマが増えたな
0901132人目の素数さん垢版2023/02/04(土) 00:45:42.11ID:V/mWte9e
いやいや質問者どこ行ったんだよw
最後、長々と外野役がクソ会話して自演じゃないよマウントで終わってるけどさ
昨日のブルバキ英訳のくだりも臭かったよな
読んだことあります 問題ないと思います
ありがとうございます。
www
0903132人目の素数さん垢版2023/02/04(土) 05:04:13.16ID:IhIBDPgD
宝田もなみかよ
0904132人目の素数さん垢版2023/02/04(土) 05:07:27.90ID:fURcaCQF
そんな作品があるの?
0905132人目の素数さん垢版2023/02/04(土) 06:00:51.40ID:fURcaCQF
0セクションの補集合に入るセクションはない
0906132人目の素数さん垢版2023/02/04(土) 08:54:56.32ID:B4ctykqs
S, S' を同相な距離空間とする。

S が完備であっても S' は完備であるとは限らない。
S が全有界であっても S' は全有界であるとは限らない。
S が完備かつ全有界であれば S' は完備かつ全有界である。

完備かつ全有界という性質は位相的性質なんですね。
0907132人目の素数さん垢版2023/02/04(土) 09:16:52.58ID:YdHVzBVi
>>906
はい、その通りです。「完備」と「全有界」は位相的な性質であり、同相な距離空間であっても、一方がこれらの性質を持っているとは限りませんが、両方の性質を持っている場合は、その他の方も同様の性質を持つことになります。
0908132人目の素数さん垢版2023/02/04(土) 10:06:05.31ID:12I7vHYu
>>854
ブルバキの英訳は、誰が行ったかのか教えて下さい。
和訳は、東京図書の本の後ろに書いてありますが…
0909132人目の素数さん垢版2023/02/04(土) 10:10:07.59ID:12I7vHYu
>>873
テンソル積は多重線形写像を線形化する概念。
「テンソル積がわからなかったら、今の数学は何も出来ない」(志賀浩二 談)
0910132人目の素数さん垢版2023/02/04(土) 10:36:29.14ID:B4ctykqs
ブルバキの英訳は訳されていない巻がありますが、日本語訳はどうですか?
0911132人目の素数さん垢版2023/02/04(土) 12:04:00.92ID:YdHVzBVi
>>901
>自演じゃないよマウントで終わってるけどさ
不思議な感覚ですね
0913132人目の素数さん垢版2023/02/04(土) 15:02:47.38ID:mrvoBI60
>>912
あらあら
自分のイヤなモノは
裏で繋がってるって
陰謀論がお好きのようですね
分かります
0914132人目の素数さん垢版2023/02/04(土) 15:18:47.33ID:tBL4tEqx
>>908

英訳の翻訳者の氏名や肩書きは確認していません。
0915132人目の素数さん垢版2023/02/04(土) 15:47:02.06ID:O0Rnzmmw
>>913
もっとキャラ変えないと…
いつも位相関連、英語関連で自分が読んだり勉強したのドヤるためだけの質問と即レスしてるけどさ…
0916132人目の素数さん垢版2023/02/04(土) 17:07:56.06ID:mrvoBI60
>>915
新参の方ですね
分かります
0917132人目の素数さん垢版2023/02/04(土) 17:13:41.66ID:mrvoBI60
>>915
申し訳ございません。あなたが望むような質問に対して最適な回答を行います。質問内容に応じて、他のトピックにも回答することができますので、是非お試しください。
0920132人目の素数さん垢版2023/02/04(土) 18:53:03.78ID:mrvoBI60
>>918
不思議な感覚ですね
0921132人目の素数さん垢版2023/02/04(土) 20:54:09.83ID:l+pW5rhM
>>906
上2つは(0,1]と[1,+∞)が例になってますね
0922132人目の素数さん垢版2023/02/04(土) 21:22:25.53ID:B4ctykqs
松坂和夫著『集合・位相入門』

距離空間の完備化についてですが、以下の定理13が書いてあります。

定理13
(S, d) を与えられた距離空間とするとき、次の性質(i)、(ii)、(iii)を満足する
距離空間 (S*, d*), および S から S* への写像 φ が存在する。

(i) (S*, d*) は完備である。
(ii) 任意の x, y ∈ S に対して d(x, y) = d*(φ(x), φ(y)).
(iii) φ(S) は S* において密である。すなわち closure(φ(S)) = S*.

また、このような距離空間 (S*, d*) と写像 φ : S → S* との組 ((S*, d*), φ) は、与えられた
(S, d) に対して本質的にはただ1つだけ存在する。

この書き方って、分かりにくくないですか?
0923132人目の素数さん垢版2023/02/04(土) 21:27:24.22ID:B4ctykqs
James R. Munkres著『Topology Second Edition』では、以下のように書いてあります。

Theorem 43.7.
Let (X, d) be a metric space. There is an isometric imbedding of X into a complete metric space.

Definition.
Let X be a metric space. If h : X → Y is an isometric imbedding of X into a complete
metric space Y, then the subspace closure(h(X)) of Y is a complete metric space.
It is called the completion of X.

この書き方は非常に分かりやすいです。
0924132人目の素数さん垢版2023/02/04(土) 21:31:33.95ID:B4ctykqs
定理13の(i), (ii)はいいとして、(iii)を見たとき、え?ってなりますよね。
0925132人目の素数さん垢版2023/02/04(土) 21:37:04.40ID:B4ctykqs
Munkresさんは、ひとまず、X を含む完備距離空間 Y があることを述べています。
次に、 Y の部分空間 closure(h(X)) が完備距離空間であることを述べています。
0926132人目の素数さん垢版2023/02/04(土) 21:48:23.92ID:B4ctykqs
松坂さんは定理13が書かれているセクションの前のセクションの演習問題として、
以下の問題を出題しています:

問題9:
S を完備な距離空間とするとき、 S の部分集合 M が部分距離空間として
完備であるためには、 M が S の閉集合であることが必要十分である。
このことを証明せよ。


この問題から、

>>925

closure(h(X)) は h(x) を含む最小の完備距離空間であることが分かります。

松坂さんはなぜMunkresさんのような書き方にしなかったのでしょうか?
いかにも松坂さんが好みそうな書き方だと思います。
0927132人目の素数さん垢版2023/02/04(土) 22:04:47.02ID:B4ctykqs
松坂和夫著『集合・位相入門』

第6章§5が「ノルム空間、Banach空間」です。

このセクションって要りますか?

関数解析で重要だと書いてありますが、それなら関数解析の本で勉強すればいいのではないでしょうか?
短いセクションで紹介しても意味があるとは思えません。
0929132人目の素数さん垢版2023/02/04(土) 22:46:32.85ID:pdmhk4kD
松坂の本のその部分は読んだ記憶にもないので確かになくてもいいかもしれないですね
0930132人目の素数さん垢版2023/02/04(土) 22:58:32.13ID:YdHVzBVi
>>922
はい、このような記述方法は専門的な内容を扱っているために一般的な読者にとっては分かりにくいかもしれません。
0931132人目の素数さん垢版2023/02/04(土) 22:59:22.15ID:YdHVzBVi
>>923
はい、このような記述方法は明確で分かりやすいため、一般的な読者にも理解しやすいと思います。
0932132人目の素数さん垢版2023/02/04(土) 23:00:56.56ID:YdHVzBVi
>>925,926
It is not possible to determine why Professor Matsuzaka chose to present the information in a different manner from Professor Munkres without additional context or communication with him. It is possible that he had different teaching goals, target audience, or preferred method of presentation. However, it is important to note that both presentations convey the same mathematical concepts and ideas, just in different ways.
0933132人目の素数さん垢版2023/02/04(土) 23:02:07.06ID:YdHVzBVi
>>927
それはあなたの使用する分野や目的によって異なります。一般的に、関数解析の分野では、ノルム空間やBanach空間が重要な役割を果たします。しかし、他の分野ではこれらのセクションは不要な場合もあります。最終的には、その本があなたにとって必要かどうかは、あなたの求める情報や目的に基づいて決定すべきです。
0934132人目の素数さん垢版2023/02/05(日) 01:01:51.59ID:n0xy5ODP
>>928
大発見ですね
0935132人目の素数さん垢版2023/02/05(日) 09:35:08.04ID:n0xy5ODP
>>924
そこが完備化の本質では?
0936132人目の素数さん垢版2023/02/05(日) 09:44:04.33ID:+9r6/2Xj
「本質的にただ一つ存在する」の部分のことを言うのなら(普遍性に慣れてなければ)まだわかる
ただ(iii)を見て「え?」とはならんだろ
0937132人目の素数さん垢版2023/02/05(日) 10:14:59.81ID:S+ZlN+mD
松坂さんの定理13の書き方は窮屈すぎます。

S を含む完備距離空間として、 closure(S) = S* となる完備距離空間 S* をいきなり登場させます。

Munkresさんの書き方では、まず X を含む完備距離空間 Y があることをまず述べています。
次に、完備距離空間 Y の部分空間 X の閉包が完備距離空間であることを述べています。

Munkresさんの書き方のほうが優れています。
0938132人目の素数さん垢版2023/02/05(日) 12:27:29.76ID:n0xy5ODP
>>937
はい、Munkresさんの書き方の方が分かりやすいということですね。各人の好みは異なりますが、理解しやすい方法を選ぶことが大切です。
0939132人目の素数さん垢版2023/02/05(日) 12:53:41.00ID:n0xy5ODP
>>936
そこは両者でどう証明されてるんでしょうね
Munkresの方はそのコメントが無いようですが
どう示されてますか?>>937
0940132人目の素数さん垢版2023/02/05(日) 14:20:38.05ID:S+ZlN+mD
>>939

松坂さんの方の一意性の部分は、具体的には、以下の「くわしくいえば」以降を証明しています。

定理13
(S, d) を与えられた距離空間とするとき、次の性質(i)、(ii)、(iii)を満足する
距離空間 (S*, d*), および S から S* への写像 φ が存在する。

(i) (S*, d*) は完備である。
(ii) 任意の x, y ∈ S に対して d(x, y) = d*(φ(x), φ(y)).
(iii) φ(S) は S* において密である。すなわち closure(φ(S)) = S*.

また、このような距離空間 (S*, d*) と写像 φ : S → S* との組 ((S*, d*), φ) は、与えられた
(S, d) に対して本質的にはただ1つだけ存在する。くわしくいえば、次のことが成り立つ:
上の ((S*, d*), φ) とともに、今1つの距離空間 (S2*, d2*) と写像 φ2 : S → S2* との組
((S2*, d2*), φ2) もやはり上の(i), (ii), (iii)を満たすならば、 S* から S2* への全単射 f で、

f ・ φ = φ2,

かつ、任意の x*, y* ∈ S* に対し

d*(x*, y*) = d2*(f(x*), f(y*))

となるものが存在する。
0941132人目の素数さん垢版2023/02/05(日) 14:23:37.21ID:S+ZlN+mD
Munkresさんのほうは、一意性は演習問題に回されていて、以下のようになっています:

p.271 Exercise 10.
Theorem (Uniqueness of the completion).

Let h : X → Y and h' : X→ Y' be isometric imbeddings of the metric space (X, d) in
the complete metric spaces (Y, D) and (Y' D'), respectively. Then there is an isometry of
(closure(h(X)), D) with (closure(h'(X)), D') that equals h' ・ h^{-1} on the subspace h(X).
0942132人目の素数さん垢版2023/02/05(日) 14:29:02.91ID:n0xy5ODP
>>941
演習問題でしか採り上げないことは(今回は)否定しないのですね
0943132人目の素数さん垢版2023/02/06(月) 04:37:09.78ID:hJqwXeGt
標数p > 0の体の円分体は巡回拡大体ではないのですぺゃ?
0944132人目の素数さん垢版2023/02/06(月) 09:11:22.29ID:PjLysmxt
素体の上に1の冪根を添加した体を円分体と呼ぶなら、正標数の円分体は全部有限体では?
0945132人目の素数さん垢版2023/02/06(月) 10:00:59.29ID:f2a0B9KR
テンソル悩んでるけど
線形性と双線型性がよくわからない
ベクトルの組(a,b)について
内積をf(a,b)=a•bとすると
f(a+a',b)
=f((a,b)+(a',0))
=f(a,b)+f(a',b)
となって
ベクトルの組そのままだと内積との対応が何やら気持ち悪いから
a⊗bっていう新しい軸を考えて
(a+a')⊗b=a⊗b+a'⊗b
ってなるようにすれば
f(a,b)=g(a⊗b)とした時に
f(a+a',b)=g(a+a'⊗b)みたいになって気持ちよく対応してくれるよね
みたいな話?
0946132人目の素数さん垢版2023/02/06(月) 10:34:43.89ID:Z3UpM1dx
>>945
>みたいな話?
みたいな話
0947132人目の素数さん垢版2023/02/06(月) 10:37:42.04ID:Z3UpM1dx
>>945
>ベクトルの組そのままだと内積との対応が何やら気持ち悪いから
どう気持ち悪い?
0948132人目の素数さん垢版2023/02/06(月) 10:58:27.59ID:AB/1OnMc
>a⊗bっていう新しい軸を考えて
テンソルの構成法には、テンソル積による構成法もあるが、それだけではダメだんだな
0949132人目の素数さん垢版2023/02/06(月) 11:14:36.40ID:D6lSg6Um
西野利雄著『多変数函数論 増補新装版』

を注文しました。

まずは1変数の複素関数論を勉強しないとだめですね。
0950132人目の素数さん垢版2023/02/06(月) 11:45:33.49ID:Z3UpM1dx
>>949
はい、1変数の複素関数論は多変数函数論の基礎となるので勉強することが重要です。
0951132人目の素数さん垢版2023/02/06(月) 11:53:44.53ID:D6lSg6Um
距離空間の完備化ですが、素朴なアイディアですね。

(x_n), (y_n) を距離空間 S のコーシー点列とする。
R の数列 d(x_n, y_n) はコーシー列になる。
R は完備だから、 d(x_n, y_n) は収束する。

d((x_n), (y_n)) := lim d(x_n, y_n) と定義すると、

d はほとんどの距離の公理を満たす。

d(x, y) = 0 ⇔ x = y

という性質だけ満たさない。

この性質が満たされるようにするために、

(x_n) 〜 (y_n) :⇔ lim d(x_n, y_n) = 0

と定義すると、「〜」は S のコーシー点列からなる集合上の同値関係になり、
(コーシー点列からなる集合)/〜 は距離空間になる。

みたいな話ですよね。

非常に素朴です。
0952132人目の素数さん垢版2023/02/06(月) 11:56:30.20ID:D6lSg6Um
S の元 x に対し、 x, x, x, … という S の点列はコーシー点列を対応させる。

x, x, x, … の属する同値類は、 x に収束する S の点列全体の集合に等しい。

非常に素朴ですね。
0953132人目の素数さん垢版2023/02/06(月) 18:28:10.42ID:b8cUv6ow
L/Kを体の拡大
標数はp > 0

L/Kが純非分離拡大

(1) 任意のα∈L\Kは、K上分離的ではない
(2) 任意のα∈Lに対して、あるn≧0が存在して、α^(p^n)∈K
(3) 任意のα∈Lに対して、αのK上の最小多項式は、あるa∈Kがあって、X^(p^n) - aの形

(1)と他の同値性がわからのい
純非分離ではなく、ただ分離的でないだけの元の最小多項式は、X^(p^n) - aの形とは限らないにょえ?
0954132人目の素数さん垢版2023/02/06(月) 19:16:59.83ID:cDpYbQWZ
(2)⇒(3)
あるnとあるa∈Kがあって、α^(p^n) - a = 0。このように書ける最小のnを取れば、αの最小多項式はX^(p^n) - α。

(3)⇒(1)
α∈L\Kの最小多項式はX^(p^n) - αの形で、微分してみれば明らかに重根持つので、αは分離的でない。

(1)⇒(2), (3)
分離的でない元は、十分大きなnに対して、p^n乗すると分離的になる
(∵ 分離的でない元の最小多項式はX^pの多項式になる。これを適当なnについてX^(p^n) = YとおいてYの多項式として見たら分離多項式になる)
(1)よりそれはKの元なので、(2)と(3)が言える
0955132人目の素数さん垢版2023/02/06(月) 21:12:41.35ID:TWnugaGY
(3)→(1)は自明
(3)でないとすれば、あるaの最小多項式f(x)で(3)を満たさないものが取れる
f(x) = Σcₖxᵏ
としvをp進付値、m= { v(k) | cₖ ≠ 0 }としてb = a^(p^m)とおけばbは多項式
g(x) = Σdₖx^(k/p^m)
の解で拡大次数を評価してg(x)はbの最小多項式
特にg'(x)と共通根を持たない
一方でdₖの作り方からいずれかのpの倍数でないkにおいてdₖ≠0
特にg'(x)≠0
さらに元々f(x)が(3)を満たさない事からdeg(f)=p^mではないのでg(x)は一次式ではない
∴K(b)/Kは分離拡大
0956132人目の素数さん垢版2023/02/06(月) 21:35:32.87ID:b8cUv6ow
なるほど
自分で理解できるように手直ししましゅ
ありがとうございます
0957132人目の素数さん垢版2023/02/06(月) 21:40:46.29ID:b8cUv6ow
うをぉー……
自分で1日考えたあとに永田先生の「可換体論」読むと、感動するな
この本最短経路で突っ走ってる。つか羽生えて飛んでるわ
0958132人目の素数さん垢版2023/02/07(火) 10:49:11.32ID:qxbE0/qv
松坂和夫著『集合・位相入門』

点列コンパクトの定義ですが、

「S の任意の点列は収束する部分列を含む。」

となっています。

これって、「S の任意の点列は S の点に収束する部分列を含む。」

と書いたほうがいいですよね。

S が距離空間 T の部分距離空間であるときに、 S の任意の点列が
T の点に収束する部分列を含む場合でも、 S は点列コンパクトということ
になってしましますよね。
0959132人目の素数さん垢版2023/02/07(火) 11:27:48.04ID:/uTnEvxf
そうですね
0960132人目の素数さん垢版2023/02/07(火) 11:38:52.99ID:E2hVVy9J
>>958
はい、その通りです。「S の任意の点列は S の点に収束する部分列を含む。」と表現するほうがわかりやすいです。また、S が距離空間 T の部分距離空間であり、かつ S の任意の点列が T の点に収束する部分列を含む場合、S は点列コンパクトであると言えます。
0961132人目の素数さん垢版2023/02/07(火) 14:35:31.81ID:CgrfKBAT
T = R
S = (0, 1)⊂Tは、点列コンパクト??
0963132人目の素数さん垢版2023/02/07(火) 17:14:48.76ID:L+AEQ/DF
>>962
自演ですよね?
0965132人目の素数さん垢版2023/02/08(水) 12:06:56.05ID:XBp4A4ka
多様体 (共立数学講座) 単行本 – 1989/3/25
村上 信吾 (著)

偏微分方程式入門 (基礎数学) 単行本 – 1998/2/1
金子 晃 (著)

を注文しました。

金子さんの偏微分方程式の本は厳密ですか?
0966132人目の素数さん垢版2023/02/08(水) 12:10:44.30ID:33WojhiJ
いっぱい本を買って、出版社の延命を助けてあげてね
0967132人目の素数さん垢版2023/02/08(水) 18:45:27.39ID:XBp4A4ka
松坂和夫著『集合・位相入門』

ついに、第6章§5「ノルム空間、Banach空間」を読み終わりました。
あとは演習問題と§6「Urysohnの距離づけ定理」のみとなりました。

大団円までもう少しですね。
0968132人目の素数さん垢版2023/02/08(水) 18:50:07.08ID:XBp4A4ka
第6章§5「ノルム空間、Banach空間」の演習問題は長いので、とりあえず飛ばして
§6「Urysohnの距離づけ定理」を読もうと思います。
0969132人目の素数さん垢版2023/02/08(水) 22:24:41.14ID:aHZHh+UU
君はキレイだから 黙ってればなんも問題ない
だけど口を開きゃ ちょっとばかりへこむ様なこと 増えてゆく
ヨーロッパは国じゃない ロバは成長してもウマじゃない
“縁日"を“ミドリのヒ "1ダースを“5個か10個だ"と 云わないで
恋心 萎えちゃう日もあるけど
誰もが振り向く 君をフるにゃ惜しいよ
0970132人目の素数さん垢版2023/02/09(木) 05:26:10.26ID:mBvSfK7C
Gを群
N_1, N_2をGの正規部分群
このときN_1∩N_2はGの正規部分群か
0971132人目の素数さん垢版2023/02/09(木) 05:43:33.63ID:vQg8mTKo
まずN_1∩N_2はGの部分群である
a, b∈N_1∩N_2とすると、N_1, N_2はGの部分群であるから、ab, a^(-1)∈N_1∩N_2

g∈Gとする
gN_1∩N_2g^(-1) = N_1∩N_2を示す

gag^(-1)∈gN_1∩N_2g^(-1)を任意に取る (a∈N_1∩N_2)
N_1, N_2は正規部分群なので、gag^(-1)∈N_1∩N_2

a∈N_1∩N_2を任意に取る
N_1, N_2は正規部分群なので、a∈gN_1g^(-1)∩gN_2g^(-1) = gN_1∩N_2g^(-1)
0972132人目の素数さん垢版2023/02/09(木) 05:48:06.60ID:/4I6aUTm
HがGの正規部分群なら剰余集合G/Hは
aH bH = abH (a, b∈G)
により群になることを示せ

正規部分群でない場合はどうか
0973132人目の素数さん垢版2023/02/09(木) 06:10:31.90ID:p7rr0vAI
aH = a'H, bH = b'H とする。
g = b^(-1)a^(-1)a'b'∈Hを示す。
a^(-1)a'∈Hなので、g∈b^(-1)Hb'。
Hは正規部分群なので、Hb' = b'H = bH = Hb、かつb^(-1)Hb = H。よって、g∈b^(-1)Hb = H。

Gが群なので、この演算でG/Hが群になることは明らか。
0974132人目の素数さん垢版2023/02/09(木) 06:45:54.26ID:p7rr0vAI
正規部分群ではない場合。
たとえばGを3次対称群S_3、Hは互換(1 2)で生成される巡回群とする。
|G| = 6, |H| = 2だから、|G/H| = 3。
(3 1)(2 3)^(-1) = (3 1)(2 3) = (1 2 3)∉Hなので、G/Hの代表系として

H, (2 3)H, (3 1)H

が取れる。

(2 3)(1 2) = (1 2 3)
(3 1)(1 2) = (1 3 2) = (1 2 3)^2

なので、(2 3)H = (1 2 3)H, (3 1)H = (1 2 3)^2H。

(3 1)(2 3) = (1 2 3)

だが

(1 2 3)(1 2 3)^2 = e

(1 2 3)H = (2 3)H ≠ Hなので、aH bH = abHはwell-definedではない。
0975132人目の素数さん垢版2023/02/09(木) 10:38:14.23ID:IKCpZgdk
>>974
>正規部分群ではない場合
ダウツ
これでは
正規部分群では無い場合に商は必ずしも群にならない証明なだけ
正規部分群では無い場合に商は決して群にならないことを示せ
0976132人目の素数さん垢版2023/02/09(木) 10:45:28.84ID:PK04NLu5
まぁしかし>>972の設問なら

>正規部分群でない場合はどうか

この一文では

必ず正規部分群になるか?

と解釈されて文句言えないのでそれを根拠に>>974を減点する事はできんやろな
0977132人目の素数さん垢版2023/02/09(木) 10:47:47.00ID:PK04NLu5
違うわ

正規部分群でない場合にも必ず群として定義されてるか

と解釈されて文句言えないや
もし正規部分群でなければ群として定義されない事まで証明するさせるつもりなら問題にその旨明示しないといけない
0978132人目の素数さん垢版2023/02/09(木) 10:57:44.78ID:lqFsfOvJ
もしかして、Gの部分群Hが
aH bH = abH (a, b∈G)
を満たすならHが正規部分群となることを示せ
と言ってる?
0979132人目の素数さん垢版2023/02/09(木) 11:33:01.13ID:P/1EbLN9
杉浦光夫他著『連続群論入門』がちくま学芸文庫からそのうち出版されると予想します。

当たりますかね?
0980132人目の素数さん垢版2023/02/09(木) 11:35:11.42ID:IKCpZgdk
>>978
他の方法でも良いけど
直接にはそれ
0981132人目の素数さん垢版2023/02/09(木) 14:00:54.57ID:QcvTyUvQ
>>975
何がダウツだ
アホかお前

「H⊂G正規部分群じゃない⇒G/Hは群にならない」
を意図しているなら質問者がそう補足すればいいだけ
外野が出しゃばんな
0982132人目の素数さん垢版2023/02/09(木) 14:09:36.92ID:QcvTyUvQ
質問者「f実数から実数への連続関数で、f(0) = -1、f(1) = 1をみたすものとします。この時、f(x) = 0をみたすxは存在しますか?fが連続でない場合はどうですか?」

回答者「中間値の定理からそのようなxは-1から1の範囲に必ず存在します。fが連続でない場合は反例があります。たとえば、f(x) = -1 (x≦0), f(x) = 1 (x > 1)がそうです」

外野「ダウツこれではfが連続でない場合は必ずしもf(x) = 0をみたすxが存在しない証明だけ連続でない場合に決してf(x) = 0になるxが存在しないことを示せ」

こいつアホすぎ
0983132人目の素数さん垢版2023/02/09(木) 14:11:08.63ID:jy674GK2
>>981
おまえ誰?w
外野が出しゃばんなや
0984132人目の素数さん垢版2023/02/09(木) 14:12:59.81ID:jy674GK2
>>982
そっちとは逆向きの質が違うって分かってなかったか
0985132人目の素数さん垢版2023/02/09(木) 14:18:07.99ID:QcvTyUvQ
イキって指摘したつもりが間違えちゃって恥ずかしかったんだね
わかるよその気持ち
でもそういう時は素直に謝ろうね
もういい歳なんだから大人になろう
0986132人目の素数さん垢版2023/02/09(木) 14:43:46.18ID:jy674GK2
>>985
おやまぁ
あれを指摘だと思うとは
0987132人目の素数さん垢版2023/02/09(木) 14:46:35.11ID:jy674GK2
ほんま
出しゃばりの外野がなけなしの知恵を絞るだけでんな
0988132人目の素数さん垢版2023/02/09(木) 14:48:24.78ID:nJ/ogibl
ID:jy674GK2
自分より馬鹿な人間がいて安心する
0989132人目の素数さん垢版2023/02/09(木) 14:49:01.95ID:jy674GK2
>>985
まあエエから
逆を証明してみ
気持ち分かるから
0990132人目の素数さん垢版2023/02/09(木) 14:57:17.36ID:jy674GK2
>>988
エスパーも増えたなw
0992132人目の素数さん垢版2023/02/09(木) 21:25:54.90ID:mDOdi02e
IDを変えても一人…って感じだよな
孤独老人なのかスレを盛り上げようとしてくれてるのかただの真性なのか
0993132人目の素数さん垢版2023/02/10(金) 00:15:38.15ID:Cq5KZojD
>>992
>>928
0994132人目の素数さん垢版2023/02/10(金) 00:16:37.34ID:Cq5KZojD
>>991
不思議な感覚ですね
0995132人目の素数さん垢版2023/02/10(金) 12:09:52.65ID:4ryvycCW
それが5ch
0996132人目の素数さん垢版2023/02/10(金) 12:11:31.55ID:4ryvycCW
5chは負け犬が堕ちる処
0997132人目の素数さん垢版2023/02/10(金) 12:12:22.30ID:4ryvycCW
勝ち馬は5chに書かない
0998132人目の素数さん垢版2023/02/10(金) 12:13:03.87ID:4ryvycCW
書く暇も動機もない
0999132人目の素数さん垢版2023/02/10(金) 12:14:37.82ID:4ryvycCW
負け犬が有り余る暇をもて余し
憎しみを叩きつける処
それが5ch
1000132人目の素数さん垢版2023/02/10(金) 12:14:44.17ID:fS4+PW1o
あほ
10011001垢版Over 1000Thread
このスレッドは1000を超えました。
新しいスレッドを立ててください。
life time: 79日 23時間 59分 24秒
10021002垢版Over 1000Thread
5ちゃんねるの運営はプレミアム会員の皆さまに支えられています。
運営にご協力お願いいたします。


───────────────────
《プレミアム会員の主な特典》
★ 5ちゃんねる専用ブラウザからの広告除去
★ 5ちゃんねるの過去ログを取得
★ 書き込み規制の緩和
───────────────────

会員登録には個人情報は一切必要ありません。
月300円から匿名でご購入いただけます。

▼ プレミアム会員登録はこちら ▼
https://premium.5ch.net/

▼ 浪人ログインはこちら ▼
https://login.5ch.net/login.php
レス数が1000を超えています。これ以上書き込みはできません。

ニューススポーツなんでも実況